Vous êtes sur la page 1sur 110

1.

A 52-year-old man presents with bright red blood ‫   وه    دم‬52  ‫ر‬ D
per rectum. He states that he has been bleeding ‫"ة‬# ‫ف‬%& ‫ *" أد أ) آن‬.‫أ   اج‬
heavily for a couple of hours. In the ER, his pulse is ‫ ه‬/.&‫ ا‬،‫ارئ‬2‫ ا‬345 6 .‫ت‬, -.
110beats/min, blood pressure is 90/50mmHg, he is >?‫ و‬،@.A‫ ز‬3C 50/90 ‫ ا"م ه‬:;< ،‫ د‬/110
cool and clammy appearing, and he has blood DC ،‫از‬.‫ ا‬EF 6 ‫>ر دم‬G - ،‫رد و"ي‬#
present on rectal examination, although he does not I5‫ ا‬6 ‫ف‬%& )‫) أ‬JC ‫"و‬. K )‫  أ‬3L‫ا‬
appear to be bleeding at the present. Which of the :‫؟‬N‫ ا‬OJ‫و‬N‫ة ا‬2P‫ ا‬6‫ ه‬6Q‫ أي  ا‬.<F‫ا‬
following is the best initial next step ? .‫ ا*ن‬J?& .A
A. Colonoscopy .‫ ن‬6&J, J?& .B
B. Flexible sigmoidoscopy .‫"ي‬S 6T‫ب أ‬.‫ أ‬-<‫ و‬.C
C. Place a nasogastric tube .6FC ‫* ور"ي‬F# ‫"ء‬.‫ ا‬.D
D. Start an of IV normal saline 9% .6.C, O OW  ‫ * دم‬.E
E. Give a transfusion of type O-negative blood

Un homme de 52 ans présente pour une rectorragie


en grande quantité. Il déclare avoir saigné pendant
quelques heures. Aux urgences, son pouls est de
110 battements / min, sa pression artérielle est de
90 / 50mmHg.Il est froid et moite, et il a du sang à
l'examen rectal, bien qu'il ne semble pas saigner
pour le moment. Laquelle des propositions
suivantes, constitue la meilleure étape initiale pour
la prise en charge du patient?
A- Coloscopie
B- Sigmoïdoscopie flexible
C- Installer une sonde nasogastrique
D- Commencer un sérum salé 9% en I.V
E- Donner une transfusion de sang de type O
négatif
2. A 25-year-old man has a colonoscopy for ‫ن  أ‬5 J?&Q -X  25  ‫ر‬ A
diagnostic evaluation of abdominal pain, weight ‫ ودم‬،‫>ل‬,‫ إ‬،‫رة وزن‬4X ،6&2# 3N 6[JP 3JJ*
loss, diarrhea, and blood in the stool. The OJ\P ‫>ب‬Q‫ ا*ن ا‬J?& >G‫ أ‬.‫از‬.‫ ا‬6
colonoscopy shows diffuse mucosal inflammation ‫ ه‬6Q‫ أي  ا‬."[‫ اج وا*ن ا‬6 Q&
in the anus and descending colon. Which of the :‫؟‬FJ ]‫آ‬N‫ ا‬EJPQ‫ا‬
following is the most likely diagnosis? .‫*ح‬Q‫>ب ا*ن ا‬Q‫ ا‬.A
A. Ulcerative colitis .‫ داء آوه‬.B
B. Crohn disease .‫ اذب‬6AL ‫ن‬5 ‫>ب‬Q‫ ا‬.C
C. Pseudomembranous colitis .‫ن‬5 ‫\ن‬, .D
D. Colon cancer

Un homme de 25 ans a une colonoscopie pour


l'évaluation diagnostique d’une douleur
abdominale, avec perte de poids, diarrhée et sang
dans les selles. La coloscopie montre une
inflammation diffuse de la muqueuse dans l'anus et
le côlon descendant. Lequel des diagnostics
suivants est le plus probable?
A- La colite ulcéreuse
B- La maladie de Crohn
C- Colite pseudomembraneuse
D- Cancer du côlon

1A
3. A 56-year-old man with known chronic kidney ،% ‫ي‬C‫>ب آ‬Q‫  ا‬6S  56  ‫ر‬ B
disease presents with a 3-day history of shortness ‫ زدة‬6 O,‫ و‬aT [5   ‫ وه‬
of breath and rapid weight gain. On examination, DC -4Q‫  ا‬،EFT‫ &" ا‬.‫ أم‬3 b& ‫ازن‬
you are able to ausculate an S3, hear crackles at the :,Q ‫خ‬TQ‫ ا‬O‫ ورؤ‬،"‫ ا*ا‬6 ‫ع آاآ‬, ،S3
bases, and see moderate jugular venous distension OJQ‫ة ا‬2P‫ ا‬6‫ ه‬6Q‫ أي  ا‬.6‫ ار" ادا‬6
(JVD). Which of the following is your next step in :‫؟‬3JJ*QQ‫ ا‬6
evaluation? .gC*‫"ى ا‬i :J2P ‫ إاء‬.A
A- Perform an echocardiogram .‫["ر‬C OJ&J, OSj‫ إاء أ‬gC\ .B
B- Order a chest x-ray JjQ‫"ل ا‬S ‫ب‬4F J&‫س ا‬J5 .C
C- Measure a creatinine to calculate renal .6.J.‫ا‬
glomerular filtrate .OJ.C*‫ ا‬AP‫*@  ا‬FQ‫ ا‬.D
D- Check for cardiac enzymes

Un homme de 56 ans avec une insuffisance rénale


chronique connue se présente pour une histoire de 3
jours d'essoufflement et de gain de poids rapide. A
l'examen, vous êtes en mesure d'ausculter un S3,
d'entendre des craquements à la base et de voir une
distension veineuse jugulaire modérée. Lequel des
éléments suivants, constitue votre prochaine étape
d'évaluation?
A- Effectuer un échocardiogramme
B- Demandez une radiographie pulmonaire
C- Mesurer la creatinine pour calculer la
filtration glomérulaire rénale
D- Vérifiez les enzymes cardiaques
4. A 39-year old woman with multiple medical ،‫"دة‬SQ OJ.\ ‫  آ‬6S  39 ‫اأة ه‬ C
problems has been noted to have progressively .6l‫ "ر‬# ‫ء‬4 ‫ي‬C‫ أن ا*[ر ا‬m "*‫و‬
worsening renal insufficiency. Which of the -& 6 OJ‫آ] أه‬N‫ ه ا‬OJQ‫ ا‬JS‫أي  ا‬
following measures is most important in the :‫ي؟‬C‫  ا"اء ا‬OJA>&‫ ا‬OC‫ ا‬D‫ل إ‬i‫ا‬
prevention of end-stage renal disease ? .JX"Q‫  ا‬n5Q‫ ا‬.A
A. Tobaco cessation ."J4JC;Q‫ ا‬O.5‫ ا‬.B
B. Triglyceride control .‫ ا"م‬, O.5‫ ا‬.C
C. Glycemic control .‫ ازن‬O.5‫ ا‬.D
D. Weight control .‫  ا[دم‬OJF# "JJ*Q‫ ا‬.E
E. Dietary sodium restriction

Une femme de 39 ans avec de multiples problèmes


médicaux a été notée pour une insuffisance rénale
qui s’aggrave progressivement. Laquelle des
mesures suivantes est la plus importante dans la
prévention de l'insuffisance rénale terminale?
A- Arrêt du tabac
B- Contrôle des triglycérides
C- Contrôle glycémique
D- Contrôle du poids
E- Restriction du sodium alimentaire

2A
5. A 45-year-old woman with type 2 diabetes presents ،2 : ‫ي‬, ‫  داء‬6S  45 ‫اأة ه‬ A
to the clinic with decreased vision in the left for 1 6 E5& >" ‫ أن‬J. qJ ‫دة‬JS‫ ا‬D‫ت إ‬
year, 1 + proteinuria, a baseline creatinin of ،1+ J‫و‬.‫ ا‬OCJ# ،O&, b& ‫ى‬4J‫ ا‬JS‫ ا‬6 O‫اؤ‬
1.6mg/dL, a low-density lipoprotein (LDL) of 135 ‫ ذات‬6F‫ ا‬J‫و‬.‫ ا‬،,‫ د‬/sC 1.6 J&‫آ‬
mg/dL, blood pressure of 145/92mmHg, and ‫ ا"م‬:;< ،,‫ د‬/sC 135 OTP&‫ ا‬O]‫ا‬
occasional chest pain for the past 2 months. Which >‫ ا‬6 -2*Q ‫"ري‬i 3‫ وأ‬،@.A‫ ز‬3C 92/145
of the following is the best medication to start the )# ‫"أ‬. ‫ ه أ دواء‬6Q‫ أي  ا‬.JXN‫ا‬
patient on at this time ? :‫ن؟‬t‫ ا‬b& O‫ا‬
A. ACE inhibitor .O.C*&‫ ا‬J4&lN‫ة ا‬JX :.] .A
B. β-blocker .QJ# g .B
C. Oral nitrate .3T‫ات  \@ ا‬Q .C
D. Thiazide diuretic .‫ز"ي‬J ‫ "ر‬.D

Une femme de 45 ans atteinte de diabète de type 2


se présente à la clinique avec une diminution de la
vision à gauche depuis 1 an, protéinurie 1+, une
creatinine de base de 1,6 mg / dL, une lipoprotéine
de basse densité (LDL) de 135 mg / dL, une
pression artérielle de 145 / 92mmHg, et des
douleurs thoraciques occasionnelles au cours des 2
derniers mois. Lequel des médicaments suivants est
le meilleur?
A- Inhibiteur de l'ECA
B- β-bloquant
C- Dérivés mitrés oral
D- Diurétique thiazidique

3A
6. A 30-year-old woman with no past medical history OJ< ‫ة‬J, >" aJ  30 ‫اأة ه‬ D
presents with a productive cough of 2-week ‫ة‬Q b& uQ& ‫ل‬S,   6‫ ت وه‬،O*#,
duration. She states she also has a runny nose,body ‫ن‬vJ,  ‫ أ‬6S >w# ‫ *" أدت‬.J.,‫أ‬
aches, congestion, and fevers for the past week. In ‫ع ارة‬T‫*ن وار‬Q‫ إ‬،>4 ‫ء‬F‫ أ‬6 ‫م‬Kx ،6T‫أ‬
office she is normotensive, with a normal pulse, ،6SJ.\ :;‫دة آن ا‬JS‫ ا‬6 .6<‫ع ا‬.,N‫ ا‬b&
and temperature of 38.3°C. Her physical EFT‫  ا‬J. .‫س‬°38.3 ‫ارة‬F‫ وا‬،6SJ.\ /.&‫ا‬
examination is significant for sinus tenderness, OJTN‫رة ا‬F‫ ا‬،OJTN‫ب ا‬Jl‫ ا‬6 ‫م‬v‫ي إ‬4‫ا‬
congested nasal turbinates, and crackles in the left 6CT4‫ ا‬4N‫ ا‬ET‫ *ل ا‬6 ‫ وآاآ‬،OJS*&Q4
lower lobe lung fields. Which one of the following 6 N‫ ا‬D‫و‬N‫ة ا‬2P‫ ا‬6‫ ه‬6Q‫ أي  ا‬.OAC
is the best initial step in management ? :‫؟‬6vS‫ ا‬J#"Q‫ا‬
A. Reassure the patient that she likely has a ‫ر‬N‫ ا‬DC >Q#i‫ن إ‬w# O‫ ا‬Ow\ .A
viral infection and it will resolve on its own .>4T ‫*ء‬C  DTQ,‫ و‬OJ,‫و‬J
B. Order a rapide strep test and treat if positive ‫*"ت‬S‫  ا‬-, ‫ي‬F EF gC\ .B
C. Prescribe amoxicillin for a likely bacterial .OJ#l‫ إ‬OlJQ&‫ ا‬I‫ إذا آ‬OlS‫وا‬
infection O#i{‫ ا‬J g.4# JCC4J4‫ أآ‬ni‫ و‬.C
D. Order chest x-ray to rule out possible .OJ|l‫ا‬
pneumonia ‫د‬S.Q,K ‫["ر‬C OJ&J, OSj‫ إاء أ‬gC\ .D
.‫ي‬A‫>ب ر‬Q# O#i{‫ل ا‬Q‫ا‬
Une femme de 30 ans sans antécédents médicaux
présente une toux productive qui date de 2
semaines. Elle déclare avoir également un nez qui
coule, des courbatures, une congestion et de la
fièvre au cours de la dernière semaine. Au bureau,
elle est normotendue, avec un pouls normal et une
température de 38.3 ° C. Son examen physique
révèle une sensibilité des sinus, des cornets nasaux
congestionnés et des crépitements dans les champs
pulmonaires du lobe inférieur gauche. Laquelle des
propositions suivantes est la meilleure première
étape dans la prise en charge de la patiente?
A- Rassurer la patiente qu'elle a probablement
une infection virale et qu'elle se résoudra
d'elle-même
B- Commander un test streptococcique rapide
et traiter si positif
C- Prescrire l'amoxicilline pour une infection
bactérienne probable
D- Demander une radiographie pulmonaire
pour exclure une éventuelle pneumonie

4A
7. A 35-year-old G2P1 at 11 weeks’gestational age 11 ‫ع‬.,N‫ ا‬6 G2P1   35 ‫اأة ه‬ A
presents with complaint of palpitations, weight ‫رة‬4X ،‫*ن‬TX   6‫ ت وه‬،F‫ ا‬
loss, nervousness and tremor. She denies prior *#, ><S IT "* .‫ف‬l‫اب وار‬2<‫ ا‬،‫وزن‬
history of thyroid problems. Laboratory studies TSH ‫ أن‬O.P‫ت ا‬,‫ أآ"ت ا"را‬.OJ5‫آ در‬
confirm that TSH is severely suppressed. Which of b> ‫ج‬v ‫ ه أ‬6Q‫ أي  ا‬.‫"ة‬# E5&Q
the following is the best treatment for this patient at :‫؟‬I5‫ا ا‬b‫ ه‬6 O‫ا‬
this time ? .PTU .A
A. PTU .QJ# ‫ت‬. .B
B. β-blockers .J4‫وآ‬JTJ .C
C. Levothyroxine .‫زول‬JQ .D
D. Methimazole

Une femme de 35 ans G2P1 à 11 semaines d’ȃge


gestationnel se présente avec des palpitations, une
perte de poids, une anxiété et un tremblement. Elle
nie avoirdes antécédants personnels de problèmes
thyroϊdien. Les examens de laboratoire confirment
que la TSH est très diminuée. Lequel des énoncés
suivants représente le meilleur traitement pour cette
patiente ?
A- PTU
B- β-bloquants
C- Levothyroxine
D- Methimazole
8. A 12-year-old presents with eye itching and 6 O   ‫   وه‬12  DQ C
redness. He has clear drainage from his eyes but no ‫) و‬J&J  <‫ وا‬n[ )" .‫ واار‬JS‫ا‬
crusting. Examination today is normal except for OFQC ‫>ب‬Q‫]&ء ا‬Q,# ،6SJ.\ EFT‫ ا‬.* ‫دون‬
mildly injected conjunctiva bilaterally. Which of ‫ج‬vS‫ ه ا‬6Q‫ أي  ا‬.gl‫ ا‬6A&| nJTX 6*Q‫ا‬
the following is the most appropriate treatment ? :‫؟‬N‫ا‬
A. Antibiotic eye drops .OJ&J ‫ة‬25 ‫ي‬J ‫ د‬.A
B. Ophthalmology consultation .‫ن‬J gJ.\ ‫رة‬Q,‫ ا‬.B
C. Anti-inflammatory eye drops .OJ&J ‫ة‬25 ‫>ب‬Q‫ د ا‬.C
D. Oral leukotriene inhibitaor .3T‫  \@ ا‬J‫ آ‬:.] .D

Un enfant de 12 ans présente des picotements et


une rougeur aux yeux. Il a un drainage clair de ses
yeux mais pas de croûtes. L'examen est aujourd'hui
normal sauf pour une conjonctive légèrement
injectée bilatéralement. Lequel des énoncés
suivants est le traitement le plus approprié?
A- Gouttes d’antibiotiques pour les yeux
B- Consultation ophtalmologique
C- Gouttes anti-inflammatoires pour les yeux
D- Inhibiteur oral des leucotriènes

5A
9. A 5-year-old child is brought into a pediatrician’s ‫ل‬T\N‫ ا‬gJ.\ ‫دة‬J D‫&ات إ‬, 5  T\ ‫أ‬ D
office for immunization and physical examination. )"‫ *" أدت وا‬.‫ي‬4‫ ا‬EFT‫ وا‬J*CQ‫ أ ا‬
The mother is concerned. Which of the following is  ‫ أي‬.vJC5 "a*2‫ ا‬J|w IF " ‫> ه‬CT\ ‫ن‬w#
a contraindication to vaccinating the child ? :‫؟‬T2‫ا ا‬b‫ ه‬J*CQ ‫ب‬.2Q,‫ ه د ا‬6Q‫ا‬
A. Acute otitis media with a temperature of ‫س‬°38 ‫ ارة‬- ‫ د‬D2,‫>ب أذن و‬Q‫ ا‬.A
38°C requiring antibiotic therapy .‫ي‬J ‫د‬# ‫ج‬v gC2Q ‫ي‬b‫وا‬
B. Previous vaccination reaction that consisted ‫  ارة‬nwQ @#, 6* S ‫ رد‬.B
of fever and fussiness that lasted for 2 days .J ‫ة‬QT ‫ج "وم‬%‫وا‬
C. History of an allergic reaction to penicillin .JCCJ4&.‫ ا‬DC a4FQ‫ ا‬.C
D. Previous vaccination reaction that consisted %‫  أز‬nwQ @#, 6* S ‫ رد‬.D
of wheezing and hypotension .:;< ‫ض‬TP‫وا‬

Un enfant de 5 ans se présente chez le pédiatre


pour vaccination et examen physique. La mère est
préoccupée, laquelle des propositions suivantes est
une contre-indication à la vaccination de l'enfant?
A- Otite moyenne aiguë avec une température
de 38 ° C nécessitant une antibiothérapie
B- Réaction survenue à une vaccination
antérieure et qui comprenait de la fièvre et
une irritation qui ont duré 2 jours
C- Antécédents d'une réaction allergique à la
pénicilline
D- Réaction survenue à une vaccination
antérieure et qui comprenait une respiration
sifflante et une hypotension
10. What is the main etiology of acute pericarditis? D
A- Idiopathic :‫د؟‬F‫ر ا‬Q‫>ب ا‬QK OJ4JA‫ت ا‬..4‫ ا‬6‫ ه‬
B- Tubercular .g.4‫>ل ا‬l .A
C- Neoplastic .6C, .B
D- Viral .6‫ ور‬.C
E- Bacterial .6,‫و‬J .D
.6| .E
Quelle est l’étiologie principale d’une péricardite
aigue ?
A- Idiopathique
B- Tuberculeuse
C- Néoplasique
D- Virale
E- Bactérienne

6A
11. Which of these is the most common etiology of A
chronic pericarditis? ‫>ب‬QK Jj ]‫آ‬N‫ ا‬g.4‫ ه ا‬6Q‫أي  ا‬
A. Neoplastic :‫؟‬%‫ر ا‬Q‫ا‬
B. Viral .6‫ ور‬.A
C. Autoimmune .6,‫و‬J .B
D. Uremic .6‫ ذا‬6& .C
E. Post-myocardial infarction .6‫ ر‬.D
.OJ.C5 OC ‫ء‬Q‫" ا‬S# .E
Laquelle des propositions suivantes est l’étiologie
la plus fréquente d’une pericardite chronique ?
A. Néoplasique
B. Virale
C. Auto-immune
D. Urémique
E. Post-infarctus du myocarde
12. You are taking charge of a 48-year-old patient with B
no cardiovascular risk factor for whom the lipid OJA‫ و‬OJ.C5 ‫ دون ا‬، 48  /
profile showed a LDLc of 2.3g / L. What are you ‫ ه‬.‫ل‬/‫غ‬2.3 =LDL ‫) أن‬CJF ‫>ت‬G‫ أ‬،‫>"دة‬
doing ? :‫؟‬S‫ا‬
A. Simple monitoring .O.5‫ اا‬.A
B. Health and diet rules .OJFi – OJA‫ا‬bL OJ .B
C. Prescription of ezetimibe .gJJQ‫ إز‬ni‫ و‬.C
D. Prescription of a statin .JQ, ni‫ و‬.D
E. Prescription of a fibrate .‫ات‬.J ni‫ و‬.E

Vous prenez en charge un patient de 48 ans, sans


facteur de risque cardiovasculaire et ayant un bilan
lipidique montrant un LDLc à 2.3g/L. Que faites-
vous ?
A. Surveillance simple
B. Règles d’hygiene et d’alimentation
C. Prescription d’ezetimibe
D. Prescription d’une statine
E. Prescription d’un fibrate
13. In case of essential hypertension (negative B
etiological assessment) resistant to a biotherapy (g.4‫>ل ا‬l) 6,,N‫ ا‬:;‫ع ا‬T‫ ار‬O 6
composed of amlodipine and ramipril, what is the J.‫د‬C‫  أ‬OT†‫ ا‬OJF‫ ا‬OlSC ‫ا*وم‬
pharmacological class to favor? :‫؟‬OCT‫ ا‬OJA‫ ا"وا‬OWT‫ ا‬6‫  ه‬،.J‫ورا‬
A. Anti-aldosterone .‫ون‬Q,"‫ د أ‬.A
B. Diuretics ‫ "رات‬.B
C. Beta blockers .QJ# ‫ت‬. .C
D. Alpha .T‫ت أ‬. .D
E. ARBs (angiotensine receptor blockers) .J4Jl‫ أ‬OC.*Q4 O‫ &ه‬.E

En cas d’hypertension essentielle (bilan étiologique


négatif) résistante à une biothérapie composée
d’amlodipine et de ramipril, quelle est la classe
pharmacologique à privilégier ?
A- Anti-aldostérone
B- Diurétiques
C- Bêta-bloquants
D- Alpha-bloquants
E- ARA (antagonistes des récepteurs
d’angiotensine)
7A
14. In acute coronary syndrome without ST elevation , C
what is the best biological parameter for prognosis ‫دة  دون‬F‫( ا‬6Q‫ )ا‬6JCJJC‫ إآ‬O‫ز‬vQ O 6
and diagnosis? ‫†ي‬.& ‫ي‬J ‫ر‬JS ‫  ه أ‬،ST ‫ع‬T‫إر‬
A. NT-proBNP :‫؟‬6[JP‫و‬
B. CPK .NT-proBNP .A
C. Troponin .CPK .B
D. Myoglobin .J#‫ و‬.C
E. D-Dimer .J#CLJ .D
.J‫د‬-‫ د‬.E
Dans unsyndrome coronarien aigusans surélévation
de ST, quel est le meilleur paramètre biologique à
visée pronostique et diagnostique ?
A. NT-proBNP
B. CPK
C. Troponine
D. Myoglobine
E. D-Dimères
15. Which of these is the most effective treatment used 3N‫ ا‬nJTPQ OJS ]‫آ‬N‫ج ا‬vS‫ ه ا‬6Q‫أي  ا‬ A
for pain relief in acute coronary syndromewith ST ‫ع‬T‫ إر‬-‫دة‬F‫( ا‬6Q‫ )ا‬6JCJJC‫ إآ‬O‫ز‬vQ 6
surevelvation? :‫؟‬ST
A. Morphine IV (4-8mg) .(sC 8-4) ‫ ور"ي‬J‫ ر‬.A
B. Benzodiazepines IV .‫ ور"ي‬J#‫ودز‬%&# .B
C. Risordan .‫وردان‬%‫ ر‬.C
D. LMWH IV .‫ ور"ي‬/TP& 6W% ‫ر ذات وزن‬.J‫ ه‬.D
E. Aspirin IV .‫ ور"ي‬J.,‫ أ‬.E

Parmi les propositions suivantes, quel est le


traitement le plus efficace utilisé pour soulager la
douleur dans un syndrome coronarien aigu avec
surélévation de ST ?
A- Morphine IV (4-8mg)
B- Benzodiazepines IV
C- Risordan
D- Héparine de faible poids moléculaire IV
E- Aspirine IV
16. What imaging test is needed quickly in the B
aftermath of a heart attack? ‫  اوري‬.QS OJQ‫رات ا‬.QXK‫أي  ا‬
A. Myocardial MRI :‫؟‬OJ.C5 OC ‫ء‬Q# O#i{‫" ا‬S# O4# ‫ااؤ‬
B. Cardiac Ultrasound .gC*‫ ا‬OCS 64J\&; J‫ ر‬.A
C. Transoesophagien ultrasound gC*C OJi ‫رة‬i .B
D. CT thoracic .‫ ايء‬. gC*‫"ى ا‬i :J2P .C
E. Myocardial scintigraphy .‫["ر‬C 6S2* [ .D
.gC*‫ ا‬OCS 6‫ [ و‬.E
Quel examen d’imagerie est rapidement nécessaire
suite à un infarctus du myocarde ?
A- IRM myocardique
B- Echocardiaque
C- Echotransoesophagienne
D- TDM thoracique
E- Scintigraphie myocardique

8A
17. Which of these is the main cause of orthostatic C
hypotension in case of normal autonomic nervous Q‫ض ا‬TPK 64JA‫ ا‬g.4‫ ه ا‬6Q‫أي  ا‬
system? 6.[S‫>ز ا‬lC 6SJ.2‫ ا‬-<‫ ا‬O 6 6#[Q{‫ا‬
A. Severe hypovolemia :‫؟‬6‫ا‬b‫ا‬
B. Severe venous insufficiency .""j ‫ دم‬3l E* .A
C. Iatrogenic drug .""j ‫[ر ور"ي‬5 .B
D. Endocrine .6vS‫ب ا‬.2Q,K‫ ا‬.C
E. Shy-dragger syndrome .3[‫ ا;"د ا‬6 ‫ل‬vQ‫ ا‬.D
.‫ درا‬-6j O‫ز‬vQ .E
Parmi les propositions suivantes quelle est la
principale cause d’hypotension orthostatique au cas
où le système nerveux autonome est normal ?
A. Hypovolémie severe
B. Insuffisance veineuse sévère
C. Iatrogènie médicamenteuse
D. Endocrinopathies
E. Syndrome de Shy-dragger

9A
18. A 59-year-old man undergoes coronary bypass 62‫ أ‬.OJCJC‫ إآ‬OCFQ -X ، 59  ‫ر‬ C
surgery. He receives vancomycin prophylactically -,Q‫م ا‬J‫ ا‬6 .O, 24 ‫ "ة‬JA5‫ و‬J4 )
for 24 hours. On the ninth postoperative day, he - ‫م‬°39.8 D‫[ إ‬Q )‫ ار‬IST‫ ار‬OJCS‫" ا‬S#
develops a fever of 39.8°C with a heart rate of 115 3C 65/105 ‫ دم‬:;<‫ د و‬/O. 115 gC5 O,
beats/min and a blood pressure of 105/65mmHg. ‫" دون اار‬J # 3WQC 6‫ا‬l‫ ا‬-5‫ ا‬.@.A‫ز‬
The surgical site is healing well with no redness or ³3/14000 ‫ء‬J.‫"اد آت ا"م ا‬S .uJl ‫أو‬
discharge. His white blood cell count is J. .‫ء‬J# ‫ل "ة آت دم‬.‫ ا‬JCF 6 >?‫و‬
14,000/mm³ and urinalysis reveals many white ‫ ا;ام ذات‬OJ.C, ‫ت‬J[  ‫ل‬.‫ زرع ا"م وا‬
blood cells per high-power field. Blood and urine  ‫ أي‬.‫ز‬Q‫آ‬v ‫ة‬P JL ‫"از‬J4‫ة أوآ‬JX
cultures grow a non-lactose-fermenting oxidase- b‫ج ه‬vS N‫ ه ا‬OJQ‫ ا‬OJF‫ادات ا‬
positive gram-negative rod. Which of the following :‫؟‬O#i{‫ا‬
antibiotics is most appropriate to treat this .J,4‫آ‬CTJ4‫ آ‬.A
infection? .‫ن‬4‫آ‬QT, .B
A. Moxifloxacin .3&J.J‫ إ‬.C
B. Ceftriaxone .‫زول‬4‫آ‬QTC,-3#Q .D
C. Imipenem .JCJ4;J .E
D. Trimethoprim-sulfamethoxazole
E. Trigecycline

Un homme de 59 ans subit un pontage coronarien.


Il reçoit de la vancomycine à titre prophylactique
pendant 24 heures. Au neuvième jour
postopératoire, il développe une fièvre de 39,8 ° C
avec une fréquence cardiaque de 115 battements /
min et une pression artérielle de 105 / 65mmHg. Le
site chirurgical guérit bien sans rougeur ni
décharge. Son compte de globules blancs est de
14,000 / mm³ et l'analyse d'urine révèle de
nombreux globules blancs par champ
microscopique. Les cultures de sang et d'urine
révèlent un bacille gram-négatif non-lactose-
fermentant oxydase positive. Lequel des
antibiotiques suivants est le plus approprié pour
traiter cette infection?
A- Moxifloxacine
B- Ceftriaxone
C- Imipénème
D- Triméthoprime-sulfaméthoxazole
E- Tigécycline

10 A
19. A family of four presents to the emergency room 345 D‫ ت إ‬،‫ أاد‬OS#‫  أر‬O OCA A
with sudden-onset abdominal cramps, and ،OJ&2# ‫ت‬l&Q ‰T ‫"ء‬#   6‫ارئ وه‬2‫ا‬
vomiting. None of them has fever or diarrhea. Four .5 .‫>ل‬,‫  ارة أو إ‬6S 3>& "‫أ‬K .‫ء‬J5‫وا‬
hours earlier, they had lunch at a roadside 3S2 6 ‫&ول ا;"اء‬Q ‫ة‬,N‫ ا‬I‫ آ‬،‫ت‬, -#‫أر‬
restaurant. They ate a variety of grilled meats, fried ،O&Q O ‫م‬F I‫ *" &و‬.@2‫ ا‬g DC
rice, and seasoned vegetables. What is the most ]‫آ‬N‫ ا‬OJt‫ ا‬6‫  ه‬.OJ, ‫ر‬X‫ و‬،6C* ‫وأرز‬
likely mechanism of their symptoms ? :‫اض؟‬N‫Š ا‬CQ FJ
A. Ingestion of preformed toxins .*.4 O ‫م‬, ‫ &ول‬.A
B. Bacterial invasion of stomach and small ‫ء‬S{‫"ة وا‬S‫"ار ا‬l 6| ‫و‬%L .B
intestine wall .O*J5"‫ا‬
C. Proliferation of ingested bacteria with @vQ ‫ج‬Q‫ ا‬- OSCQ.‫ ا‬3J|‫ا‬l‫ر ا‬Q‫ ا‬.C
subsequent production of toxin active ‫ء‬S{‫ ا‬OJ\P vX l‫ ا‬O2j&‫م ا‬4C
against small bowel mucosal cells .O*J5"‫ا‬
D. Bacterial invasion of large intestine wall .O?JC;‫ء ا‬S{‫"ار ا‬l 6| ‫و‬%L .D
E. Secondary malabsorption caused by the  6 ‫) زدة‬.., ‫[ص |ي‬Q‫ء ا‬, .E
pathogen’s overgrowth in the proximal O*J5"‫ء ا‬S{‫ ا‬6 O<‫ا ا‬S‫ا‬
small intestine .O.*‫ا‬

Une famille de quatre personnes se présente à la


salle d'urgence pour une apparition soudaine de
crampes abdominales accompagnées de
vomissement. Aucun d'entre eux n'a de la fièvre ou
une diarrhée. Quatre heures plus tôt, ils ont déjeuné
dans un restaurant en bordure de route. Ils ont
mangé une variété de viandes grillées, de riz frit et
de légumes assaisonnés. Quel est le mécanisme le
plus probable causant leurs symptômes?
A- Ingestion de toxines préformées
B- Invasion bactérienne de la paroi de
l'estomac et de l'intestin grêle
C- Prolifération des bactéries ingérées avec
production subséquente de toxine active
contre les cellules de la muqueuse de
l'intestin grêle
D- Invasion bactérienne de la paroi du gros
intestin
E- Malabsorption secondaire causée par la
prolifération de l'agent pathogène dans
l'intestin grêle proximal

11 A
20. An 18-year-old woman presents with a 2-day   6‫ ت وه‬، 18 ‫اأة ه‬ C
history of sore throat. Which of the following Ol  ‫ أي‬.J b& @CF‫ ا‬6 ‫>ب‬Q‫ا‬
constellation of symptoms and signs is most - 54‫آ] ا‬N‫ ا‬6‫ ه‬OJQ‫رات ا‬j{‫اض وا‬N‫ا‬
consistent with group-A streptococcal pharyngitis ? :‫ أ؟‬Ol‫*"ت  ا‬S# ‫م‬SC.‫>ب ا‬Q‫ا‬
A. Fever, anorexia, dysphagia and hoarseness .OF#‫ و‬،-C# 4 ،OJ>j ‫ *"ان‬،‫ ارة‬.A
B. Fever, runny nose, cough, myalgia and poor ،6C 3‫ أ‬،‫ل‬S, ،6T‫ن أ‬vJ, ،‫ ارة‬.B
appetite .OTJS< OJ>j‫و‬
C. Fever, no cough, tonsillar exudates, and ،O‫ إازات ز‬،‫ل‬S, ‫"ون‬# ،‫ ارة‬.C
tender anterior cervical lymphadenopathy O.5‫ ا‬6 OJN‫ ا‬O‫و‬TC‫ل ا;"د ا‬vQ‫وا‬
D. Fever, cough, pharyngeal erythema and .>P‫و‬
dysphagia .-C# 4‫م و‬SC.‫ اار ا‬،‫ل‬S, ،‫ ارة‬.D
E. Fever, trismus, dysphagia and neck swelling .O.5‫ ورم ر‬-C# 4 ،‫ز‬%< ،‫ ارة‬.E

Une femme de 18 ans se présente avec un mal de


gorge datant de 2 jours. Laquelle des propositions
suivantes de symptômes et de signes est la plus
compatible avec la pharyngite streptococcique du
groupe A?
A- Fièvre, anorexie, dysphagie et enrouement
B- Fièvre, écoulement nasal, toux, myalgie et
manque d'appétit
C- Fièvre, pas de toux, exudats amygdaliens et
lymphadénopathie cervicale antérieure
sensible
D- Fièvre, toux, érythème pharyngé et
dysphagie
E- Fièvre, trismus, dysphagie et gonflement
du cou

12 A
21. A 49-year-old man has a history of athlete’s foot ‫"اء ا*"م‬# O#i{# ‫ة‬J, )" ، 49  ‫ر‬ C
but is otherwise healthy when he develops sudden  6S ‫"أ‬# ‫ة‬wl .‫"ة‬J OF[# -QQ )&‫ و‬6<‫ا‬
onset of fever and pain in the right foot and leg. On >G‫أ‬.D&J‫ق وا*"م ا‬4‫ ا‬6 3‫ع ارة وأ‬T‫ار‬
physical examination, the foot and leg are fiery red ‫ق ذات ن أ‬4‫ي أن ا*"م وا‬4‫ ا‬EFT‫ا‬
with a well-defined indurated margin that appears 35TQ ‫ي‬b‫ وا< "ا وا‬gC[ ‫ ه‬- ‫ري‬
to be rapidly advancing. There is tender inguinal  .OJ#‫ ا{ر‬OJTC‫*" ا‬S‫ ا‬6 3P ‫ه&ك‬.-, #
lymphadenopathy. Which organism is the most :‫؟‬O#i{‫ ا‬b> FJ ]‫آ‬N‫ ا‬g.4‫ ا‬S‫ه ا‬
likely cause of this infection ? .O‫و‬.‫ ا‬O‫&*د‬S‫ ارات ا‬.A
A. Staphylococcus epidermidis .‫ ا*"م‬OTS, .B
B. Tineapedis .OFJ*‫ ا‬O"*S‫ ارات ا‬.C
C. Streptococcus pyogenes .O2CQP ‫ت‬JA‫ها‬K .D
D. Mixed anaerobic infection .T‫أ‬-OJvF‫ ا‬O"* ‫ رات‬.E
E. Alpha-hemolytic streptococci

Ayant des antécédents de pied d'athlète, un homme


de 49 ans, sain, développe soudainement de la
fièvre et des douleurs au pied droit et à la jambe. À
l'examen physique, le pied et la jambe sont rouge
feu avec une marge indurée bien définie qui semble
avancer rapidement. Il y a une lymphadénopathie
inguinale sensible. Quel organisme est la cause la
plus probable de cette infection?
A- Staphylococcus epidermidis
B- Tineapedis
C- Streptococcus pyogenes
D- Infection anaérobie mixte
E- Streptocoques alpha-hémolytiques
22. An 18-year-old male patient has been seen in the g.4# ‫دة‬JS‫ ا‬D‫  إ‬، 18  ‫ب‬j C
clinic for urethral discharge. He is treated with IM @\  ‫ن‬4‫آ‬QT4# u "* .OJCJC‫إازات إ‬
ceftriaxone, but the discharge has not resolved and ‫ أن‬- ‫ ا{ازات‬OC 6>Q& 3 ‫ و‬،S‫ا‬
the culture has returned as no growth. Which of the ‫ ه‬6Q‫ أي  ا‬.6|  DC ‫ "ل‬3 ‫رع‬%‫ا‬
following is the most likely agent to cause this :‫؟‬FJ ]‫آ‬N‫ ا‬g.4‫ ا‬S‫ا‬
infection ? .‫ن‬4‫آ‬QT4 O‫ *و‬OJ&# ‫ رات‬.A
A. Cetriaxone-resistant gonococci .OJA;..‫"|ة ا‬Q‫ ا‬.B
B. Chalmydiapisttaci .O]F‫"|ة ا‬Q‫ ا‬.C
C. Chalamydia trachomatis .:J4.‫ ا‬F‫وس ا‬J .D
D. Herpes simplex virus .OA‫"|ة ا‬Q‫ ا‬.E
E. Chlamydia pneumonia

Un patient de 18 ans s’est présenté à la clinique


pour un écoulement urétral. Il est traité avec de la
Ceftriaxone IM, mais la sécrétion n'a pas été
résolue et le résultat de la culture est négatif.
Lequel des éléments suivants est l'agent le plus
susceptible d’avoir causé cette infection?
A- Gonocoques résistants à la Ceftriaxone
B- Chalmydiapisttaci
C- Chalamydia trachomatis
D- Virus de l'herpès simplex
E- Chlamydia pneumonia

13 A
23. A 27-year-old man has fever, macular rash, and ‫"ي‬C T\ ،‫  ارة‬6S ، 27  ‫ر‬ B
lymphadenopathy. He had unprotected sex with a JL O5v ‫ *" رس‬.OJTC‫*" ا‬S‫ ا‬3P‫ و‬،6S*#
male partner 2 weeks before the onset of these b‫"ء ه‬# .5 J.,‫ أ‬b& ‫Š ذآ‬j - Ow
symptoms and has just learned that the partner is ‫وس‬JT# ‫) [ب‬j ‫ن‬w# QC 3C "*‫ و‬،‫اض‬N‫ا‬
infected with human immunodeficiency virus EF /‫ *" أى ا‬.O.4Q‫ ا‬O&‫ ا‬E*
(HIV). The patient’s rapid HIV test is negative. ‫ر‬.QX‫  ه أ إ‬.OJ.C, OlJQ&‫ ا‬I‫ وآ‬-, ‫أ"ز‬
What is the best test to evaluate this patient for HIV "& O.4Q‫ ا‬O&‫ ا‬E* ‫وس‬JT# O#i{‫ ا‬3JJ*Q
infection ? :‫؟‬/‫ا ا‬b‫ه‬
A. HIV Enzyme Linked Immunosorbent Assay 3%w# :.‫ ا‬6&‫ ا‬%Q‫ة ا‬S .A
(ELISA) .(‫ا‬%J‫ا{"ز )إ‬
B. PCR for HIV RNA . HIV RNA‫از ل‬JJ.‫ ا‬OC4C, T .B
C. Western blot testing .‫ت‬C# ‫ن‬Q,‫ر و‬.QX‫ ا‬.C
D. Glycoprotein 120 Elisa testing .‫ي‬, J‫و‬# 120 ‫ا‬%J‫ر ا‬.QX‫ ا‬.D
E. PCR for HIV DNA .HIV DNA ‫از ل‬JJ.‫ ا‬OC4C, T .E

Un homme de 27 ans a de la fièvre, des éruptions


maculaires et une lymphadénopathie. Il a eu des
relations sexuelles non protégées avec un partenaire
masculin deux semaines avant l'apparition de ces
symptômes et vient d'apprendre que le partenaire
est infecté par le virus de l'immunodéficience
humaine (VIH). Le test rapide du VIH du patient
est négatif. Quel est le meilleur test pour évaluer ce
patient pour l'infection par le VIH?
A- Épreuve immuno-enzymatique (ELISA)
B- PCR pour l'ARN du VIH
C- Test de Western blot
D- Glycoprotéine 120 tests Elisa
E- PCR pour l'ADN du VIH

14 A
24. An 18-year-old high school student presents to the 345 D‫  إ‬، 18  O,‫ "ر‬g\ A
emergency room with a 1-day history of right knee O.‫ اآ‬6 3‫ م وا"  أ‬b&  ‫ارئ وه‬2‫ا‬
pain, swelling, and redness. He is quarterback on ‫ @ آة‬6 ‫ دع‬gK ‫ ه‬.‫ رم واار‬،D&J‫ا‬
the school’s football team. He remembers falling ‫) أ|&ء‬Q.‫ رآ‬DC -5‫آ أ) و‬bQ )‫ إ‬.O,‫"م ا"ر‬5
on the knee while practicing 2 days ago. The knee 15 ,‫ وأر‬O.‫آ‬C ‫ل‬%# ‫ *" أي‬.J b& g‫"ر‬Q‫ا‬
is tapped and 15mL of cloudy fluid is sent for cell .‫رع‬%‫ام وا‬L OX2 ،vP‫"اد ا‬SQ A4‫  ا‬
count, gram stain, and culture. The Gram stain OJ#l‫ت  رات إ‬l ‫ام‬L OX2 ‫>ت‬G‫أ‬
shows gram-positive cocci in clusters. Which of the :‫؟‬SC ‫ر‬4 ‫ ه أ‬6Q‫ أي  ا‬.‫ا;ام‬
following is the best course of action ? O‫رة ا‬Q,‫ وا‬J4T# ‫"ء‬.‫ ا‬.A
A. Start vancomycin and consult orthopedic .‫?م‬S‫ا‬
surgery .‫?م‬S‫ ا‬O‫رة ا‬Q,‫ ا‬.B
B. Consult orthopedic surgery .‫رع‬%‫ ا‬uAQ ‫?ر‬Q# "J‫و‬%J&JC# ‫"ء‬.‫ ا‬.C
C. Start linezolid awaiting culture results .‫ن‬4‫آ‬QT4# ‫"ء‬.‫ ا‬.D
D. Start ceftriaxone 64J\&; J‫ ر‬gC\‫ و‬J4vJQ# ‫"ء‬.‫ ا‬.E
E. Start telavancin and order MRI of the knee .O.‫آ‬C

Un lycéen de 18 ans se présente à la salle d'urgence


avec des antécédents de douleur au genou droit,
d'enflure et de rougeur. Il est quart-arrière dans
l'équipe de football de l'école. Il se souvient d'être
tombé sur le genou pendant qu'il s’entraînait il y a
deux jours. Le genou est ponctionné et 15 ml de
liquide trouble sont envoyés pour la numération
cellulaire, la coloration de Gram et la culture. La
coloration de Gram montre des cocci à Gram
positif en agglomération. Lequel des énoncés
suivants est le meilleur plan d'action?
A- Commencer un traitement avec de la
vancomycine et consulter un chirurgien
orthopédique
B- Consulter un chirurgien orthopédique
C- Commencer un traitement avec de la
linezolide en attendant les résultats de la
culture
D- Commencer un traitement avec de la
ceftriaxone
E- Commencer un traitement avec de la
télavancine et procéder avec une IRM du
genou

15 A
25. A 45-year-old woman has pain in her fingers on "& >S#i‫ أ‬6 3‫    أ‬45 ‫اأة ه‬ B
exposure to cold, arthralgias and difficulty ‫م‬S2‫ ا‬-C# 6 O#Si‫ و‬iT 3‫ أ‬،‫د‬.C ‫ض‬SQ‫ا‬
swallowing solid food. She has a few ‫"ون‬# ‫ ا["ر‬DC JC5 ‫ات‬JSj -, >" .gC[‫ا‬
telangiectasias over the chest but no erythema of ‫ ه&ك‬.O24.&‫ح ا‬24‫ ا‬DC ‫ ا) أو‬6 ‫اار‬
the face or extensor surfaces. There is slight ‫  ه‬.‫ع‬bl‫ذرع وا‬N‫ ا‬،"J‫" ا‬C 6 OT.T\ O‫آ‬,
thickening of the skin over the hands, arms and :‫؟‬6[JP ‫أ إاء‬
torso. What is the best diagnostic workup ? .CCP-‫<"اد ادة‬N‫ي وا‬J|‫ ا‬S‫ ا‬.A
A. Rheumatoid factor and anti-CCP antibodies ‫ وأ<"اد‬70-Scl-‫ د‬،‫ د وي‬.B
B. Antinuclear, anti-Scl-70 and anticentromere .‫ي‬%‫ اآ‬3J4*C ‫دة‬
antibodies ‫ز‬QJQ&J4C ‫&ز وأ<"اد دة‬J‫ آ‬J‫ آ‬.C
C. Creatinine kinase (CK) and antisynthetase .(1-Jo-‫)] د‬
antibodies (such as anti-Jo-1) .J&‫ وآ‬BUN .D
D. BUN and creatinine "J‫ ا‬a;# ‫اض وا?اه‬N‫ج ا‬Q&Q,‫ ا‬.E
E. Reproduction of symptoms and findings by .‫رد‬.‫ء ا‬#
immersion of hands in cold water

Une femme de 45 ans a mal aux doigts lorsqu'elle


est exposée au froid. Elle se plaint également
d’arthralgies et de difficulté à avaler de la
nourriture solide. Elle a quelques télangiectasies sur
la poitrine, mais pas d'érythème sur le visage ou les
surfaces extenseurs. Il y a un léger épaississement
de la peau sur les mains, les bras et le torse. Quel
est le meilleur bilan diagnostique?
A- Facteur rhumatoïde et anticorps anti-CCP
B- Anticorps antinucléaires, anti-Scl-70 et
anticentromères
C- Créatine Kinase (CK) et des anticorps
antisynthétase (tels que anti-Jo-1)
D- Azote uréique sanguin et créatinine
E- Reproduction des symptômes et des
résultats par immersion des mains dans l'eau
froide

16 A
26. A 20-year-old man complains of arthritis and eye uJ>‫ و‬iT ‫>ب‬Q‫    ا‬20  ‫ر‬ C
irritation. He has a history of burning on urination.  J. .‫ل‬.Q‫ة  @ &" ا‬J, )" .JS‫ ا‬6
On examination, there is a joint effusion of the D&J‫ ا‬O.‫ اآ‬6 6C[T ‫ب‬.[‫ أن ه&ك ا‬EFT‫ا‬
right knee and a rash of the glans penis. Which of 6Q‫ أي  ا‬.gJ*‫ ا‬OT DC ‫"ي‬C T\‫و‬
the following is correct ? :‫ ؟‬JFi
A. Neisseria gonorrhoeae is likely to be OT  OJ&# ‫ت‬J4 ‫  ا زرع‬.A
cultured from the glans penis .gJ*‫ا‬
B. The patient is likely to be rheumatoid ‫ي‬J|‫ ")  ر‬/‫  ا أن ا‬.B
factor-positive .6#l‫إ‬
C. An infection process of the gastrointestinal g.4 ‫  أن‬6>‫>ز ا‬l‫ ا‬6 O#i‫ إ‬.C
(GI) tract may precipitate this disease .‫ا اض‬b‫ه‬
D. An ANA is very likely to be positive .6#l‫  ا "ا أن ن إ‬ANA .D
E. Creatinine kinase will be elevated .-T ‫ف‬, ‫&ز‬J‫ آ‬J‫ آ‬.E

Un homme de 20 ans se plaint d'arthrite et


d'irritation des yeux. Il a des antécédents de brûlure
à la miction. À l'examen, il y a un épanchement
articulaire du genou droit et une éruption du gland
du pénis. Lequel des énoncés suivants est correct?
A- Neisseria gonorrhoeae est susceptible d'être
cultivée à partir du gland du pénis
B- Le patient est susceptible un facteur
rhumatoïde positif
C- Un processus d'infection du système gastro-
intestinal peut précipiter cette maladie
D- Une ANA est très susceptible d'être positive
E- Creatinine kinase sera élevée

17 A
27. A 35-year-old-right-handed construction worker  ،D&J‫" ا‬J# S  35  ‫&ء‬#  C
presents with complaints of nocturnal numbness .D&J‫ " ا‬Q 3‫ وأ‬6CJ ‫"ر‬X   ‫وه‬
and pain involving the right hand. Symptoms wake ‫ ه&ك‬." %># ‫ول‬% 3| ‫ و‬m*JQ4 )CSl ‫اض‬N‫ا‬
him and are then relieved by shaking his hand.  .OJ#l‫ إ‬J&J Ov .OT‫ اا‬6 ‫ ار‬/S#
There is some atrophy of the thenar eminence. :‫؟‬/‫ا ا‬b> OFJ[ ‫ أ‬6‫ه‬
Tinel sign is positive. What is the best advise for )‫"ا‬S DC OF )Q.5 ‫ أن‬A‫آ" دا‬wQ‫ ا‬.A
this patient ? Š‫ أن ذ‬qJ ،‫&ء‬.‫ ا‬-5 6 S‫أ|&ء ا‬
A. Always ensure he has a firm grip on his .:,Q‫ ا‬g[S‫ ا‬DC :;‫  ا‬nTPJ,
equipment while working on the "S# ‫ء‬4 ‫[) آ‬S DC "JCl‫@ ا‬J.2 .B
construction site as this may relieve .‫د‬QS‫) ا‬CS )A‫أدا‬
pressure on the median nerve .5 OJA"# OlS‫ آ‬vJ 3[S ‫ة‬J. -<‫ و‬.C
B. Apply ice to his wrist each evening after .O‫ا‬l# ‫ ا*ار‬bX‫أ‬
performing his usual work 6A%JT‫ج ا‬vS‫ ا‬D‫ إ‬/‫ ا‬F .D
C. Wear a wrist splint ar night as initial .3N‫ ا‬nJTPQ ‫{اء ر‬
therapy prior to considering surgery
D. Refer the patient to physical therapy for
exercises to alleviate the pain

Un ouvrier de construction de 35 ans, droitier, se


plaint d'engourdissements nocturnes et de douleurs
de la main droite. Les symptômes le réveillent et
sont ensuite soulagés en secouant la main. Il y a
une certaine atrophie de l'éminence thénar. Le signe
de Tinel est positif. Quel est le meilleur conseil
pour ce patient?
A- Assurez-vous toujours qu'il tient fermement
son équipement pendant qu’il travaille sur
le chantier de construction, car cela pourrait
soulager la pression exercée sur le nerf
médian.
B- Appliquer de la glace sur son poignet
chaque soir après avoir effectué son travail
habituel
C- Porter une attelle de poignet pendant la nuit
comme traitement initial avant de
considérer la chirurgie
D- Référer le patient à la thérapie physique car
les exercices pourrait soulager la douleur

18 A
28. A 35-year-old man who has been incarcerated for 3 ،‫&ات‬, 3 ‫ة‬QT l4 ‫ آن‬، 35  ‫ر‬ C
years develops slowly progressive back pain >?‫ ا‬6 3‫ء أ‬:.#‫ و‬Jl‫ر ") "ر‬2Q "5‫و‬
eventually accompanied by fatigue, intermittent ‫رة‬4X ،6CJ ‫ق‬S ،OS2*Q D ،gS - #F[
fevers, night sweats, weight loss, and pain in the 6 nS< D‫ر إ‬2 D&J‫ ا*"م ا‬6 3‫ وأ‬،‫وزن‬
right foot that progresses to weakness of the right ‫ت‬FJT[‫ء وا‬J.‫"اد ات ا‬S .D&J‫ق ا‬4‫ا‬
leg. His WBC count and platelet count are normal 6SJ.\ ،vP‫ ا‬6SJ.\ ‫ و ") * دم‬،6SJ.\
but he has a normochromic, normocytic anemia. ‫  ه‬.OJSJ.\ O".‫ وا‬OC‫ ا‬nAG‫ ا‬.‫غ‬.[‫ا‬
Renal and hepatic functions are normal. What is the :‫؟‬FJ ]‫آ‬N‫ ا‬EJPQ‫ا‬
most likely diagnosis ? .‫"د‬SQ ‫ ?م‬6* ‫ ورم‬.A
A. Multiple myeloma .‫ض دم *ي د‬J#‫ ا‬.B
B. Acute myeloid leukemia (AML) .(‫ت‬# ‫ع )داء‬P&‫ ا‬, .C
C. Spinal tuberculosis (Pott disease) .‫ض‬SQ4‫ع ا‬P&‫>ب ا‬Q‫ ا‬.D
D. Transverse myelitis .O‫*ر‬T‫ ا‬iT‫ل ا‬vQ‫ ا‬.E
E. Spondyloarthropathy

Un homme de 35 ans qui a été incarcéré pendant 3


ans développe lentement des maux de dos
progressivement accompagnés de fatigue, de
fièvres intermittentes, de sueurs nocturnes, de perte
de poids et de douleurs au pied droit qui
progressent jusqu'à la faiblesse de la jambe droite.
Son nombre de globules blancs et sa numération
plaquettaire sont normaux, mais il présente une
anémie normochrome normocytaire. Les fonctions
rénales et hépatiques sont normales. Quel est le
diagnostic le plus probable ?
A- Myélome multiple
B- Leucémie myéloïde aiguë (LMA)
C- La tuberculose vertébrale (maladie de Pott)
D- Myélite transverse
E- Spondylarthropathie

19 A
29. A 64-year-old woman presents with 6 weeks of  6S 6‫ ت وه‬، 64 ‫اأة ه‬ A
fatigue, dyspnea, and night sweats. She lost 5kg. "* .-J#,‫ أ‬6 b& 6CJ ‫ق‬S‫ و‬،aT& @J< ،gS
she has no history of trauma, has never had surgery, ‘‫ "> ر‬aJ .>‫ام  وز‬LCJ‫ آ‬O4X ‫ت‬4X
and takes no medications. Chest x-ray reveals a ‫>ت‬G‫ أ‬.O‫&ول أدو‬Q K‫ و‬،O‫ا‬l -P 3 ،O<
large right-sided pleural effusion. Thoracentesis DC J.‫ آ‬6.& ‫ب‬.[‫["ر ا‬C OJ&J4‫ ا‬OSjN‫ا‬
yields pleural fluid that appears milky. Pleural fluid DC 6.& A, ‫ل ا["ر‬%#  uQ .N‫ ا‬gl‫ا‬
triglyceride level is 500mg/dL. Which of the 6.&l‫ ا‬A4‫ ا‬6 "J4JC; ‫ى‬Q4 .6.JC j
following disorders is most likely in this patient ? 6‫ ه‬OJQ‫ت ا‬#‫ا‬2<K‫ أي  ا‬.,‫د‬/sC500 ‫ه‬
A. Lymphoma :‫؟‬O‫ ا‬b‫ &" ه‬FJ ]‫آ‬N‫ا‬
B. Congestive heart failure .‫وي‬T ‫ ورم‬.A
C. Pulmonary embolism .6*Q‫ ا‬gC5 ‫[ر‬5 .B
D. Pneumonia .‫ي‬A‫ ا[م ر‬.C
E. Systemic lupus erythematous .OA‫ ذات ر‬.D
.O‫ >ز‬OJ O.A‫ ذ‬.E
Une femme de 64 ans se plaint de fatigue, de
dyspnée et de sueurs nocturnes qui datent de 6
semaines. Elle a perdu 5kg. Elle n'a aucun
antécédent de traumatisme, n'a jamais subi
d'intervention chirurgicale et ne prend aucun
médicament. La radiographie thoracique révèle un
large épanchement pleural droit. La thoracentèse
produit un liquide pleural qui semble laiteux. Le
niveau de triglycérides liquides pleuraux est de 500
mg / dL. Lequel des troubles suivants est le plus
probable chez ce patient?
A- Lymphome
B- Insuffisance cardiaque congestive
C- Embolie pulmonaire
D- Pneumonie
E- Lupus érythémateux systémique

20 A
30. A 40-year-old woman has had increasing fatigue @J<‫ و‬gSQ‫ا" &"ه ا‬%Q  40 ‫اأة ه‬ A
and shortness of breath for 6 months. Physical ‫ت‬v ‫ي‬4‫ ا‬EFT‫> ا‬G‫ أ‬.>j‫ أ‬6 b& aT&Q‫ا‬
examination reveals normal vital signs and a resting .O‫ &" اا‬%97 Jl4‫ع أوآ‬.j‫ وإ‬OJSJ.\ OJ
O² saturation of 97%. Her lungs are clear without EF >G‫ أ‬.%‫ة أو أز‬XX ‫"ون‬#‫ و‬OF<‫ وا‬OA‫ا‬
rales or wheezing. Cardiac examination shows a OPT‫ و‬6| gC5 ‫رز [ت‬# ‫ي‬A‫ ن ر‬gC*‫ا‬
prominent pulmonary component of the second ‫"ل‬.Q 6Q‫ى وا‬4J‫ ا‬E*‫ ا‬O 6 O OJ<.*‫ا‬
heart sound (P²) and a soft systolic murmur at the .‫رزة‬# v O O.5‫ ?> أوردة ا‬.aT&Q‫ ا‬-
left sternal border trhat varies with respiration. Her N‫ ا‬J2.‫ ا‬3P OJ&J4‫ ا["ر ا‬OSj‫>ت أ‬G‫أ‬
neck veins show a prominent v wave. Chest x-ray .O%‫ اآ‬OA‫ ا‬J‫ اا‬-,‫و‬
shows right ventricular hypertrophy and b> EJP -< N‫ ا‬OJQ‫ة ا‬2P‫ ا‬6‫ ه‬
enlargement of the central pulmonary arteries. :‫؟‬O‫ا‬
What is the best next step in establishing a .gC*‫"ى ا‬i :J2P .A
diagnosis in this patient ? ‫ر أول‬Q‫"رة ا‬5 ‫س‬J5 - aT&Q‫س ا‬J5 .B
A. Echocardiogram .3FT‫" ا‬J4‫أوآ‬
B. Spirometry with measurement of diffusing .">l‫ر ا‬.QX‫ ا‬.C
capacity of carbon monoxide .1– T‫ أ‬J4# ‫ى د‬Q4 .D
C. Exercise stress test .N‫ ا‬gC*‫ة ا‬245 .E
D. Alpha-1 antitrypsin level
E. Right heart catheterization

Une femme de 40 ans a eu une fatigue croissante et


un essoufflement pendant 6 mois. L'examen
physique révèle des signes vitaux normaux et une
saturation en O² au repos de 97%. Ses poumons
sont clairs sans râles ou respiration sifflante.
L'examen cardiaque montre une composante
pulmonaire notable du deuxième bruit cardiaque
(P²) et un souffle systolique doux à la limite
sternale gauche qui varie avec la respiration. Ses
veines du cou montrent une onde en V
remarquable. La radiographie thoracique montre
une hypertrophie ventriculaire droite et une
hypertrophie des artères pulmonaires centrales.
Quelle est la meilleure étape suivante pour établir
un diagnostic chez ce patient?
A- Échocardiogramme
B- Spirométrie avec mesure de la capacité de
diffusion du monoxyde de carbone
C- Exercice de stress test
D- Niveau d'alpha-1 antitrypsine
E- Cathétérisme cardiaque droit

21 A
31. A 65-year-old man with mild congrestive heart 6*Q‫ ا‬gC5 ‫[ر‬5  6S  65  ‫ر‬ C
failure is scheduled to receive total hip K .‫رك‬C ‫"ال آ‬.Q,‫ ا‬DC ‫[ل‬F‫? ا‬Q&‫ و‬nJTX
replacement. He has no other underlying disease ‫ع‬T‫ر‬# ‘‫ ") ر‬aJ‫ى و‬X‫  أاض أ‬6S
and no history of hypertension, recent surgery, or  ‫ أي‬.6% ‫اب‬2<‫ أو ا‬،O]" O‫ ا‬،:;<
bleeding disorder. Which of the following is the ‫[م‬K‫  ا‬O5C N‫ ه ا{اء ا‬6Q‫ا‬
best approach to prevention of pulmonary embolus :‫؟‬/‫ا ا‬b‫ي &" ه‬A‫ا‬
in this patient ? .‫م‬/sC75 J.,‫ أ‬.A
A. Aspirin 75mg/d .‫م‬/sC25 J.,‫ أ‬.B
B. Aspirin 25mg/d J "Cl‫ ا‬IF sC30 ‫ر‬#4‫ إآ‬.C
C. Enoxaparin 30mg subcutaneously bid .J
D. Early ambulation .. ‫ف‬S,‫ إ‬.D
E. Graded compression elastic stocking .O O2L< ‫ ارب‬.E

Un homme de 65 ans présentant une insuffisance


cardiaque légère est prévu pour une prothèse totale
de la hanche. Il n'a aucune autre maladie sous-
jacente et aucun antécédent d'hypertension, de
chirurgie récente ou de trouble de la coagulation.
Laquelle des propositions suivantes est la meilleure
approche pour la prévention de l'embolie
pulmonaire chez ce patient?
A- Aspirin 75 mg / jour
B- Aspirine 25 mg / jour
C- Enoxaparine 30mg sous-cutanée deux fois
par jour
D- Ambulation précoce
E- Bas élastique de compression gradué
32. An obese 50-year-old woman complains of ‫ م‬،‫   أرق‬، 50 ‫ ه‬O&"# ‫اأة‬ A
insomnia, daytime sleeping, and fatigue. During a 6S >‫ أ‬J. ‫ ا&م‬O,‫ &" درا‬.gS‫ و‬،‫أ|&ء ا&>ر‬
sleep study she is found to have recurrent episodes - – O4‫ ا‬6 O# 30 6‫ – ا‬6j ‫ع‬.j‫إ‬
of arterial desaturation – about 30 events per hour – ‫ ه‬6Q‫ أي  ا‬.‫"ادي‬4‫ ا‬aT ‫ع‬2*‫ ا‬DC O‫أد‬
with evidence of obstructive apnea. Which of the :‫؟‬O‫ ا‬b> ‫ر‬QP‫ج ا‬vS‫ا‬
following is the treatment of choice for this patient? OJ4T&Q‫ق ا‬2C Q4 6#l‫ إ‬:;< .A
A. Nasal continuous positive airway pressure .OJTN‫ا‬
B. Uvulopalatopharyngoplasty .‫>ة‬C‫م وا‬SC.‫&Š وا‬F‫ رأب ا‬.B
C. Hypocaloric diet .‫ات‬F‫ ا‬JC5 OJ ‫ ?م‬.C
D. Tracheostomy .DL‫ ; ا‬.D
E. Oxygen via nasal cannula .OJT‫ أ‬OJ&5 . Jl4‫ أوآ‬.E

Une femme obèse âgée de 50 ans se plaint


d'insomnie, de sommeil diurne et de fatigue. Au
cours d'une étude du sommeil, des épisodes
récurrents de désaturation artérielle ont été
constatés - environ 30 événements par heure - avec
des signes d'apnée obstructive. Lequel des éléments
suivants est le traitement de choix pour ce patient?
A- Pression nasale positive continue des voies
respiratoires
B- Uvulopalatopharyngoplastie
C- Diète hypocalorique
D- Trachéotomie
E- Oxygène par canule nasale

22 A
33. A 57-year-old man presents with hemoptysis and ‫ ا"م‬qT   ‫  وه‬، 57  ‫ر‬ C
generalized weakness. His symptom begans with J;i ‫ ا"م‬qT&# ‫اض‬N‫"أت ا‬# .3S nS<‫و‬
small-volume hemoptysis 4 weeks ago. Over the ،JJ<‫ ا‬J.,N‫ "ى ا‬DC .-J#,‫ أ‬4 b& 3lF‫ا‬
past 2 weeks, he has become weak and feels “out of  ‫ج‬X")S<‫ن و‬w# S‫ و‬TJS< .i‫أ‬
it”. His appetite has dismished, and he has lost 4.5 s‫ آ‬4.5 "*‫ و‬،‫م‬S2‫ ا‬DC )QJ>j I[5& ."‫ة‬2J4‫ا‬
kg of weight. He has a 45-pack-year history of .&, O.C 45 6‫ ه‬JX"Q‫ ا‬6 )J, .)‫ وز‬
cigarette smoking. Physical examination is ‫ص‬FT‫>ت ا‬G‫ أ‬.WJj mFC 3 ‫ي‬4‫ ا‬EFT‫ا‬
unremarkable. Laboratory studies reveal a mild 118 [‫دم ا‬i OJ5‫ و‬nJTX ‫ * دم‬O.P‫ا‬
anemia and a serum sodium value of 118Eq/L. 6 OCQ‫ آ‬OJ&J4‫ ا["ر ا‬OSj‫>ت أ‬G‫ أ‬.‫ ل‬/‰ 6CC
Chest x-ray shows a 5cm left mid-lung field mass ،n[& ‫ع‬4‫ ا‬- 3, 5 ‫ى‬4J‫ ا‬OA‫ ا‬n[Q& *
with widening of the mediastinum suggesting J‫ ا‬.OJT[&‫ ا‬OJTC‫*" ا‬S‫ ا‬3P D‫ إ‬J 
mediastinal lymphadenopathy. MR scan of the ]‫آ‬N‫ ا‬g.4‫  ه ا‬.WJj mFC 3 ‫"غ‬C 64J\&;‫ا‬
brain is unremarkable. What is the most likely :‫اض؟‬N‫ ا‬b> FJ
cause of his symptoms ? .OJA‫ ا>ا‬gS‫\ن ا‬, .A
A- Bronchial carcinoid .OA‫ ا‬6 O"L O\, .B
B- Adenocarcinoma of the lung .OA‫ة ا‬J;[‫ ا‬OJCP‫ ا‬O\, .C
C- Small cell carcinoma of the lung .‫ي‬A‫اج ر‬X .D
D- Lung abcess .‫ي‬A‫ت ا‬Jjj‫ ورم ا‬.E
E- Pulmonary aspergilloma

Un homme de 57 ans présente une hémoptysie et


une faiblesse généralisée. Ses symptômes ont
commencé par une hémoptysie de petit volume il y
a 4 semaines. Au cours des deux dernières
semaines, il est devenu faible et se sent affaibli.
Son appétit a chuté et il a perdu 4.5 kg de son
poids. Il a fumé la cigarette pendant 45 ans. Son
examen physique est sans particularité. Les études
de laboratoire révèlent une anémie légère et une
valeur de sodium sérique de 118Eq / L. La
radiographie thoracique montre une masse de
champ pulmonaire médio-pulmonaire gauche de 5
cm avec un élargissement du médiastin suggérant
une lymphadénopathie médiastinale. La
scintigraphie IRM du cerveau est sans particularité.
Quelle est la cause la plus probable de ses
symptômes?
A- Carcinoïde bronchique
B- Adénocarcinome du poumon
C- Carcinome à petites cellules du poumon
D- Abcès du poumon
E- Aspergillome pulmonaire

23 A
34. A 60-year-old obese man complains of excessive ‫س >ري‬S   ، 60  "# ‫ر‬ E
daytime sleepiness. He has been in good health :;< ‫ع‬T‫]&ء ار‬Q,# ‫"ة‬J OF[# -QQ ‫ ه‬.‫ط‬T
except for mild hypertension. He drinks alcohol in O‫ *" أدت زو‬.‫"ال‬Q# ‫ل‬F‫ إ) ب ا‬.nJT\
moderation. The patient’s wife states that he snores EF .‫ "ة ات‬m*JQ4‫ و‬vJ P )‫ أ‬/‫ا‬
at night and awakens frequently. Examination of :‫؟‬g4N‫ ا‬6‫ ه‬OJQ‫ت ا‬,‫ أي  ا"را‬.6SJ.\ ‫م‬SC.‫ا‬
the oropharynx is normal. Which of the following .‫ أط ا&م‬3JJ*Q 6A#>‫ دغ آ‬:J2P .A
studies is most appropriate ? .aT&Q‫@ ا‬J<  ‫ي‬FQC O>Q‫ ا‬: .B
A. Electroencephalogram (EEG) to assess .6‫ ا‬O2 ‫ع‬.j‫ إ‬.C
sleep patterns .‫ أ|&ء ا&م‬aT&Q‫ت ا‬v O,‫ درا‬.D
B. Ventilation pattern to detect apnea .‫ ا&م‬O,‫ درا‬.E
C. Arterial O² saturation
D. Study of muscles of respiration during sleep
E. Polysomnography

Un homme obèse âgé de 60 ans se plaint d'une


somnolence diurne excessive. Il est en bonne santé
à l’exception d’une hypertension légère. Il boit de
l'alcool avec modération. La femme du patient
déclare qu'il ronfle la nuit et se réveille
fréquemment. L'examen de l'oropharynx est
normal. Laquelle des études suivantes est la plus
appropriée?
A. Electroencéphalogramme (EEG) pour
analyser les habitudes de sommeil
B. Modèle de ventilation pour détecter l'apnée
C. Saturation artérielle en O²
D. Étude des muscles de la respiration pendant
le sommeil
E. Polysomnographie

24 A
35. A 72-year-old man comes to the office with 6S ‫دة وه‬JS‫ ا‬D‫   إ‬72  ‫ر‬ A
intermittent symptoms of dyspnea on exertion, ،">l‫ &" ا‬aT&Q‫@ ا‬J OS2*Q ‫ أاض‬
palpitations, and cough occasionally productive of ،6.C*‫ ا‬-4Q‫ &" ا‬.J‫"م أ‬C uQ& ‫ل‬S,‫ و‬،‫*ن‬TX
blood. On cardiac auscultation, a low-pitched DC OQX OJ\4.‫ هدرة ا‬OPT /JTX # IS,
diastolic rumbling murmur is faintly heard at the :‫؟‬OPT&‫ ا‬b> FJ ]‫آ‬N‫ ا‬g.4‫  ه ا‬.O*‫ا‬
apex. What is the most likely cause of the murmur? .‫ب‬.‫ ا‬OC 6 OJ|‫ ر‬D .A
A. Rheumatic fever as a youth ."JS# "‫ أ‬b& :;< ‫ع‬T‫ ار‬.B
B. Long-standing hypertension O&4‫ل ا‬vX OQi OJ.C5 OC ‫ء‬Q‫ ا‬.C
C. A silent MI within the past year .OJ<‫ا‬
D. A congenital anomaly .6*CX gJ .D
E. Anemia from chronic blood loss .O&% ‫رة دم‬4X  u ‫ * دم‬.E

Un homme de 72 ans se présente au cabinet avec


des symptômes intermittents de dyspnée à l'effort,
des palpitations et une toux produisant parfois du
sang. Lors d'une auscultation cardiaque, un léger
murmur diastolique grondant est entendu à l'apex.
Quelle est la cause la plus probable du souffle
cardiaque?
A. La fièvre rhumatismale dans sa jeunesse
B. L'hypertension de longue durée
C. Un IM silencieux au cours de la dernière
année
D. Une anomalie congénitale
E. Anémie résultante d’une perte de sang
chronique
36. A 21-year-old man presents in the emergency room ‫ارئ وه‬2‫ ا‬345 D‫  إ‬، 21  ‫ر‬ B
with new onset of slurred speech and right 6 6*j ‫ل‬%X‫م و‬v‫ ا‬3]SCQ ‫"ة‬lQ4 O‫"ا‬#  
hemiparesis. On ausculatation the patient has a 6 OJ<.*‫ ا‬OPT )" ‫ آن‬-4Q‫ &" ا‬.D&J‫ ا‬O>l‫ا‬
systolic murmur at the pulmonic region with a O ‫ \ل‬DC OJ\4.‫ ا‬OS*S5 - OA‫ ا‬O*2&‫ا‬
diastolic rumble along the left sternal border. The I#|‫ و‬34*& 6]‫ ا‬gC*‫ت ا‬i .‫ى‬4J‫ ا‬E*‫ا‬
second heart sound is split and fixed relative to ‫اض‬N ‫ ا‬g.4‫ ه ا‬.aT&QC O.4&#
respiration. What is the likely cause of patient’s :‫؟‬/‫ا‬
symptom ? .J2.‫ ا‬% 6 gJ .A
A. Ventricular septal defect .‫ذ‬N‫ ا‬% 6 gJ .B
B. Atrial septal defect (ASD) .O4‫ ا‬OJ‫ ا*&ة ا‬.C
C. Patent ductus arteriosus .>#N‫[ر ا‬5 .D
D. Aortic insufficiency .>#N‫@ ا‬J .E
E. Coarctation of the aorta

Un homme de 21 ans se présente dans la salle


d'urgence avec une nouvelle apparition de troubles
de l'élocution et une hémiparésie droite. A
l'auscultation, le patient présente un souffle
systolique à la région pulmonaire avec un
grondement diastolique le long de la bordure
sternale gauche. Le deuxième son cardiaque est
divisé et fixe par rapport à la respiration. Quelle est
la cause probable du symptôme du patient?
A. Défaut septal ventriculaire
B. Défaut septal auriculaire (TSA)
C. Persistance du canal artériel
D. Insuffisance aortique
E. Coarctation de l'aorte
25 A
37. You are volunteering with a dental colleague in a ‫دة‬J 6 ‫&ن‬,N‫ ا‬g\ J‫ ز‬- ‫ع‬2QQ I‫أ‬ D
community indigent clinic. A nurse has prepared a D<‫ ا‬OFAK O<‫ *" أ"ت ا‬.‫ز‬S‫ ا‬-QlC
list of patients who are scheduled for a dental ‫ن‬QF "5 bC‫ وا‬3> OJ&, OlS ‫ا*ر إاء‬
procedure and may need antibiotic prophylaxis D<‫ ا‬OFAK  .*.4 OJA5‫ و‬OJ ‫[دات‬
beforehand. Of the patients listed below, who  "JTQ4 ‫ أن‬FJ ]‫آ‬N‫  ه ا‬،‫آرة أد‬b‫ا‬
would be most likely to benefite from antibiotic gC5 ‫;ف‬j ‫>ب‬Q‫  ا‬OFC 6A5‫ي ا‬JF‫ا[د ا‬
prophylaxis to prevent infective endocarditis (IE) ? :‫؟‬6|
A. A 17-year-old adolescent boy with @J  6S O&, 17  ‫ب‬j .A
coarctation of the aorta .>#N‫ا‬
B. A 26-year-old woman with a ventricular 6 gJ  6S  26 ‫ اأة ه‬.B
septal defect (VSD) repaired in childhood OC 6 )vi‫ ى إ‬J2.‫ ا‬%
C. A 42-year-old woman with mitral valve .OT2‫ا‬
prolapse 6"  6S  42 ‫ اأة ه‬.C
D. A 65-year-old woman with prosthetic aortic .6‫ا‬Q‫ا[م ا‬
valve >#‫م أ‬i >"  65 ‫ اأة ه‬.D
E. A 72-year-old woman with aortic stenosis .6&2i‫ا‬
(AS) @J  6S  72 ‫ اأة ه‬.E
.>#‫أ‬
Vous faites du bénévolat avec un collègue dentaire
dans une clinique communautaire sans ressources.
Une infirmière a préparé une liste de patients qui
doivent subir une intervention dentaire et qui
pourraient avoir besoin d'une prophylaxie
antibiotique au préalable. Parmi les patients
énumérés ci-dessous, qui serait le plus susceptible
de bénéficier d'une prophylaxie antibiotique pour
prévenir l'endocardite infectieuse ?
A. Un adolescent de 17 ans avec une
coarctation de l'aorte
B. Une femme de 26 ans avec une
malformation du septum ventriculaire
réparée pendant l'enfance
C. Une femme de 42 ans avec un prolapsus de
la valve mitrale
D. Une femme de 65 ans avec une prothèse
valvulaire aortique
E. E. Une femme de 72 ans présentant une
sténose aortique

26 A
38. A 23-year-old man complains of persistent mK "* .3A‫"اع دا‬i    23  ‫ر‬ A
headache. He has noticed gradual increase in his  DC ‫اء‬b ‫س‬J5‫) و‬X ‫س‬J5 6 OJl‫زدة "ر‬
ring size and his shoe size over the years. On ‫ت‬i )" ‫ آن‬،‫ي‬4‫ ا‬EFT‫ &" ا‬.J&4‫ا‬
physical examination, he has a peculiar deep, *F‫ ا‬EF J .3*‫@ أف وŠ أ‬J gL
hollow-sounding voice and a prognathic jaw. 6‫  ه‬.JJL"[‫ ا‬J*‫ ا‬D D‫[ي إ‬.‫ا‬
Bedside visual field testing suggests bitemporal :‫؟‬O#C2‫ ا‬OJ‫و‬N‫ت ا‬,‫ا"را‬
hemianopsia. What initial studies are indicated ? 6C[ J4v )#   .A
A. Serum insulin-like growth factor (IGH-1) .JQ‫آ‬K‫و‬.‫ت ا‬Q4‫و‬
and prolactin levels .6.[‫ت هن ا& ا‬Q4 .B
B. Morning growth hormone levels ‫زون‬Q4‫"آ‬# ‫ح‬.‫ول ا‬%J‫ى ار‬Q4 .C
C. Overnight dexamethasone-suppressed .‫ >را‬vJ
cortisol level ‫ج‬4‫س ا‬J5 3JJ*Q OllC 6. ‫رة‬i .D
D. Lateral skull film to assess sella turcica size .6‫آ‬Q‫ا‬
E. GHRH-stimulated growth hormone level .&‫ر >ن ا‬F‫ى ا>ن ا‬Q4 .E

Un homme de 23 ans se plaint de maux de tête


persistants. Il a remarqué une augmentation
graduelle de la grosseur de son doigt et de sa
pointure au cours des années. À l'examen physique,
il a une voix profonde et creuse et une mâchoire
prognathe. Les tests au champ visuel au chevet
suggèrent une hémianopsie bitemporale. Quelles
études initiales sont indiquées?
A. Facteur de croissance analogue à l'insuline
(IGH-1) et taux de prolactine
B. Niveaux d'hormone de croissance du matin
C. Niveau de cortisol inhibé par la
dexaméthasone pendant la nuit
D. Radio de crâne latérale pour évaluer la taille
de la selle turcique
E. Niveau d'hormone de croissance stimulé par
GHRH

27 A
39. A patient with small cell carcinoma of the lung ،OA‫ ا‬6 ‫ة‬J;[‫ ا‬vP‫ ا‬O\,  6S / C
develops increasing fatigue but is otherwise alert ).&Q ‫Š ه‬b vX  ، "‫ا‬%Q gS )" ‫ر‬2Q
and oriented. Serum electrolytes show serum ‫ارد ا[ ?> أن ا[دم‬j EF .)Q‫و‬
sodium 0f 118mEq/L. there is no evidence of ‫ض‬TP‫ ا‬،O‫ ذ‬J‫ " د‬K .‫ ل‬/‰ 6CC118
edema, orthostatic hypotension, or dehydration. OJ,‫ ذات أ‬%‫ل آ‬.‫ ا‬.‫ف‬Tl ‫ أو‬،6#[Q‫ ا‬
Urine is concentrated with an osmolality of J&‫ ا‬،[‫ ا‬BUN .‫ل‬/‫ ل‬6CC550
550mmol/L. serum BUN, creatinine, and glucose 6‫ ه‬6Q‫ أي  ا‬.6SJ.2‫ل ا‬l‫ ا‬6 ‫آز‬C;‫وا‬
are within normal range. Which of the following is :‫؟‬g4N‫ة ا‬2P‫ا‬
the next appropriate step ? .Q‫دل ا‬S 6FC ‫ل‬CF# 3J*CQ‫ ا‬.A
A. Normal saline infusion .‫ل‬.‫ إدرار ا‬.B
B. Diuresis .A‫ا‬4# "JJ*Q‫ ا‬.C
C. Fluid restriction .JCJ,C‫ د‬.D
D. Demeclocycline .Q‫ ا‬6 6FC ‫ل‬CF# 3J*CQ‫ ا‬.E
E. Hypertronic saline infusion

Un patient atteint d'un carcinome pulmonaire à


petites cellules développe une fatigue croissante
mais reste alerte et orienté. Les électrolytes sériques
montrent un taux sérique de sodium de 118mEq /
L. Il n'y a aucune preuve d'œdème, d'hypotension
orthostatique ou de déshydratation. L'urine est
concentrée avec une osmolalité de 550mmol / L.
l'azote uréique sanguin, la créatinine et le glucose
sont dans la marge normale. Laquelle des étapes
suivantes est la prochaine étape appropriée?
A. Infusion de solution physiologique salée
B. Diurèse
C. Restriction de liquide
D. Deméclocycline
E. Infusion de solution salée hypertronique

28 A
40. A 30-year-old man is evaluated for a thyroid "* .OJ5‫"ات در‬J*S  30  ‫ ر‬3JJ* 3 "* C
nodule. The patient reports that his father died from ‫\ن ا"رق وأن‬, g.4# D "‫ أن وا‬/‫أد ا‬
thyroid cancer and that a brother had a history of %J‫ آ‬.‫رة‬Q OC‫ت آ‬J[  6S ‫*) آن‬J*j
recurrent renal stones. Blood calcitonin  5‫ أ‬6SJ.2‫ )ا‬/‫ام‬LJ#2000 ‫ ا"م‬JQJ4‫آ‬
concentration is 2000pg/mL (normal is <100) ; [‫ ا‬6 ‫ت‬T,T‫م وا‬J4‫ت ا‬Q4 ،(100
serum calcium and phosphate levels are normal. EQP ‫ اح‬gJ.\ "& /‫ل ا‬,‫ إر‬3 .OJSJ.\
The patient is referred to a thyroid surgeon. Which ‫ أ‬6;.& OJQ‫ت ا‬,‫ أي  ا"را‬.OJ5‫ ا;"ة ا"ر‬6
of the following studies should also be obtained ? :‫>؟‬JC ‫[ل‬F‫ا‬
A. Obtain a liver scan .".C 4 DC ‫[ل‬F‫ ا‬.A
B. Measure parathormone level .‫ى هن رات ا"رق‬Q4 ‫س‬J5 .B
C. Measure urinary metanephrines .‫ل‬.‫ ا‬TQ ‫س‬J5 .C
D. Administer suppressive doses of thyroxine ‫س‬J5‫ و‬J4‫وآ‬J]C O2.] ‫ء ت‬2‫ إ‬.D
and measure levels of thyroid-stimulating- .‫"رق‬C ‫ض‬F‫ى ا>ن ا‬Q4
hormone .-‫د ا‬J# /‫ ا‬OlS .E
E. Treat the patient with radioactive iodine

Un homme de 30 ans est évalué pour un nodule


thyroïdien. Le patient affirme que son père est
décédé d'un cancer de la thyroïde et qu'un frère
avait des antécédents de calculs rénaux récurrents.
La concentration de calcitonine dans le sang est de
2000pg / mL (la normale est <100); les taux
sériques de calcium et de phosphate sont normaux.
Le patient est référé à un chirurgien
endocrinologique. Laquelle des études suivantes
devrait également être obtenue?
A. Obtenir une scintigraphie hépatique
B. Mesurer le niveau de parathormone
C. Mesurer les métanéphrines urinaires
D. Administrer des doses suppressives de
thyroxine et mesurer les taux de
thyréostimuline
E. Traiter le patient avec de l'iode radioactif

29 A
41. A 40-year-old man complains of slowly ‫ر‬2Q 3S nS<    40  ‫ر‬ C
progressive generalized weakness, weight loss, 3[S‫ ا‬6 3‫ وأ‬،2.‫ ا‬6 3‫ أ‬،‫رة وزن‬4X ،‫ء‬:.#
abdominal pain, and wrist and knee pain over the 6 ) J5 .OJ<‫ ا‬OCJC*‫> ا‬jN‫ل ا‬vX O.‫واآ‬
past several months. He was told at an urgent care ‫  ا*"ار‬vJC5 -T ‫ ا"م‬, ‫"ل‬S ‫ارئ أن‬2‫ا‬
visit that his blood sugar was a little higher than O>  ‫"ي‬.‫ ض آ‬OJCA ‫ة‬J, ‫ ه&ك‬.6SJ.2‫ا‬
normal. There is a family history of liver disease on ‫  ط‬6S /‫ أن ا‬J. ،EFT‫ &" ا‬."‫وا‬
his father’s side. On examination, the patient has m ‫ آ‬.".‫ ا‬O DC ‫س‬4l‫ و‬Q& s.[
diffuse hyperpigmentation and palpable liver edge. J[S‫ ا‬6 nJT\ ‫"د‬SQ iT ‫>ب‬Q‫أ ا‬
Mild polyarthritis of the wrists and ‫ أو‬EF ‫  ه أ‬.OJv4‫ ا‬OJS&4‫ ا‬iT‫وا‬
metacarpophalangeal joints is also noted. What is ‫ا‬b‫ ه‬OC EJP DC ‫"ة‬4C ‫ص‬FT‫ ا‬Ol
the best test or combination of tests to help you :‫؟‬/‫ا‬
diagnose this patient’s problem ? ‫@ و"ول‬TQ‫ ا‬- ‫"اد دم آ‬S .A
A. Complete blood count with differential and .‫ب ا‬v*Q,K‫ا‬
a comprehensive metabolic panel HbA1C .B
B. Hemoglobin A1C ،""F‫ط ا‬.‫ر‬K OC‫ ا*"رة ا‬،""F‫ ا‬.C
C. Iron, total iron-binding capacity, and ferritin .JT‫وا‬
D. Alpha-1 antitrypsin level .1– T‫ أ‬J4# ‫ى د‬Q4 .D
E. Liver-spleen scan .".‫آ‬-‫ل‬F\ 4 .E

Un homme de 40 ans se plaint d'une faiblesse


généralisée lentement progressive, d'une perte de
poids, de douleurs abdominales et de douleurs au
poignet et au genou au cours des derniers mois. On
lui a dit lors d'une visite de soins d'urgence que sa
glycémie était un peu plus élevée que la normale. Il
y a des antécédents familiaux de maladie du foie du
côté de son père. A l'examen, le patient présente
une hyperpigmentation diffuse et un bord palpable
du foie. Une polyarthrite légère des poignets et des
articulations métacarpophalangiennes est également
notée. Quel est le meilleur test ou la meilleure
combinaison de tests pour vous aider à
diagnostiquer le problème de ce patient?
A. Formule sanguine complète avec
différentiel et un panneau métabolique
complet
B. Hémoglobine A1C
C. Fer, capacité totale de fixation du fer et
ferritine
D. Niveau d'alpha-1 antitrypsine
E. Scan du foie et de la rate

30 A
42. A 32-year-old woman complains of abdominal pain n5Q 2.‫ ا‬6 3‫   ا‬، 32 ‫اأة ه‬ A
off and on since age of 17. She notices abdominal ‫خ‬TQ# S 6‫ ه‬. 17 ‫ آن ه‬b& ‫د‬S 3|
bloating relieved by defecation as well as ‫ك‬4‫>ل وا‬,‫Š ه&ك ا‬b‫;ط آ‬Q# ‫ول‬% 6&2#
alternating diarrhea and constipation. She has no ،6‫ ه‬n% ،‫رة وزن‬4X  6S K .‫&وب‬Q
weight loss, GI bleeding, or nocturnal diarrhea. On ‫م‬v‫ أن "> إ‬EFT‫ &" ا‬J. .6CJ ‫>ل‬,‫أو ا‬
examination, she has slight LLQ tenderness and .2.C 6CT4‫ ا‬4N‫ ا‬-#‫ ا‬6 ‫زي‬L 2# ‫خ‬TQ‫وا‬
gaseous abdominal distension. Laboratory studies, ‫"اد آ‬S Q 6Q‫ وا‬،O.P‫ص ا‬FT‫ا‬
including complete blood count, are normal. Which 6‫و‬N‫ ه ا{اء ا‬6Q‫ أي  ا‬.OJSJ.\ ،‫ت‬C
of the following is the most appropriate initial :‫؟‬g4N‫ا‬
approach? u& ‫ د‬،OJA‫ا‬b;‫ف ا‬JN‫دة ا‬%# OJi .A
A. Recommend increased dietary fiber, .>j "S# EFT‫ ا‬OS#Q‫ و‬،‫وم‬%C‫&" ا‬
antispasmodics as needed, and follow up ‫  أ ااء‬OJ‫ ه‬gJ.\ OS‫ ا‬.B
examination in 2 months .‫ ا*ن‬J?&
B. Refer to gastroenterologist for colonoscopy .J.,‫ة أ‬QT 3T‫  \@ ا‬JC,‫ا‬Q .C
C. Two weeks of oral tetracycline - OC OJ‫ت ه‬,‫ درا‬OC4C, gC\ .D
D. Order UGI series with small bowel follow- .O*J5"‫ء ا‬S{‫ ا‬O,‫ درا‬OS#Q
through .O*J5"‫ء ا‬S{‫  ا‬O%X bX‫ أ‬gC\ .E
E. Order small bowel biopsy

Une femme de 32 ans se plaint de douleurs


abdominales depuis l'âge de 17 ans. Elle remarque
des ballonnements abdominaux soulagés par la
défécation ainsi qu'une alternation entre diarrhée et
constipation. Elle n'a aucune perte de poids,
saignements gastro-intestinaux ou diarrhée
nocturne. À l'examen, elle a une légère sensibilité
au quart inférieur gauche et une distension
abdominale gazeuse. Les études de laboratoire, y
compris une numération formule, sont normales.
Laquelle des propositions suivantes est la plus
appropriée?
A. Recommander une augmentation des fibres
alimentaires, des antispasmodiques au
besoin, et un examen de suivi dans 2 mois
B. Se référer à un gastro-entérologue pour une
coloscopie
C. Deux semaines de tétracycline par voie
orale
D. Série de l'ordre UGI avec suivi de l'intestin
grêle
E. Commander une biopsie de l'intestin grêle

31 A
43. A 55-year-old white woman has had recurrent  6S ، 55 ‫ ه‬،/J#N‫ق ا‬S‫اأة  ا‬ A
episodes of alcohol-induced pancreatitis. Despite .‫ل‬F# ‫ض‬F ‫&س‬# ‫>ب‬QK ‫رة‬Q ‫ت‬#
abstinence, the patient develops postprandial  O‫ ”ن ا‬،‫اف‬,{‫  "م ا‬3L‫ ا‬DC
abdominal pain, bloating, weight loss despite good ‫رة‬4X ،OPT ،‫م‬S2‫" &ول ا‬S# 2.‫ ا‬6 3‫ أ‬
appetite, and bulky, foul-smelling stools. Kidney, ‫ة‬J.‫ آ‬OCQ‫از ذات آ‬# ،‫"ة‬Jl‫> ا‬QJ>j  3L# ‫وزن‬
ureter, bladder (KUB) x-ray shows pancreatic ،gF‫ ا‬،OJCC OJ&J4‫ ا‬OSjN‫>ت ا‬G‫ أ‬.OWJ, OFA‫ورا‬
calcifications. In this patient, you should expect to gl 6Q‫ أي  ا‬.6,&# aC ‫ ود‬،O]‫ا‬
find which of the following ? :‫؟‬O‫ ا‬b‫) &" ه‬S5
A- Diabetes mellitus .‫ي‬4‫ داء ا‬.A
B- Malabsorption of fat-soluble vitamins D and 3,"# OCF&‫&ت ا‬JQJTC ‫[ص‬Q‫ء ا‬, .B
K .‫ ع د و ك‬
C- Positive fecal occult blood test .6#l‫از إ‬.‫ ا‬6 6TP‫ ا"م ا‬EF .C
D- Courvoisier sign .)‫ آراز‬Ov .D
E- Markedly elevated amylase .‫ظ‬FC ‫ز‬vJ‫ع أ‬T‫ ار‬.E

Une femme blanche de 55 ans a eu des épisodes


récurrents de pancréatite induite par l'alcool.
Malgré l'abstinence, la patiente présente des
douleurs abdominales postprandiales, des
ballonnements, une perte de poids malgré une
bonne appétit et des selles volumineuses et
malodorantes. Les rayons X du rein, de l'uretère et
de la vessie (KUB) montrent des calcifications
pancréatiques. Chez ce patient, vous devriez vous
attendre à trouver lequel des éléments suivants?
A- Diabète sucré
B- Malabsorption des vitamines
liposolubles D et K
C- Test de sang occulte fécal positif
D- Panneau de Courvoisier
E- Taux d’amylase fortement élevé
44. Acute hyperkalemia is associated with which of the ‫ات‬JJ;  ‫ أي‬- ‫د‬F‫م ا‬J,# ‫ا@ ط‬Q A
following electrocardiographic changes? :‫؟‬OJQ‫ ا‬6A#>‫ ا‬gC*‫ ا‬:J2P
A. QRS widening .QRS-, .A
B. Prolongation of the ST segment
.ST -2* O\‫ إ‬.B
C. A decrease in the PR interval
D. Prominent U waves .PR ‫ة‬Q 6 E* .C
E. T-wave flattening .OPj U ‫ ت‬.D
.T-O 24 .E
L'hyperkaliémie aiguë est associée à laquelle des
changements suivants électrocardiographiques?
A. Élargissement du QRS
B. Prolongation du segment ST
C. Une diminution de l'intervalle PR
D. Ondes U proéminentes
E. Aplatissement des ondes T

32 A
45. Serum elevation of gamma GT may occur in the ‫ آ‬6 OJC[‫ ا‬GT L 6 ‫"ث زدة‬F ‫ أن‬ B
following situations except one: :‫  "ا‬OJQ‫ت ا‬KF‫ا‬
A - Liver cancer .".‫\ن ا‬, .A
B - Gilbert's disease
.‫ت‬.C ‫ داء‬.B
C - During alcoholism
D - Cancer of the pancreas head .‫ل‬F‫ ا‬6\S ‫ أ|&ء‬.C
E - During a phenobarbital treatment .‫&س‬.‫ ا‬O5 ‫\ن‬, .D
.‫ل‬QJ#‫ر‬#&JT# ‫ج‬vS‫ أ|&ء ا‬.E
Une élévation du taux sérique de la gamma GT
peut s'observer dans les situations suivantes, sauf
une. Laquelle ?
A - Au cours des cancers du foie
B - Au cours de la maladie de Gilbert
C - Au cours de l'alcoolisme
D - Au cours d'un cancer de la tête du
pancréas
E - Au cours d'un traitement au
phénobarbital
46. A 44 y old male complains of erectile dysfunction. OTJG‫ و‬6 CX    44  ‫ر‬ C
He is treated for hypertension and for a psychiatric ‫ و ض‬:;‫ع ا‬T‫  ار‬uS ‫ ه‬.‫[ب‬QK‫ا‬
disease. Which is the least likely medication he is 5‫&و> ه أ‬Q ‫ي‬b‫ ا‬OJQ‫ ا‬O‫دو‬N‫ أي  ا‬.64T
taking that could be responsible for this problem:
:‫؟‬OC‫ ا‬b‫  ه‬K‫†و‬4 ‫ أن ن‬FJ
A. Propranolol
B. Metoclopramide .‫ال‬#‫و‬# .A
C. Furosemide ."J‫ا‬#CQ .B
D. Domperidone ."JJ,‫و‬J .C
E. Neuroleptics .‫"ون‬.‫ دو‬.D
.‫ دات ذهن‬.E
Un homme de 44ans se plaint de problèmes
érectiles. Il est traité pour hypertension et pour une
maladie psychiatrique.Parmi tous ces médicaments,
lequel serait le moins responsable de ce problèmes :
A. Propranolol
B. Metoclopramide
C. Furosemide
D. Domperidone
E. Neuroleptiques

33 A
47. Which artery is MOST likely to be diseased in a 67 ‫ &" ر‬O#i— <S ]‫آ‬N‫ن ه ا‬j ‫أي‬ B
year old, male patient , non-smoker and ‫م ا"م‬Fj ‫اب‬2<‫ ") ا‬،X" K ، 67 
dyslipidemic, who suffers a syncopal episode :‫؟‬OJL‫ د‬O‫ و‬E*& OJ 6L ‫  ب‬6S‫و‬
secondary to cerebral ischemia?
.6.4‫ ان ا‬.A
(A) Carotid
(B) Vertebrobasilar .‫*ري ا*"ي‬T‫ ان ا‬.B
(C) Anterior communicating artery .6N‫ ا‬i‫ ان ا‬.C
(D) Anterior cerebral artery .6N‫ ا‬6L"‫ ان ا‬.D
(E) Posterior communicating artery .6TCP‫ ا‬i‫ ان ا‬.E

Quelle artère est la plus susceptible d'être atteinte


chez un homme de 67 ans, non fumeur et
dyslipidémique, qui souffre d'un épisode syncopal
secondaire à une ischémie cérébrale?
(A) Carotide
(B) Vertebrobasilaire
(C) Artère communicante antérieure
(D) Artère cérébrale antérieure
(E) Artère communicante postérieure
48. All of the following have been associated with the "& ‫ن‬4-TJQ, O‫ز‬vQ ‫ ء‬- @‫ا‬Q 6C  ‫ آ‬A
development of Stevens-Johnson syndrome in a 32 :‫   "ا؟‬32 ‫اأة ه‬
year old female EXCEPT .O‫"ات >ز‬JA‫و‬JQ, .A
(A) systemic steroids
.‫رات‬2T‫ ا‬.B
(B) mycoplasma
(C) herpes simplex virus .:J4.‫ ا‬F‫وس ا‬J .C
(D) anticonvulsants .‫ج‬vQX‫ دات ا‬.D
(E) a seasonal profile .OJ, O .E

Tous les éléments suivants ont été associés au


développement du syndrome de Stevens-Johnson
chez une femme de 32 ans SAUF
A. stéroïdes systémiques
B. mycoplasme
C. virus de l'herpès simplex
D. anticonvulsivants
E. un profil saisonnier

34 A
49. A 25-year-old man complains of pain and "J‫ ا‬6 ‫ ورم‬3‫    أ‬25  ‫ر‬ D
swellingin the hand and forearm, perioral ‫ك‬4— )Q‫و‬F "S# ‫ء‬J5‫ وا‬،3T‫"ر ل ا‬X ،"4‫وا‬
numbness, and vomiting after trying to catch a  ‫ آ‬.@.A‫ ز‬3C 60/90 ‫ ا"م‬:;< .OClCl‫ ا‬DSw#
rattlesnake. Blood pressure is 90/60 mm Hg. All of
:‫  "ا‬g,& ‫ج‬v ‫ ه‬6C
the following are appropriate therapies EXCEPT
A. fluid resuscitation .A‫ا‬4# ‫ش‬S{‫ ا‬.A
B. administration of 10 vials of antivenin .DSN‫ ا‬34 ‫ * د‬10 ‫ء‬2‫ إ‬.B
C. measurement of coagulation factors and .‫ت‬FJT[‫] وا‬PQ‫س ا ا‬J5 .C
platelets .‫راع‬bC ‫ ري‬OT -# .D
D. immediate fasciotomy of the arm .3N‫ ا‬OlS .E
E. pain medication

Un homme de 25 ans se plaint de douleur et


d'enflure de la main et de l'avant-bras,
d'engourdissement péribuccal et de vomissements
après avoir tenté d'attraper un serpent à sonnettes.
La pression artérielle est de 90/60 mm Hg. Tous les
traitements suivants sont appropriés, SAUF
A. réanimation liquidienne
B. l'administration de 10 flacons d'antivenin
C. mesure des facteurs de coagulation et des
plaquettes
D. fasciotomie immédiate du bras
E. médicaments contre la douleur
50. All of the following are contraindications to 6T‫ب أ‬.‫ أ‬Q ‫ب‬.2Q,‫ ه د ا‬6C  ‫آ‬ C
passing a nasogastric tube EXCEPT :‫"ي  "ا‬S
A. suspected perforation of the esophagus .‫يء‬C -5Q‫]*ب ا‬K‫ ا‬.A
B. confirmed perforation of the esophagus
.‫يء‬C "‫]*ب ا†آ‬K‫ ا‬.B
C. history of esophageal varices
D. nearly complete obstruction of the .‫ ايء‬6‫ة "وا‬J, .C
esophagus due to benign or malignant ‫*ت‬J  u ‫يء‬C ‫) آ‬.j ‫"اد‬4‫ ا‬.D
strictures .O]J.X ‫"ة أو‬J
E. presence of an esophageal foreign body .‫ ايء‬6 gL 34 ‫ ود‬.E

Toutes les conditions suivantes sont des contre-


indications au passage d'un tube nasogastrique
SAUF
A. une perforation présumée de l'œsophage
B. perforation confirmée de l'œsophage
C. l'histoire des varices oesophagiennes
D. obstruction presque complète de l'œsophage
en raison de sténoses bénignes ou malignes
E. la présence d'un corps étranger oesophagien

35 A
51. Which of the following statements is TRUE 6 ‫ف‬vQXv O.4&# OFJFi OJQ‫رات ا‬.S‫ أي  ا‬E
regarding radiographic differences in appearance :‫؟‬O?JC;‫ وا‬O*J5"‫ء ا‬SN‫ ا‬J# 6S‫ا?> ا‬
between the small and large bowels?  ‫ز‬. 6Q‫ ا‬O<SQ4‫ ا‬OJ2P‫ ا]ت ا‬.A
A. Transverse linear densities that arise from the
"‫ ه‬nlQ‫ ا‬D‫*> إ‬2# "Q‫ء و‬SN‫"ار ا‬
bowel wall and extend part way into the lumen
are characteristically seen in the small intestine .O*J5"‫ء ا‬SN‫ ا‬6 %J #
B. Haustrae are the transverse linear densities O<SQ4‫ ا‬OJ2P‫ ا]ت ا‬6‫ت ه‬.J.*‫ ا‬.B
found in the small intestine .O*J5"‫ء ا‬SN‫ ا‬6 ‫ادة‬
C. Transverse linear densities that extend # "Q 6Q‫ ا‬O<SQ4‫ ا‬OJ2P‫ ا]ت ا‬.C
completely across the bowel lumen are found 6 ‫ دة‬6‫ء ه‬SK‫ ا‬nl . ‫آ‬
inthe colon .‫ا*ن‬
D. Plicae circulares are the transverse linear
OJ2P‫ ا]ت ا‬6‫ ه‬OA‫ت ا"ا‬J&]‫ ا‬.D
densities found in the colon
E. Haustrae are less numerous and situated farther .‫ ا*ن‬6 ‫ ادة‬O<SQ4‫ا‬
apart than plicae circulares >S#  ‫"ا‬JS# -<Q‫ "دا و‬5‫ أ‬6‫ت ه‬.J.*‫ ا‬.E
.OA‫ت ا"ا‬J&]‫ أآ]  ا‬/S.‫ا‬
Laquelle des affirmations suivantes est VRAIE
concernant les différences radiographiques entre le
grele et le colon?
A. Les densités linéaires transversales qui
proviennent de la paroi intestinale et
s'étendent à mi-chemin dans la lumière sont
caractéristiquement visibles dans l'intestin
grêle
B. Les haustrae sont les densités linéaires
transversales trouvées dans l'intestin grêle
C. Les densités transversales linéaires qui
s'étendent complètement à travers la lumière
de l'intestin se trouvent dans le côlon
D. Les plicae circulaires sont les densités
linéaires transversales trouvées dans le
côlon
E. Les haustrae sont moins nombreux et situés
plus éloignés que les plicae circulaires
52. A 38 year old male presents with severe abdominal 6 3‫   وه   أ‬38 ‫ر‬ A
pain, diagnosed as pancreatitis. Which of the .‫&س د‬# ‫>ب‬Q# )[JP 3 qJ ،""j 2.‫ا‬
following drugs is NOT associated with acute ‫&س‬.‫>ب ا‬Q‫ ا‬- @‫ا‬Q K OJQ‫ ا‬O‫دو‬N‫أي  ا‬
pancreatitis?
:‫د؟‬F‫ا‬
(A) Heparin
(B) Furosemide .‫ر‬.‫ ه‬.A
(C) Rifampin ."JJ,‫و‬J .B
(D) Salicylates .J.T‫ ر‬.C
(E) Warfarin .‫ت‬vJ4JC, .D
.‫ وارر‬.E
Un homme de 38 ans présente une douleur
abdominale sévère, diagnostiquée comme une
pancréatite. Lequel des médicaments suivants n'est
pas associé à une pancréatite aiguë?
(A) Héparine
(B) Furosémide
(C) Rifampine
(D) Salicylates
(E) Warfarine

36 A
53. Which of the following is NOT a predisposing :‫@؟‬QT‫ر ا‬2Q WJ> v aJ 6Q‫أي  ا‬ B
factor for the development of a hernia? .‫*ء‬4Q,K‫ ا‬.A
(A) Ascites .O&4‫ ا‬.B
(B) Obesity
.64J‫ آ‬nJC .C
(C) Cystic fibrosis
(D) Chronic obstructive pulmonary disease .%‫ي ا‬A‫"اد ا‬4K‫ داء ا‬.D
(E) Peritoneal dialysis .65Ti ‫ دل‬.E

Lequel des éléments suivants n'est PAS un facteur


prédisposant au développement d'une hernie?
(A) L’ascite
(B) L'obésité
(C) La fibrose kystique
(D) Maladie pulmonaire obstructive
chronique
(E) Dialyse péritonéale
54. A 67-year-old female is admitted to the hospital g.4# DTQ4‫ ا‬D‫ إ‬ICX‫  أد‬67 ‫اأة ه‬ C
with a hip fracture after a fall. Which of the OJQ‫ ا‬O?N‫ أي  ا‬.‫*ط‬4‫" ا‬S# ‫ ارك‬6 4‫آ‬
following regimens constitutes appropriate venous ‫]ري ار"ي‬P‫[م ا‬K‫  ا‬O.,&‫ ا‬O5‫ ا‬
thromboembolism prophylaxis for this patient?
:‫؟‬O‫ ا‬b>
A. Intermittent pneumatic compression devices
B. Subcutaneous unfractionated heparin .-2*Q ‫ي‬A‫ ر‬:;< ‫ة‬%>‫ أ‬.A
C. Subcutaneouslow-molecular-weight heparin ."Cl‫ ا‬IF ‫أ‬%l JL ‫ر‬.‫ ه‬.B
D. Warfarin, with a target international ."Cl‫ ا‬IF /TP& 6W% 6‫ر ذات وز‬.‫ ه‬.C
normalized ratio (INR) of 1.5 to 2.0 J# (INR) 6‫ دو‬6? ‫"ل‬S - ،‫ وارر‬.D
E. A and B .2.0 D‫ إ‬1.5
.B ‫ و‬A .E
Une femme de 67 ans est admise à l'hôpital avec
une fracture de la hanche après une chute. Lequel
des schémas suivants constitue une prophylaxie
pour thromboembolie veineuse appropriée pour ce
patient?
A. Dispositifs de compression pneumatique
intermittente
B. Héparine sous-cutanée non fractionnée
C. Héparine sous-cutanée de bas poids
moléculaire
D. Warfarine, avec un rapport international
normalisé cible(INR) de 1,5 à 2,0
E. A et B

37 A
55. Mr Farid is diagnosed with stomach cancer; his ‫ أن‬gl ،" "J4C ‫"ة‬S ‫\ن‬, EJP 3 "* E
symptoms should include everything except: :‫  "ا‬6C  ‫ أا<) آ‬Q
A - Epigastric pains .OJ,j ‫م‬Kx .A
B - Vomiting food
.‫م‬S2‫ء ا‬6* .B
C - Hematemesis
D - recurrent phlebitis .‫ء دم‬6* .C
E - Constipation .‫ر‬Q ‫]ري‬X ‫>ب‬Q‫ ا‬.D
.‫ك‬4‫ إ‬.E
Mr Farid est diagnostiqué d’un cancer de
l'estomac ; ses symptomes devraient inclure tout
sauf :
A - Douleurs épigastriques
B - Vomissements alimentaires
C - Hématémèse
D - Phlébites récidivantes
E - Constipation
56. All these markers are associated with a severe ""j Ll, ‫ داء‬- @‫ا‬Q ‫ت‬vS‫ ا‬b‫ ه‬-J A
Sjögren with poor prognosis except: :‫‰  "ا‬J, EJP ‫ذات‬
A- Blepharitis .Tl‫>ب ا‬Q‫ ا‬.A
B- Hypocomplementemia
.‫ ا"م‬6 OQ‫ ا‬E* .B
C Purpura
D-Raynaud with digital ulcerations .O .C
E-Cryoglobulinemia .-#iN‫ ا‬6 ‫ *ت‬- ‫ ر&د‬.D
.‫ ا"م‬6 O‫د‬.‫&ت ا‬J#C;‫ ود ا‬.E
Tous ces marqueurs sont associés à un Sjögren
grave avec mauvais pronostic sauf :
A- Blépharite
B- Hypocomplémentémie
C- Purpura
D- Raynaud avec ulcérations digitales
E- Cryoglobulinémie
57. The epididymis is responsible for everything :‫  "ا‬6C  ‫†ول  آ‬4 ‘#.‫ا‬ E
except: .O&‫ات ا‬JF‫ * ا‬.A
A. Sperm transport .O&‫ات ا‬JF‫ ا‬O‫ ج آ‬.B
B. Maturation of sperm mobility
.O&‫ات ا‬JF‫ ا‬O#[X ‫ ج‬.C
C. Maturation of sperm fertility
D. Sperm storage .O&‫ات ا‬JF‫ ا‬%P .D
E. Secretion of 70% of the ejaculate liquid .‫ف‬b*‫ ا‬A,  %70 ‫ إاز‬.E

L’épididyme est responsable de tout ce qui suit


sauf :
A. Transport du sperme
B. Maturation de la mobilité du sperme
C. Maturation de la fertilité du sperme
D. Stockage du sperme
E. Sécrétion de 70% du liquide d’éjaculat

38 A
58. Which of the following is NOT a complication of ‫>ب‬Q‫ت ا‬T  aJ ‫ ه‬6Q‫أي  ا‬ D
acute pancreatitis? :‫د؟‬F‫&س ا‬.‫ا‬
A. Adult respiratory distress syndrome .J;.‫ &" ا‬OJ4T&Q‫ ا‬O*A‫ ا‬O‫ز‬vQ .A
(ARDS)
.6.C5 6C :J.] .B
B. Myocardial depression
C. Disseminated intravascular coagulopathy .Q& ‫]ري ور"ي‬X ‫ل‬vQ‫ ا‬.C
(DIC) .‫[ص‬Q‫ء ا‬, .D
D. Malabsorption .OJ,&# O#‫ آذ‬O4J‫ آ‬.E
E. Pancreatic pseudocyst

Lequel des énoncés suivants n'est PAS une


complication de la pancréatite aiguë?
A. Le syndrome de détresse respiratoire de
l'adulte (SDRA)
B. Dépression myocardique
C. coagulopathie intravasculaire disséminée
(CIVD)
D. Malabsorption
E. Pseudokyste pancréatique
59. A 55-year-old female presents to the ED with a ‫ارئ‬2‫ ا‬345 D‫  ت إ‬55 ‫اأة ه‬ C
fever 4 days after undergoing a laparoscopic >X "S# ‫ أم‬4 b& ‫   ارة‬6‫وه‬
cholecystectomy. What is the MOST likely cause .‫ي‬J?& ‫[ل ارة‬WQ,K
of the fever?
:‫ارة؟‬FC FJ ]‫آ‬N‫ ا‬g.4‫ ه ا‬
(A) Pneumonia
(B) Thrombophlebitis .OA‫ ذات ر‬.A
(C) Urinary tract infection .‫]ري‬P‫>ب ار" ا‬Q‫ ا‬.B
(D) Wound infection .OJ# Š4 ‫ن‬Q‫ ا‬.C
(E) Deep venous thrombosis .‫ح‬l‫ ا‬3|l .D
.@J ‫]ر ور"ي‬X .E
Une femme de 55 ans se présente à l'urgence avec
une fièvre 4 jours après avoir subi une
cholécystectomie laparoscopique. Quelle est la
cause la plus probable de la fièvre?
(A) Pneumonie
(B) Thrombophlébite
(C) infection des voies urinaires
(D) Infection de la plaie
(E) Thrombose veineuse profonde

39 A
60. Clostridium tetani is the organism responsible for  ‫†ول‬4‫ ا‬6|l‫ ا‬S‫ ا‬6‫ ه‬O‫از‬%‫ ا‬OJ]2‫ا‬ B
causing tetanus. All of the following statements ‫از‬%C O.4&# OJQ‫رات ا‬.S‫ آ ا‬.‫از‬%‫داء ا‬
regarding tetanus are TRUE EXCEPT :‫  "ا‬OFJFi
A. tetanospasmin, an exotoxin produced by
)lQ& 6‫ر‬X ‫ن‬T‫ ه ذ‬،J,.,Q .A
C.tetani, is responsible for the clinical
manifestations of tetanus  ‫†ول‬4 ‫ وه‬،O‫از‬%‫ ا‬OJ]2‫ا‬
B. tetanospasmin is released into the CNS after .‫از‬%‫ "اء ا‬O4‫ا?ه ا‬
C. tetani crosses the blood–brain barrier ‫>ز‬l‫ ا‬X‫ دا‬D‫ إ‬J‫ز‬.,Q ‫ر‬FQ .B
C. clinical manifestations of tetanus include OJ]2‫ر ا‬. "S# ‫ي‬%‫ اآ‬6.[S‫ا‬
generalized muscular rigidity, violent .‫ ا"ي‬6L"‫ ا‬%FC O‫از‬%‫ا‬
muscular contractions, and instability of the ‫از‬%‫ "اء ا‬O4‫ ا?ه ا‬Q .C
autonomic nervous system
،nJ& 6C EC* ،3S 6C gC[
D. the most common presenting complaint of
patients with generalized tetanus is pain and .‫إرادي‬v‫ ا‬6.[S‫>ز ا‬l‫ ا‬6 ‫*ار‬Q,‫"م ا‬
stiffness in the masseter muscle "& Jj ]‫آ‬N‫رزة ا‬.‫ إن اى ا‬.D
E. tetanospasmin prevents the release of 6‫ ه‬3S‫از ا‬%‫"اء ا‬# J#[‫ ا‬D<‫ا‬
GABA and glycine from presynaptic nerve .O;<‫ ا‬OCS‫ ا‬6 gC[‫ و‬3‫أ‬
terminals GABA ‫ر‬F -& J,.,Q .E
.5  ‫[ب‬N‫  >ت ا‬J4JCL‫و‬
Clostridium tetani est l'organisme responsable du
.OJ.‫ا‬
tétanos. Toutes les affirmations suivantes
concernant le tétanos sont VRAIES SAUF
A. la tétanospasmine, une exotoxine produite
par C. tetani, est responsable des
manifestations cliniques du tétanos
B. la tétanospasmine est libérée dans le SNC
après C. tetani se retrouve dans le sang
C. les manifestations cliniques du tétanos
incluent la rigidité musculaire généralisée,
les contractions musculaires violentes et
l'instabilité du système nerveux autonome
D. la plainte la plus fréquente des patients
atteints de tétanos généralisé est la douleur
et la rigidité dans le muscle masséter
E. La tétanospasmine empêche la libération de
GABA et la glycine des terminaisons
nerveuses présynaptiques
61. Which of the following animals is NOT a potential ‫   "اء‬aJ ‫ ه‬OJQ‫ات ا‬JF‫أي  ا‬ D
carrier of rabies? :‫؟‬gC‫ا‬
(A) Dogs .‫ب‬v‫ ا‬.A
(B) Bats
.JTP‫ ا‬.B
(C) Skunks
(D) Squirrels .‫ن‬#?‫ ا‬.C
(E) Cows .g&4‫ ا‬.D
.*.‫ ا‬.E
Lequel des animaux suivants N'EST PAS un
potentiel porteur de la rage?
(A) Chiens
(B) Chauves-souris
(C) Mouffettes
(D) Écureuils
(E) Vaches

40 A
62. What is the least likely reason for epigastralgia ‫ أ|&ء‬6,‫ ا‬3 FJ 5N‫ ا‬g.4‫ ه ا‬ B
during pregnancy :‫؟‬F‫ا‬
A. Decreased lower oesophageal sphincter tone .OJCT4‫ [ة ايء ا‬O* 6 E5& .A
B. Decreased secretion of gastrin
.Q,;‫ إاز ا‬6 E5& .B
C. Mechanical pressure due to the size of the
uterus .3‫ ا‬3l  u 6JJ :;< .C
D. Decreased gastric emptying .‫"ي‬S‫ ا{اغ ا‬6 E5& .D
E. Increase in gastroesophageal reflux .6W‫"ي ا‬S‫ع ا‬l‫ر‬K‫ زدة ا‬.E

Quelle est la raison la moins probable pour une


epigastralgie durant la grossesse
A. Diminution du tonus du sphincter
oesophagien inferieur
B. Diminution de sécrétion de la gastrine
C. Pression mécanique du à la taille de
l’utérus
D. Diminution de la vidange gastrique
E. Augmentation de reflux gastro-
oesophagien
63. Secondary Sjögren can be found with all these b‫ ه‬-J 6 ‫ ا]ي‬Ll, ‫ا" داء‬Q ‫ أن‬ B
diseases except: :‫اض  "ا؟‬N‫ا‬
A- Lupus .O.Ab‫ ا‬.A
B- Behçet's disease
.Il># ‫ داء‬.B
C- Rheumatoid arthritis
D- Mixed Connectivity (Sharp) .‫ي‬J|‫ ا‬iT‫>ب ا‬Q‫ ا‬.C
E-Scleroderma .:CQP‫ ام ا‬uJ4&‫ داء ا‬.D
."Cl‫ ا‬gC[ .E
La maladie de Sjögren secondaire peut être
retrouvée avec toutes ces maladies sauf :
A- Lupus
B- Maladie de Behçet
C- Polyarthrite rhumatoïde
D- Connectivite mixte ( Sharp)
E- Sclérodermie

41 A
64. For a 40 y old patient with an acute necrotizing ‫&س‬# ‫>ب‬Q‫  ا‬6S  40  / A
pancreatitis of biliary origin, what is the best ‫  ه أ‬،‫اوي‬Ti ‫ي د ذات ["ر‬P
therapeutic proposal? :‫ج؟‬vSC ‫اح‬Q5‫ا‬
A. The drainage technique of choice is
J?&Q‫ ا‬6‫رة ه‬QP‫ ا‬n[Q‫ ا‬OJ&* .A
endoscopic retrograde
cholangiopancreatography when it is 6,&.‫[ ا‬Q‫ ا‬O*2# 6CX‫ا"ا‬
available. ..‫ا‬Q ‫ &" ن‬-‫اوي اا‬T[‫ا‬
B. Surgical treatment should be preferred as .‫ |ن‬:P‫ آ‬OCT 6‫ ه‬OJ‫ا‬l‫ ا‬OlS‫ ا‬.B
second line. D*. 6"Q‫م ا‬l>‫ ذات ا‬n[Q‫ت ا‬J&* .C
C. Minimally invasive drainage techniques .‫ل‬SQ,K‫ ا‬OCJC5
remain poorly used. 6‫ ه‬6"Q‫م ا‬l>‫ ذات ا‬n[Q‫ت ا‬J&* .D
D. Minimally invasive drainage techniques are
.bJT&Q‫ ا‬O.Si
difficult to implement

Pour une pancréatite aiguë nécrosante d’origine


biliaire, quelles sont les propositions thérapeutiques
exactes ?
A. La technique de drainage de choix est la
cholangiopancréatographie rétrograde
endoscopique lorsque celle-ci est
disponible.
B. Le traitement chirurgical doit être privilégié
en deuxième intention.
C. Les techniques de drainage mini-invasives
restent peu utilisées.
D. Les techniques de drainage mini-invasives
sont difficiles à mettre en application.
65. What is the first-line test for a peptic ulcer? :‫؟‬OJ>‫ ا‬O*‫ ا‬EF 6 ‫ول‬N‫ ا‬:P‫ ه ا‬ C
A. Basal gastric tubing .‫"ة ا*"ي‬S‫ب ا‬.‫ أ‬.A
B. Basal gastrinemia .‫ ا"م ا*"ي‬Q,L .B
C. Research of Helicobacter Pylori in the
-5‫ ا‬6 OJ#‫ا‬.‫ت ا‬,*Q‫ي  ا‬FQ‫ ا‬.C
antral region
D. Abdominal computed tomography .‫ا;ري‬
E. Searching for hyperthyroidism .2.C ‫ري‬F 6*.\ [ .D
.‫ي  ط ط ا"رق‬FQ‫ ا‬.E
Quel est l'examen à réaliser en première intention
dans le bilan d'un ulcère gastroduodénal ?
A. Tubage gastrique basal
B. Gastrinémie basale
C. Recherche d'Helicobacter Pylori dans la
région antrale
D. Tomodensitométrie abdominale
E. Recherche d'une hyperthyroïdie

42 A
66. A 49-year-old woman is admitted for an evaluation ‫ر‬S‫ ا‬g.4# 3JJ*Q ISX ، 40 ‫ اأة ه‬D
of weakness. She complains of fatigue with - ،‫ت‬vS‫"ام ا‬PQ,‫ ا‬- gS   6‫ ه‬.‫ه‬#
repetitive muscle use, with significant fatigue and " "5 gSQ‫ا ا‬b‫ ه‬.‫م‬J‫ ا‬O> 6 -C# 4‫ و‬J.‫ آ‬gS
dysphagia by the end of the day. Her activities have
6l2<‫ ا‬aT @J< ‫ وه&ك‬،>\  ‫رز‬# #
been significantly limited due to her fatigue, and
there is significant orthopnea.During her 137 ‫دم‬i :‫ي‬.P‫ ا‬JCFQ‫> ا‬G‫ أ‬،3JJ*Q‫ أ|&ء ا‬.‫رز‬#
evaluation, laboratory analysis reveals: Sodium 137 "‫ر‬C‫ آ‬،‫ ل‬/ ‰ 6CC 3.8 ‫م‬J,# ،‫ ل‬/ ‰ 6CC
meq/L, potassium 3.8 meq/L, chloride 94 meq/L, .‫ ل‬/ ‰ 6CC 31 ‫ت‬F 6| ،‫ ل‬/ ‰ 6CC 94
bicarbonate 31 meq/L. An arterial blood gas shows 3C 60 PaCO2 ،PH 7.33 >G‫ أ‬6‫ز ا"م ا‬L
pH of 7.33, PaCO2 60 mmHg, and PaO2 65 OJ&J4‫ ا["ر ا‬OSj‫ * أ‬.@.A‫ ز‬3C 65 PaO2 ‫ و‬،@.A‫ز‬
mmHg. A chest x-ray is interpreted as “poor %92 ‫ ه‬Jl4‫وآ‬N‫ع ا‬.j‫ إ‬."6*J>‫>" ا‬l‫ ا‬nS< " ‫ه‬
inspiratory effort.” The oxygen saturation is 92%
.OJSJ.\ O‫ ه و‬O‫و‬Q‫ا‬-O>Q‫ ا‬EF .O;‫  ا‬6
on room air. A ventilation-perfusion scan has
normal perfusion. Which of the following tests will g., nS 6 FJ ]‫آ‬N‫ ه ا‬OJQ‫ص ا‬FT‫أي  ا‬
most likely identify the cause of this patient’s :‫؟‬O‫ ا‬b‫ &" ه‬64T&Q‫ ا‬FQ‫ا‬
respiratory acidosis? .‫"غ‬C ‫ري‬F 6*.\ 4 .A
A. CT scan of the brain .3FT‫" ا‬J4‫ول أوآ‬N ‫ر‬QK‫"رة ا‬5 .B
B. Diffusing capacity for carbon monoxide .‫ ايء‬J?& .C
C. Esophagoscopy ‫*ء‬CQ,K‫ ا‬OJS<‫ و‬6) O4*‫ ا‬OJF‫ ا*"رة ا‬.D
D. Forced vital capacity (supine and upright)
.(O*Q,K‫ ا‬OJS<‫ و‬6‫و‬
E. Pulmonary angiogram
.OA‫ ر‬OJA‫رة و‬i .E
Une femme de 49 ans est admise pour une
évaluation de faiblesse. Elle se plaint de fatigue
musculaire, avec une fatigue importante et une
dysphagie en fin de la journée. Ses activités ont été
considérablement limitées en raison de sa fatigue,
et il y a une orthopnée significative. Au cours de
son évaluation, l'analyse en laboratoire révèle:
Na 137 meq / L, potassium 3,8 meq / L, chlorure 94
meq /L, bicarbonate 31 meq / L. Un gaz du sang
artériel montre un pH de 7,33, PaCO2 de 60 mmHg
et PaO2 de 65 mmHg. La radiographie thoraxest
interprétée comme un «mauvais effort
inspiratoire». La saturation d'oxygène est de 92% à
l'air ambiant. Un Scan de ventilation-perfusion est
normal. Lequel des tests suivants sera très
probablement necessaire pour identifier la cause de
l’acidose respiratoire ce patient?
A. Scanner cérébral
B. Capacité de diffusion du monoxyde de
carbone
C. Oesophagoscopie
D. Capacité vitale forcée (en décubitus dorsal
et en position verticale)
E. angiographie pulmonaire

43 A
67. A 75-year-old smoking man with a history of ‫ة‬l& ‫\ن‬4 ‫ة‬J, -  75  X" ‫ر‬ C
laryngeal cancer treated by surgical resection and ‫ج‬v‫ و‬6‫[ل ا‬WQ,# )QlS I qJ
radiotherapy presents for thirst with a feeling of ‫ر‬S‫ وا‬2S‫  وه   ا‬،OSjN#
sticky tongue. He has noticed this for a few months
 ‫>ر و‬j OS# b& Š‫ ذ‬mK "* .‫ج‬% ‫ن‬4C#
but his condition is currently worsened after an
angina treated by antibiotic therapy. All these J‫>ب ز‬QK )<S "S# )Q ‫ءت‬, ‫ أن‬q.
measures are effective to improve his condition JS‫ ا‬b‫ آ ه‬.‫ي‬J ‫د‬# OlS‫ ا‬I qJ
except: :‫)  "ا‬Q J4F 6 OS 6‫ه‬
A- Prescribe antimycotics .‫ت‬2 ‫ د‬ni‫ و‬.A
B- Make the HbA1C test .HBA1C EF ‫ إاء‬.B
C- Make a biopsy of the salivary glands .OJ#SC‫  ا;"د ا‬O%X bX‫ أ‬.C
D- Drink a lot of water
.‫  اء‬J]‫ب ا‬j .D
E- Start a PPI and stop smoking
 n5Q‫ون وا‬.‫ ا‬OP :.]# ‫"ء‬.‫ ا‬.E
Un homme de 75 ans, tabagique avec antécédent de .JX"Q‫ا‬
cancer du larynx traité par résection chirurgicale et
radiothérapie, se présente pour soif avec langue
collante. Il a remarqué ceci depuis quelques mois
mais son état est majoré actuellement après une
angine traitée par antibiothérapie. Toutes ces
mesures sont efficaces pour améliorer son état
sauf :
A- Prescrire des antimycotiques
B- Faire le dosage de HbA1C
C- Faire une biopsie des glandes salivaires
D- Boire beaucoup d’eau
E- Débuter un IPP et arrêter le tabac
68. A 40-year-old woman, working as a cook in a   ،3S2 6 OX.\ S ، 40 ‫ اأة ه‬D
restaurant, complains of burning eyes, especially at O - ‫ء‬4‫ ا‬6 S‫ أ|&ء ا‬OiX ،>J&J 6 @
work in the evening with pruritus and redness. ‫ء‬# 4;‫ &" ا‬Q5† ‫اض‬N‫ ا‬b‫ >"أ ه‬.‫واار‬
They are temporarily quenched by the application
:‫  "ا؟‬K.* .QS EJP  6C  ‫ آ‬.‫رد‬.‫ا‬
of cold water. All these diagnoses are possible
except: .‫ت‬,"S‫ ا‬g.4# PQ# OFQC ‫>ب‬Q‫ ا‬.A
A- Conjunctivitis by irritation because of .‘.2‫ ارة ا‬g.4# PQ# OFQC ‫>ب‬Q‫ ا‬.B
lenses .Ll, ‫ داء‬.C
B- Conjunctivitis by heat irritation of .OJ*‫>ب ا‬Q‫ ا‬.D
cooking .‫ول‬%S J ‫ف‬T .E
C- Sjögren's disease
D- Keratitis
E- Isolated eye dryness

Une femme âgée de 40 ans, cuisinière dans un


restaurant, se plaint de brûlûres oculaires, surtout le
soir au travail avec prurit et rougeur. Elles sont
calmées transitoirement par l’application d’eau
froide. Tous ces diagnostics sont possibles sauf :
A- Conjonctivite par irritation aux lentilles
B- Conjonctivite par irritation à la chaleur
des cuissons
C- Maladie de Sjögren
D- Kératite
E- Sècheresse oculaire isolée

44 A
69. A 52-year-old alcoholic man presents to a local 345 D‫  إ‬،‫ل‬F‫ ا‬DC " ، 52  ‫ ر‬D
emergency room with purulent, productive cough, 6 @J< ،‫""ي‬i uQ& ‫ل‬S,  6S ‫ارئ وه‬2‫ا‬
shortness of breath, right-sided chest pain, and ‫ع‬T‫ وار‬،‫  ا["ر‬N‫ ا‬gl‫ ا‬DC 3‫ أ‬،aT&Q‫ا‬
fever. He thinks his symptoms started a few days
"& .‫ أم‬OS# b& ‫"أت‬# )<‫*" أن أا‬QS )‫ إ‬.‫ارة‬
ago. On examination, he has a temperature of
38.8°C, heart rate of 96 beats/min, respirations of 96 gC*‫ت ا‬#< ‫"ل‬S ،‫م‬°38.8 )‫ ار‬I‫ آ‬،EFT‫ا‬
22 breaths/min, oxygen saturation of 85% on room %85 Jl4‫وآ‬N‫ع ا‬.j‫ إ‬،‫ د‬/ aT 22 aT&Q‫ ا‬،‫ د‬/ O.
air, and a blood pressure of 115/92mmHg. He has )&,‫ أ‬.@.A‫ ز‬3C 92/115 ‫ ا"م‬:;<‫ و‬،O;‫  ا‬6
poor dentition and fetid breath. There is dullness to *F‫ ا‬DC ‫ &" ا*ع‬OJi‫ ه&ك أ‬.Q aT&‫ وا‬OWJ,
percussion over the right lower lung field, and rales .gl‫ ا‬OJA&| ‫ة‬XX -4‫ و‬،N‫ ا‬6CT4‫ي ا‬A‫ا‬
are auscultated bilaterally. A chest radiograph 34*‫ ا‬6 N‫ ا‬gl‫ ا‬6 OQ ‫ ا["ر‬OSj‫?> أ‬
shows a right-sided opacity in the superior portion
-A, ‫ى‬Q4 ‫ ود‬- N‫ ا‬T,N‫ ا‬ETC ‫ي‬CS‫ا‬
of the right lower lobe with an air-fluid level
present. There appears to be right-sided .N‫ ا‬gl‫ ا‬DC 6&Q "C[ ‫"و‬.  DC ‫ ه&ك‬.6A‫ها‬
parenchymal consolidation as well. Which of the ‫&دا‬Q,‫ ا‬،FJ ]‫آ‬N‫ ا‬g.4‫ ا‬S‫ ه ا‬6Q‫أي  ا‬
following is the most likely etiologic organism :‫ض؟‬S‫ا ا‬b>
based on this presentation? .‫ء‬4C‫ت ا‬J.‫ ا‬.A
A. Candida glabrata .‫ا‬%CT{‫وس ا‬J .B
B. Influenza virus .OJC4‫ات ا‬2TQ‫ ا‬.C
C. Mycobacterium tuberculosis
.OJ>‫ ا‬O"*S‫ ارات ا‬.D
D. Peptostreptococcus
E. Streptococcus pneumonia .OA‫*"ت ا‬S‫ ا‬.E

Un homme alcoolique de 52 ans se présente à la


salle d'urgence avec une toux purulente et
productive, essoufflement de la respiration, douleur
thoracique droite et de la fièvre. Il pense que ses
symptômes ont commencé il y a quelques jours. Il a
une température de 38,8 ° C, la fréquence cardiaque
de 96battements / min, respirations de 22
respirations / min, saturation en oxygène de 85% à
l'air ambiant, et une pression artérielle de
115/92mmHg. Il a une mauvaise dentition et une
mauvaise haleine. Il existe une matité à la
percussion au champ pulmonaire inférieur droit, et
les râles sont auscultés bilatéralement. Une
radiographie thoracique montre une opacité du côté
droit dans la partie supérieure du lobe inférieur
droit avec un niveau d'air-fluide présent avec
possiblement une consolidation parenchymateuse
droite aussi. Lequel des suivants est l'organisme
étiologique le plus probable basé sur cette
présentation?
A. Candida glabrata
B. Virus de la grippe
C. Mycobacterium tuberculosis
D. Peptostreptococcus
E. La pneumonie à Streptococcus

45 A
70. A 30-year-old female with end-stage renal disease )QC 6 ‫ي‬C‫  داء آ‬6S  30 ‫ ه‬D]‫ أ‬E
who receives her dialysis through a tunneled ،>TQ‫ آ‬6 )J*T ‫ة‬245 . ‫ل‬FQ -P‫ و‬OJA>&‫ا‬
catheter in her shoulder presents with fever and .>?‫ ا‬T,‫ أ‬6 ""j 3‫   ارة وأ‬6‫ت وه‬
severe low back pain. On examination, she is
‫ و‬،)5SQ‫ و‬O JL I‫ آ‬،EFT‫&" ا‬
uncomfortable and diaphoretic but
hemodynamically stable. She has a soft 2/6 early )J<.*‫ "@ ا‬OPT >" .‫*ة‬Q4 O"‫> ا‬QJJ&‫د‬
systolic flow murmur. Her line site is red and warm ‫ أ ودا‰ دون إازات‬g*]‫ ا‬-5 .6/2 O ‫ة‬.
with no pustular exudates. She is very tender over .‫>ه‬G T,‫ أ‬DC al‫"" &" ا‬j 3w# S 6‫ ه‬.O]#
her lower back. Neurologically, she is completely ‫ت‬t J‫ " د‬K . OJC, 6‫ ه‬،OJ.[S‫ ا‬OJ&‫ ا‬
intact. There is no evidence of Janeway lesions, ‫"اد آت ا"م‬S .‫ روث‬-*# ‫ أو‬،C,‫ *" أو‬،‫اي‬
Osler nodes, or Roth spots. Her white count is  ‫ آ‬Q ‫ أن‬gl ‫ري‬T‫ ا‬3JJ*Q‫ ا‬.16700 ‫ء‬J.‫ا‬
16,700. Immediate evaluation should include all of
:‫  "ا‬6C
the following except:
A. MRI of the lumbar spine .OJ&2*‫*ات ا‬TC 64J\&; J‫ ر‬.A
B. removal of her dialysis catheter .‫ل‬FQ‫ة ا‬245 O‫ إزا‬.B
C. transthoracic echocardiogram .‫ ا["ر‬. gC*‫"ى ا‬i :J2P .C
D. two sets of blood cultures followed by v J4 >JC ‫رع ا"م‬% ‫ن‬Ql .D
vancomycin as well as gram-negative .‫ ا;ام‬OJ.C, )J2; 
coverage .2.C ‫ري‬F 6*.\ 4 .E
E. Abdominal CT scan

Une femme de 30 ans atteinte d'insuffisance rénale


terminale qui effectue une dialyse à l'aide d'un
cathéter tunnelisé pres de son épaule présente de la
fièvre et une lombalgie sévère. À l'examen, elle est
inconfortable et diaphorétique mais
hémodynamiquement stable. Elle a un léger
souffle systolique 2/6 précoce. Son site de ponction
est rouge et chaud sans exsudats pustuleux. Le bas
du dos est sensible à la palpation.
Neurologiquement, le bilan clinique est
complètement normal. Il n'y a aucune évidence de
lésions de Janeway, de nœuds d'Osler ou de taches
de Roth. Son nombre de blancs est de 16 700.
L’évaluation immédiate devrait inclure tous les
éléments suivants, sauf:
A. IRM du rachis lombaire
B. retrait de son cathéter de dialyse
C. échocardiographie transthoracique
D. deux séries de cultures sanguines suivies de
vancomycine ainsi que la couverture des
gram-négatifs
E. Scanner abdominal

46 A
71. A 78-year-old male presents to the clinic C‫دة وه  أ) آ‬JS‫ ا‬D‫   إ‬78  ‫ ر‬D
complaining that every time he shaves with a .6‫  ا‬gJ; ،3J*Q4‫ ا‬O5vF‫س ا‬# )&5‫@ ذ‬CF
straight razor, he passes out. His symptoms have /S# 6 .JJ<‫ ا> ا‬6 ‫اض‬N‫ ا‬b‫ ) ه‬I|"
been occurring for the last 2 months. Occasionally,
.Šb‫ آ‬6‫  ا‬gJ; ،O*J< O5 ‫ &" "ي‬،‫ن‬JN‫ا‬
when he puts on a tight collar, he passes out as
well. The loss of consciousness is brief, he has no ،O*‫ا‬Q ‫ادر‬# )" aJ‫ و‬،%J‫ و‬6‫ب  ا‬J;‫ا‬
associated prodrome, and he feels well afterward. mFC O"*‫ ا‬OJ<‫) ا‬J, .Š‫" ذ‬S# "J )w# S‫و‬
His past medical history is notable for hypertension ‫) ه‬v .‫ول ا"م‬Q4‫ وط آ‬:;< ‫ع‬T‫ار‬
and hypercholesterolemia. His only medication is I‫ آ‬،‫ي‬4‫ ا‬EFT‫ &" ا‬.:* "‫ز‬J‫ر‬C‫"روآ‬J‫ه‬
hydrochlorothiazide. On physical exam his vital ‫]&ء‬Q,# 6SJ.\ gC*‫ ا‬EF‫ و‬،OJSJ.\ OJF‫را) ا‬j‫إ‬
signs are normal, and his cardiac exam is normal 6[JPQ‫ ا‬EFT‫ ه ا‬6Q‫ أي  ا‬.-#‫ اا‬gC*‫ت ا‬i
with the exception of a fourth heart sound. Which
:‫؟‬g4N‫ ا‬6Q‫ا‬
of the following is the most appropriate next
diagnostic test? .‫ ا{>دي‬gC*‫"ى ا‬i :J2P .A
A. Stress echocardiography .‫م‬JQ‫د&ز ا‬w# [Q‫ ا‬.B
B. Adenosine thallium scan .O.5C ‫ري‬F 6*.\ [ .C
C. Computed tomogram of the neck .6.4‫ ا‬gJl‫Š ا‬J" .D
D. Carotid sinus massage .EFT‫ ا‬O‫ \و‬O‫ إ‬.E
E. Tilt table test

Un homme de 78 ans se présente à la clinique en se


plaignant que chaque fois qu'il se rase avec un
rasoir, il s'évanouit. Ses symptômes ont eu lieu au
cours des 2 derniers mois. Parfois, lorsqu'il enfile
un col étroit, il s'évanouit également. La perte de
conscience est brève, il n'a aucun prodrome associé,
et il se sent bien après. Ses antécédents médicaux
sont notables pour l'hypertension et
l'hypercholestérolémie. Son seul médicament est
l'hydrochlorothiazide. À l'examen physique, ses
signes vitaux sont normaux et son examen
cardiaque est normal à l'exception d'un quatrième
bruit cardiaque. Lequel des tests suivants est le plus
approprié?
A. Échocardiographie de stress
B. Scan à l'adénosine thallium
C. CT scan du cou
D. Massage des sinus carotidiens
E. test de la table inclinable

47 A
72. A 42-year-old woman presents to the emergency room ‫"ء‬# - ‫ارئ‬2‫ ا‬345 D‫  ت إ‬42 ‫ اأة ه‬E
with acute onset of shortness of breath. She recently had ‫رج‬X >"‫ زرة ا‬6 ‫ا‬X† I‫ آ‬.aT& @J ‫د‬
been to visit her parents out of town and rode in a car
for about 4 h each way. Two days ago, she developed a "S# .l‫ آ ا‬6 ‫ت‬, 4 6‫ا‬F ‫رة‬J4‫دت ا‬5‫ و‬O&"‫ا‬
mild calf pain and swelling, but she thought that this ‫*"ت‬Q‫ و&> ا‬،OC#‫ ا‬6 ‫ ورم‬:J4# 3w# ‫ت‬Sj ،J
was not unusual after having been sitting with her legs DC ‫د‬QK‫س وا‬Cl‫" ا‬S# ‫ دي‬JL ‫  أا‬3 ‫ا‬b‫أن ه‬
dependent for the recent trip. On arrival to the ،‫ارئ‬2‫ ا‬345 D‫> إ‬i‫ &" و‬.OC‫Š ا‬C 6 >JC‫ر‬
emergency room, she is noted to be tachypneic. The
vital signs are: blood pressure 98/60 mmHg, heart rate :;< :OJF‫رات ا‬j{‫ ا‬.aT ‫ع‬4 O 6 >‫ أ‬m
114 beats/min, respiratory rate 28 breaths/min, SaO2 114 gC*‫ت ا‬#< ‫"ل‬S ،@.A‫ ز‬3C 60/98 ‫ا"م‬
92% on room air, weight 89 kg. The lungs are clear  6 %92 SaO2 ،‫ د‬/aT 28 aT&Q‫ ا‬O, ،‫د‬/O.
bilaterally. There is pain in the right calf with ‫ ه&ك‬.J.l‫ ا‬6 OF<‫ وا‬OA‫ ا‬.s‫ آ‬89 ‫ ازن‬،O;‫ا‬
dorsiflexion of the foot, and the right leg is more
swollen when compared to the left. An arterial blood gas ‫ وا‬،‫*"م‬C 6‫>ا‬G ‫ ا]&ء‬- D&J‫ ا‬OC#‫ ا‬6 3‫أ‬
measurement shows a pH of 7.22, PaCO2 18 mmHg, ‫س‬J5 >G‫ أ‬.‫ى‬4J‫  ا ا‬.‫ أآ‬# O‫ر‬Q D&J‫ا‬
and PaO2 68 mmHg. Kidney and liver function are ،@.A‫ ز‬3C18 PaCO2 ،7.22 PH 6‫ز ا"م ا‬L
normal. A CT scan is performed and confirms a "* .OJSJ.\ OJC‫" وا‬.‫ ا‬OTJG‫ و‬.@.A‫ ز‬3C 68 PaO2
pulmonary embolus. All of the following agents can be
used alone as initial therapy in this patient except: .OA‫ ر‬Oi ‫آ" ود‬w‫ري و‬F 6*.\ [Q ISX
A. enoxaparin, 1 mg/kg SC twice daily )JA"# OlS‫"ا> "ه آ‬PQ,‫  ا‬OJQ‫ا ا‬S‫آ ا‬
B. fondaparinux, 7.5 mg SC once daily :‫]&ء‬Q,# O‫ ا‬b‫&" ه‬
C. tinzaparin, 175 units/kg SC once daily J "Cl‫ ا‬IF s‫آ‬/sC 1 ،‫ر‬#4‫ إآ‬.A
D. unfractionated heparin IV adjusted to maintain .J
activated partial thromboplastin time (aPTT)
two to three times the upper limit of normal ‫" ة وا"ة‬Cl‫ ا‬IF sC 7.5 ،a‫ر&آ‬#‫ "ا‬.B
E. warfarin, 7.5 mg PO once daily to maintain INR .J
at 2–3 ‫" ة‬Cl‫ ا‬IF s‫ آ‬/ ‫ و"ة‬175 ،‫ر‬#‫ا‬%&J .C
.J ‫وا"ة‬
Une femme de 42 ans se présente à la salle d'urgence
avec un début d'essoufflement aigu. Elle a récemment
6 )Q ‫"ل‬S "‫أ ور‬%l JL ‫ر‬.‫ ه‬.D
rendu visite à ses parents en region et a roulé dans une 6A%l‫ ا‬JQ,v#.‫ ز و‬OJi †
voiture pendant environ 4 heures à chaque fois. Il y a .6SJ.2‫" ا‬F‫  ا‬DC‫ أ‬J ‫ل ة أو‬ST‫ا‬
deux jours, elle a développé une légère douleur au ‫ ة وا"ة‬3T‫  \@ ا‬- 7.5 ،‫ وارر‬.E
mollet et un gonflement, mais elle pensait que ce n'était
pas inhabituel après avoir été assise avec ses jambes .3-2 DC INR OJ[ J
pendantes lors du récent voyage. À son arrivée à la salle
d'urgence, elle est notée tachypnéique. Les signes vitaux
sont: tension artérielle de 98/60 mmHg, fréquence
cardiaque de 114 battements / min, fréquence
respiratoire de 28 respirations / min, SaO2 de 92% dans
l'air ambiant, poids de 89 kg. Les poumons sont dégagés
bilatéralement. Il y a une douleur dans le mollet droit
avec dorsiflexion du pied, et la jambe droite est plus
enflée par rapport à la gauche. Une mesure des gaz du
sang artériel montre un pH de 7,22, PaCO2 18 mmHg,
et PaO2 68 mmHg. La fonction rénale et hépatique est
normale. Un scanner hélicoïdal est réalisé et confirme
une embolie pulmonaire. Tous les agents suivants
peuvent être utilisés seuls comme traitement initial chez
elle sauf:
A. énoxaparine, 1 mg / kg SC deux fois par jour
B. Fondaparinux, 7,5 mg SC une fois par jour
C. tinzaparine, 175 unités / kg SC une fois par jour
D. héparine IV non fractionnée ajusté pour
maintenir le temps de céphaline activée (TCA)
deux à trois fois la limite supérieure de la
normale
E. warfarine, 7,5 mg PO une fois par jour pour
maintenir l'INR à 2-3
48 A
73. A 31-year-old woman presents to your clinic  6‫دة وه‬JS‫ ا‬D‫ ت إ‬، 31 ‫ اأة ه‬E
complaining of painful arthritis that is worse in the "& ‫ح‬.[‫ &" ا‬35TQ ‫ي‬b‫ وا‬3† iT ‫>ب‬Q‫ ا‬
mornings when she wakes up. She was recently ‫>ب‬QK ‫ن‬J gJ.\ "& ‫ا‬X† >JJ* ‫ *" ى‬.m*JQ4
evaluated by an ophthalmologist for uveitis in her
‫ي‬.P‫ ا‬EFT‫> * ا‬G‫ أ‬.D&J‫&> ا‬J 6 OJ%*‫ا‬
right eye. A recent laboratory report shows an
erythrocyte sedimentation rate of 48 mm/h. Which 6Q‫ أي  ا‬.O, / 3C 48 ‫ ا"م‬T] O, ،JXN‫ا‬
of the following will be helpful in distinguishing  a‫ ا&آ‬n‫>ب ا;ر‬Q‫@ ا‬T DC "4 ‫ف‬,
relapsing polychondritis from rheumatoid arthritis :‫ او"ي؟‬iT‫>ب ا‬Q‫ا‬
(RA)? iT ‫>ب‬Q‫ ا‬- @‫ا‬Q iT ‫>ب‬Q‫ ا‬.A
A. Arthritis associated with RA is nonerosive. .‫™آ‬Q JL ‫رو"ي‬
B. Eye inflammation is absent in relapsing n‫>ب ا;ر‬Q‫ ا‬- @‫ا‬Q K JS‫>ب ا‬Q‫ ا‬.B
polychondritis.
.a‫ا&آ‬
C. Relapsing polychondritis will not present
with vasculitis. ‫>ب‬Q‫ ا‬- @‫ا‬Q K a‫ ا&آ‬n‫>ب ا;ر‬Q‫ ا‬.C
D. Relapsing polychondritis will present with .OJ‫و‬N‫ا‬
high-titer of rheumatoid factor.  - @‫ا‬Q a‫ ا&آ‬n‫>ب ا;ر‬Q‫ ا‬.D
E. The arthritis of relapsing polychondritis is .6 ‫ر‬JS ‫رو"ي ذات‬
asymmetric. JL ‫ ه‬n‫>ب ا;ر‬Q‫ ا‬6 iT‫>ب ا‬Q‫ ا‬.E
.G&Q
Une femme de 31 ans se présente à votre clinique
pour une arthrite douloureuse qui s'aggrave le matin
quand elle se réveille. Elle a récemment été évaluée
par un ophtalmologiste pour une uvéite dans l'œil
droit. Un rapport de laboratoire récent montre un
taux de sédimentation des érythrocytes de 48 mm /
h. Lequel des éléments suivants sera utile pour
distinguer la polychondrite récidivante de la
polyarthrite rhumatoïde (PR)?
A. L'arthrite associée à la polyarthrite
rhumatoïde est non érosive.
B. L'inflammation des yeux est absente dans la
polychondrite récidivante.
C. La polychondrite récidivante ne présentera
pas de vascularite.
D. La polychondrite récidivante présentera
avec le titre élevé du facteur rhumatoïde.
E. L'arthrite de la polychondrite récurrente est
asymétrique.

49 A
74. A 44-year-old woman seeks evaluation for irregular JL O>‫ "ور> ا‬JJ* gC2  44 ‫ اأة ه‬A
menstrual cycles with heavy menstrual bleeding. ‫ن‬w# ‫ *" أدت‬.""j /J n% - O*‫ا‬Q‫ وا‬O?Q&‫ا‬
She reports that her menses had been regular with ‫&ت‬S‫ ا‬A‫ أوا‬b&  28 ‫"ورة‬# JSJ.\ ‫> آن‬J
28-day cycles since her early twenties. However,
>‫ دور‬،OJ<‫ ا‬OQ4‫> ا‬jN‫ ا‬6 ،‫ و‬.‫ ه‬
for the past 6 months, her cycles have been 22–25
days with heavy associated bleeding that is unusual ‫ دي‬JL ""j n% - O.[Q‫  و‬25-22 I‫آ‬
for her. She has had rare hot flashes and sleep ‫اب‬2<‫ وا‬O&X, ‫ت‬.‫"ث > ه‬F  ‫ درا‬.> O.4&#
disturbance. She is requesting assistance in b‫ ه‬DC ‫ة‬2J4C ‫"ة‬4‫ ا‬gC2 6‫ ه‬.‫ ا&م‬6
controlling these symptoms. You suspect she is ‫ع‬2*‫ ا‬.5  ‫ة‬QT‫ ا‬6 >‫*"ت أ‬Q‫ *" ا‬.‫اض‬N‫ا‬
perimenopausal, and hormonal testing on day 2 of "‫> أآ‬JF 6]‫م ا‬J‫ ا‬6 6>‫ ا‬EFT‫ وا‬،q2‫ا‬
her menses confirms this suspicion. You are ‫  ي‬-# ‫ج‬v ‫ح‬Q* I‫ أ‬.‫ا‬b‫*دك ه‬Q‫Š ا‬
considering treatment with oral contraceptives for
JL  ‫>  "وث‬Q5‫ أا<> و‬DC ‫ة‬2J4C
control of her symptoms and to protect against
unintended pregnancy. Which of the following ‫ب‬.2Q,‫ د ا‬.QS ‫ف‬, 6Q‫ أي  ا‬.‫*[د‬
would be considered a contraindication to use of :‫؟‬OT‫ ا‬F‫ ا‬-& ‫ب‬. ‫"ام‬PQ,K
oral contraceptive pills: .‫ ه‬.A
A. Her age .q2‫ارة ا‬%L .B
B. Menorrhagia .‫ة‬J[*‫ ا"ورات ا‬.C
C. The short cycles .‫رق‬N‫ ا‬.D
D. Insomnia
.O&X4‫ت ا‬.>‫ ا‬.E
E. Hot flushes

Une femme de 44 ans consulte pour une évaluation


pour de cycles menstruels irréguliers avec des
saignements menstruels abondants. Elle rapporte
que ses règles ont été régulières avec des cycles de
28 jours depuis sa jeune vingtaine. Cependant, au
cours des 6 derniers mois, ses cycles ont été de 22 à
25 jours avec des saignements associés lourds qui
sont inhabituels pour elle. Elle a eu des bouffées de
chaleur rares et des troubles du sommeil. Elle
demande de l'aide pour contrôler ces symptômes.
Vous soupçonnez qu'elle est en périménopause, et
le test hormonal au jour 2 de ses règles confirme
cette suspicion. Vous envisagez un traitement avec
des contraceptifs oraux pour contrôler ses
symptômes et pour vous protéger contre une
grossesse non désirée. Lequel parmi tous les
éléments suivants serait considéré comme une
contre-indication à l'utilisation de pilules
contraceptives orales, chez elle:
A. Son age
B. La menorragie
C. Les cycles courts
D. L’insomnie
E. Les bouffées de chaleur

50 A
75. About knee trauma, all of the following statements  OFJFi OJQ‫رات ا‬.S‫ آ ا‬،O.‫ اآ‬O< O.4&# C
are correct EXCEPT ONE. Which ? :‫"ا‬
A. A sprained knee may be accompanied by a .OJv‫ ه‬Ox - O.‫ اآ‬6|‫ا@ و‬Q ‫  أن‬.A
meniscal lesion
.OJv>‫ ا‬Ot‫) ا‬.., ‫  أن ن‬°15 6&]‫ ا‬.B
B. A 15 ° flessum can be caused by a meniscal
lesion " K ،O‫و‬%S‫ ا‬OJv>‫ت ا‬t‫ ا‬6 .C
C. In isolated meniscal lesions, no effusion is .‫ب‬.[‫ا‬
found Ox ‫ أن ه&ك‬6&S ‫ ري‬OJ#l{‫ ا‬OvS‫ ا‬.D
D. A positive sign of McMurray means there is .OJv‫ه‬
a meniscal lesion O"i g.4 ‫  أن‬J2P‫ ا‬O.‫ اآ‬6|‫ و‬.E
E. Serious sprained knee can cause patella .OT<‫ا‬
shock

A propos des traumatismes du genou, tous les


énoncés suivants sont corrects SAUF UN. Lequel ?
A. Une entorse du genou peut s'accompagner
d'une lésion méniscale
B. Un flessum de 15° peut être causé par une
lésion méniscale
C. En cas de lésion méniscale isolée, on ne
retrouve jamais d'épanchement
D. Un signe de McMurray positif signe une
lésion méniscale
E. Une entorse grave du genou peut causer un
choc rotulien
76. A 35 y old man complains of bone pain. Among b‫ < ه‬.3?S‫ ا‬6 3‫    أ‬35  ‫ ر‬B
these benign tumoral lesions, which is the one that ‫ي‬b‫ ا‬3N‫ ا‬- @‫ا‬Q ‫ي‬b‫  ه ا‬،‫"ة‬JF‫ ا‬OJ‫ت ار‬t‫ا‬
corresponds to pain with nocturnal recrudescence? :‫؟‬vJ ‫"د‬lQ
A. Osteogenic exostosis
.6? ‫ ن‬.A
B. Osteoid osteoma
C. Non-osteogenic fibroma .6? ‫ ورم‬.B
D. The essential bone cyst .w&‫ ا‬6? JL 6TJ ‫ ورم‬.C
E. The isolated chondroma .OJ,,N‫?م ا‬S‫ ا‬O4J‫ آ‬.D
.‫ول‬%S 6‫و‬L ‫ ورم‬.E
Un homme de 35 ans se plaint de douleurs
osseuses. Parmi ces lésions tumorales bénignes,
quelle est celle qui détermine des douleurs à
recrudescence nocturne ?
A. L'exostose ostéogénique
B. L'ostéome ostéoïde
C. Le fibrome non ostéogénique
D. Le kyste essentiel des os
E. Le chondrome isolé

51 A
77. A 37-year-old woman has significant nocturnal O*2& 6 J.‫ آ‬6CJ 3‫  أ‬6S  37 ‫ اأة ه‬D
pain in the scapular area as a result of work at her ‫ل‬S # >Ql "* .>%& 6 S‫  ا‬u nQ‫ا‬
home. You treat it effectively for a calcification "* .O‫ ق اآ‬OCS‫ و ا‬6 -<Q‫ ا‬aCQ‫ ا‬
located in the tendon of the supraspinous muscle.
 -J#,‫" "ة أ‬S# ‫ى‬X‫Š ة أ‬S ‫ور‬QC ‫ت‬
She consults you again a few weeks after this
episode for a difficulty in mobilizing the shoulder. "* .nQ‫Š ا‬F 6 O#Si g.4# ‫Š ا&ب‬C ‫"وث‬
You notice a limitation of passive mobility on  .‫ي‬4‫ ا‬EFT‫"ودة &" ا‬F OCST& O‫ آ‬I?K
clinical examination. What is the most likely :‫؟‬FJ ]‫آ‬N‫ ا‬EJPQ‫ه ا‬
diagnosis? .O‫ق ا ق اآ‬% .A
A. Rupture of the supraspinatus tendon .i‫ا‬Q O‫>ب و ق اآ‬Q‫ ا‬.B
B. Unbroken tendonitis of the supraspinatus .aC *# .C
C. Residual calcification
.gF4C #5 O?TF ‫>ب‬Q‫ ا‬.D
D. Retractable Capsulitis
E. Symptomatic Chondrocalcinosis .6<S‫ ا‬n‫س ا;ر‬v‫ آ‬.E

Une femme de 37 ans présente des douleurs


nocturnes importantes dans le moignon scapulaire à
la suite de travaux de transformations dans sa
maison. Vous la traitez correctement et
efficacement pour une calcification située dans le
tendon du muscle supraépineux. Elle vous consulte
à nouveau quelques semaines après cet épisode
pour une difficulté à mobiliser l'épaule. Vous
constatez une limitation de la mobilité passive à
l'examen clinique. Quel est le diagnostic le plus
probable ?
A. Rupture du tendon du supraépineux
B. Tendinite non rompue du supraépineux
C. Calcification résiduelle
D. Capsulite rétractile
E. Chondrocalcinose symptomatique
78. Which of the following organisms is the major 3?S 64JA‫ ا‬g.4‫ ا‬6‫ ه‬OJQ‫ت ا‬S‫أي  ا‬ A
cause of most travelers’ diarrhea? :‫؟‬4‫ت &" ا‬K>,{‫ا‬
(A)E.coli .OJ*‫ ا‬OJ{‫ ا‬.A
(B)Campylobacter
.OTJ2S‫ ا‬.B
(C)Vibrio
(D)Giardia .‫ ات‬.C
(E)Shigella .‫رد‬J .D
.vC;Jj .E
Lequel des organismes suivants est la principale
cause de la plupart des diarrhées du voyageur?
(A) E. coli
(B) Campylobacter
(C) Vibrio
(D) Giardia
(E) Shigella

52 A
79. A 26 y old female complains of dyspnea. >G‫ أ‬.aT& @J<    26 ‫ ه‬D]‫أ‬ C
Ultrasound reveals mitral stenosis. Which of the ‫ات‬JJ;Q‫ أي  ا‬.6 @J ‫ق ا[ت‬
following pathological change does not occur in :‫؟‬6Q‫@ ا‬JQ‫ ا‬6 ‫"ث‬F K OJQ‫ ا‬OJ<‫ا‬
mitral stenosis .4N‫ذ ا‬N‫ ا‬:;< ‫ زدة‬.A
A) Increased left atrial pressure .4N‫ذ ا‬N‫ ا‬-, .B
B) Left atrium dilatation .6&J2# 3P .C
C) Ventricular hypertrophy .‫]ات‬P‫ ا[م ا‬.D
D) Embolisation of clots
.OFJFi v‫ أ‬O#N‫ آ ا‬.E
E) All of the above

Une femme de 26 ans se plaint de dyspnée.


L'échographie révèle une sténose mitrale. Lequel
des changements suivants ne se produit pas dans la
sténose mitrale
A) La pression de l’oreillette gauche est
élevée
B) Une dilatation de l’oreillette gauche
C) Hypertrophie ventriculaire gauche
D) Embolisation de caillots à distance
E) Tout ce qui précède

53 A
80. Which of the following patients is MOST likely to  "JTQ4 ‫ أن‬FJ ]‫آ‬N‫ ه ا‬D<‫ء ا‬K†‫ أي  ه‬C
benefit from antibiotic therapy in addition to n[‫@ و‬j D‫ إ‬O<‫ إ‬OJF‫دات ا‬# ‫ج‬vS‫ا‬
abscess incision and drainage? :‫اج؟‬P‫ا‬
A. A previously healthy female with a
‫"ة‬L 6 ‫اج‬P# O#[‫* و‬#, OJC, D]‫ أ‬.A
Bartholin’s gland abscess and no history
suggesting a high risk for STD OJ \P D‫ إ‬J ‫ة‬J, K‫ و‬J#
B. A healthy 25-year-old male with recurrence .J4& O*& ‫اض‬N
of a pilonidal abscess that first occurred 2 ‫اج‬P‫  ار ا‬6S  25  ‫ ر‬.B
years previously .JQ&, b& ‫ي "ث أول ة‬b‫ي وا‬S‫ا‬
C. A febrile 50-year-old female with NIDDM ‫ع ارة‬T‫    ار‬50 ‫ ه‬D]‫ أ‬.C
and recurrence of axillary hydradenitis DC "QS JL ‫ي‬, ‫  داء‬6S 6‫وه‬
suppurativa
6FJ*‫ ا‬OJ2#{‫ ا‬OJ5S‫>ب ا;"د ا‬Q‫ وا‬J4{‫ا‬
D. A 35-year-old male with a sebaceous gland
cyst that has been present for 2 years and .‫ر‬Q‫ا‬
has now become infected O"L O4J‫  آ‬6S  35  ‫ ر‬.D
E. An intravenous drug user without fever or IF.i‫ن أ‬t‫ وا‬JQ&, b& ‫ت‬w 6Q‫ وا‬OJ‫زه‬
tachycardia presenting with a 2-cm2 deltoid .O|lQ
abscess that developed 7 days after “skin ‫"ون‬# ‫* ار"ي‬F‫"رات  \@ ا‬P " .E
popping” OJ‫ ا"ا‬6 ‫اج‬X   gC5 ‫رع‬4 ‫ارة أو‬
"ClQ‫ أم  "ا‬7 "S# w ‫ي‬b‫ وا‬² 3, 2
Lequel des patients suivants est le plus susceptible
de bénéficier d'un traitement antibiotique en plus de ."‫"ي‬Cl‫ا‬
l' incision et drainage d’un abcès?
A. Une femme auparavant en bonne santé avec
un abcès de Bartholin et aucun antécédent
suggérant un risque élevé de MST
B. Un homme de 25 ans en bonne santé avec
récidive d'un abcès pilonidal survenu 2 ans
auparavant
C. Femme fébrile âgé de 50 ans avec DNID et
récidive d'hydraénite axillaire suppurée
D. Un homme de 35 ans avec un kyste de
glande sébacée présent depuis 2 ans et est
maintenant infectée
E. Un utilisateur de drogue par voie
intraveineuse sans fièvre ou tachycardie
présentant un abcès de 2-cm2 du deltoïde
qui s'est développé 7 jours après une
«abcedation cutanee»

54 A
81. A female patient reports a recent sexual contact O5v DC ‫ا‬X† I‫> آ‬w# D]‫ أ‬O ‫ *" أدت‬E
with a partner who has just been treated for a ‫ض‬# )Q#i‫ إ‬.Qj‫)  ا‬v 3 Šj - OJ4&
suspected STD. During the examination, a urine  .J#l‫ آن إ‬F‫ر ا‬.QX‫ إ‬،EFT‫ أ|&ء ا‬.J4& ‫&*ل‬
test for pregnancy is found to be positive. Which of
O‫ ا‬F .5 )Ti‫ و‬t‫ي ا‬JF‫ه اد ا‬
the following antibiotics is safe to prescribe before
referring the patient to an obstetrician for prenatal :‫دة؟‬K‫ ا‬.5 O&SC "J‫ و‬OJA4 gJ.2
care? .JCJ,‫ أ‬.A
(A) Acyclovir .J4‫و‬Q‫ أز‬.B
(B) Azithromycin .3J4JT, .C
(C) Cefixime .‫"ازول‬J‫و‬Q .D
(D) Metronidazole .OFJFi v‫ أ‬O#N‫ آ ا‬.E
(E) All of the above

Une patiente rapporte un contact sexuel récent avec


un partenaire qui vient d'être traité pour une MST
suspectée. Lors de l'examen, un test urinaire de
grossesse s'avère positif. Lequel des antibiotiques
suivants est sûr de prescrire avant de référer le
patient à un obstetricien pour suivi :
A. Acyclovir
B. Azithromycine
C. Céfixime
D. Métronidazole
E. Toutes ces réponses

55 A
82. All of the following statements about  OFJFi ‫ ط ا"رق‬E* ‫ ل‬OJQ‫رات ا‬.S‫ آ ا‬B
hypothyroidism are TRUE EXCEPT :‫"ا‬
A. prevalence is greater in women than in men .‫ء أآ]  ال‬4&‫ر &" ا‬Q‫ ا‬.A
B. amiodarone and lithium may mask the
OJQ‫اض ا‬K‫ن ا‬JTP "5 ‫م‬J]JC‫دارون وا‬J‫ أ‬.B
presentation secondary to elevation of
thyroid hormone levels .65‫ى ا>ن ا"ر‬Q4 ‫ع‬T‫ر‬K
C. in secondary hypothyroidism, thyroid- ‫ى ا>ن‬Q4 ،‫ ط ا"رق ا]ي‬E* 6 .C
stimulating hormone (TSH) levels are .‫ دة‬/TP& ‫( ن‬TSH)‫"رق‬C ‫ض‬F‫ا‬
usually low "& -Aj ‫ ط ا"رق ه‬E* ‫  أن‬3L# .D
D. although hypothyroidism is common in "5 ‫اض‬N‫ ”ن "رة ا‬، 60  .‫ أآ‬3‫ ه‬
those older than 60 years, a paucity of ..Si EJPQ‫ ا‬Sl
symptoms may make the diagnosis difficult
‫ ط‬E*  u ‫ ط ا"رق‬E* "S#  .E
E. postablation hypothyroidism is a cause
ofprimary hypothyroidism .6A".‫ا"رق ا‬

Toutes les déclarations suivantes sur


l'hypothyroïdie sont VRAIES SAUF
A. la prévalence est plus grande chez les
femmes que chez les hommes
B. l'amiodarone et le lithium peuvent masquer
la présentation secondaire à l'élévation des
taux d'hormones thyroïdiennes
C. dans l'hypothyroïdie secondaire, les niveaux
de thyréostimuline (TSH) sont
généralement faibles
D. bien que l'hypothyroïdie soit courante chez
les personnes âgées de plus de 60 ans, un
manque de symptômes peut rendre le
diagnostic difficile
E. post-hypothyroïdie est une cause de
l'hypothyroïdie primaire
83. All the following abnormalities are common in a ‫ [ب‬/ "& OSAj 6‫ ه‬OJQ‫ت ا‬#‫ا‬2<K‫ آ ا‬C
patient with myxedema EXCEPT :‫  "ا‬OJ\P O‫ذ‬#
A. respiratory: hypoventilation, hypoxia .Ol4‫ أآ‬E* ،O> E* :OJ4T& .A
B. central nervous system: confusion, lethargy,
.O#.JL ،‫ل‬X ،‫ك‬.‫ ار‬:‫ي‬%‫ آ‬6.[ ‫ >ز‬.B
coma
C. electrolyte: hypernatremia, water retention .‫س ء‬.Q‫ ا‬،‫دم ا"م‬i E* :O‫رد‬j .C
D. cardiovascular: bradycardia .gC5 †\. :OJA‫ و‬OJ.C5 .D
E. thermoregulatory: hypothermia .34l‫ض ارة ا‬TP‫ ا‬:O‫ ار‬.E

Toutes les anomalies suivantes sont fréquentes chez


les patients avec myxœdème SAUF
A. respiratoire: hypoventilation, hypoxie
B. système nerveux central: confusion,
léthargie, coma
C. électrolyte: hypernatrémie, rétention d'eau
D. cardiovasculaire: bradycardie
E. thermorégulation: hypothermie

56 A
84. Regarding trauma of the abdomen in a patient with ‫دث‬F ‫ض‬S / OJ&2.‫<ت ا‬C O.4&# C
a car accident , the viscera most commonly affected :‫؟‬6‫ ه‬Jj ]‫آ‬N‫|ة ا‬wQ‫ء ا‬N‫ ا‬،‫رة‬J,
is: .".‫ ا‬.A
A. The liver
.‫"ة‬S‫ ا‬.B
B. The stomach
C. The spleen .‫ل‬F2‫ ا‬.C
D. The intestine .‫ء‬SN‫ ا‬.D
E. The colon .‫ ا*ن‬.E

Lors des traumatismes de l'abdomen chez un


accidente de la voie publique, le viscère le plus
souvent touché est:
A - Le foie
B - L'estomac
C - La rate
D - L'intestin
E - Le côlon
85. To achieve best results with antibiotic prophylaxis, ،‫ي‬JF‫د ا‬# 6A5‫ج ا‬vS‫ ا‬- uAQ ‫@ أ‬J*FQ A
care must be taken when it is given in the course of :O‫ا‬l‫) أ|&ء ا‬A2‫ر &" إ‬bF‫ ا‬6X gl
surgery: " < ،A‫ دا‬6‫ا‬l‫ ا‬S‫@ ا‬.4 ‫ أن‬gl .A
A. Always precede the surgical procedure,
I5‫  و‬،n[‫ و‬O, D‫ إ‬O,  D[5‫أ‬
within a maximum of 1h to 1h30, from the
time of the injection of the anesthesia ."PQ‫* ا‬
B. Perform in the post-operative phase, within 30  D‫ <" " أد‬،O‫ا‬l‫" ا‬S# )A2‫ إ‬.B
a minimum of 30 min. to 1 h. .O, D‫ إ‬O*J5‫د‬
C. It should be done during the procedure, 3Q "& ،6‫ا‬l‫ ا‬S‫ل ا‬vX 3Q ‫ أن‬gl .C
once the anesthetic product is injected, / ‫ و‬O, 12  D[5‫ < " أ‬،"PQ‫* ا‬
within a maximum of 12 hours and / or .O, 24 JA&]Q,‫أو ا‬
exceptionally 24 hours.
D[5‫ < " أ‬،6‫ا‬l‫ ا‬S‫@ ا‬.4 ‫ أن‬gl .D
D. Precede the procedure, within a maximum
of 1 to 2 days .J D‫ م إ‬
E. Is best done 6 hours after the incision .@‫ت  ا‬, I, "S# )A2‫ إ‬T .E

Pour que l’antibioprophylaxie ait des résultats


souhaités, il faut veiller au moment approprié de
son application en chirurgie. Elle doit :
A. Toujours précéder l'acte opératoire, dans un
délai maximum de 1h à 1h30, au moment de
l’injection de l’anesthésie.
B. S’effectuer en phase post-opératoire, dans
un délai minimum de 30 min. à 1h.
C. se faire pendant l’acte opératoire, une fois
que le produit anesthésique sera injecté,
dans un délai maximum de 12 h et/ou
exceptionnellement 24h.
D. Précéder l’acte opératoire, dans un délai
maximum de 1j à 2j.
E. Se fait au mieux 6h apres l’incision

57 A
86. Each of the following cardiac conditions is OJ O.A‫ ذ‬- @‫ا‬Q OJQ‫ ا‬OJ.C*‫ت ا‬KF‫ ا‬-J B
associated with systemic lupus erythematosus :‫  "ا؟‬O‫>ز‬
(SLE) EXCEPT .‫ر‬Q‫>ب ا‬Q‫ ا‬.A
(A) pericarditis .>#N‫@ ا‬J .B
(B) aortic stenosis .‫&ق‬P‫ ا‬.C
(C) angina .6‫و‬L 6SC< ‫>ب‬Q‫ ا‬.D
(D) costochondritis .gC*‫ ا‬OC ‫>ب‬Q‫ ا‬.E
(E) myocarditis

Chacune des affections cardiaques suivantes est


associée au lupus érythémateux disséminé (LED)
SAUF
(A) péricardite
(B) sténose aortique
(C) l'angine
(D) costochondrite
(E) myocardite
87. Which of the following is the least common O‫"ا‬# 6 Jj 5N‫ض ا‬S‫ ه ا‬6Q‫أي  ا‬ C
symptom at the very beginning of pregnancy? :‫؟‬F‫ا‬
A. Cramps in the lower abdomen .2.‫ ا‬T,‫ت أ‬l& .A
B. Painful breasts .‫م ا]"ي‬Kx .B
C. Edema of the lower limbs .DCT4‫\اف ا‬N‫ ا‬O‫ وذ‬.C
D. Fatigue
.gS .D
E. Delay of menses
./JF‫ ا‬JXw .E
Parmi les suivantes quel est le symptôme qui est
rare au tout début d’une grossesse
A. Crampes dans le bas ventre
B. Seins douloureux
C. Œdème des membres inferieurs
D. Fatigue
E. Retard de règles
88. A pregnant woman in her 32nd week of gestation is ‫ت‬TC, ‫&ول‬Q F‫  ا‬32 ‫ع‬.,N‫ ا‬6  ‫ اأة‬A
given magnesium sulfate for pre-eclampsia. The O4‫رة ا‬j{‫ ا‬.‫ع‬l‫ر‬K‫م  أ *"ت ا‬%J&;‫ا‬
earliest clinical indication of hypermagnesemia is :6‫م ا"م ه‬%J&; ‫ط‬T# O#i— ‫ة‬.‫ا‬
a. Loss of deep tendon reflexes .OJ?S‫ – ا‬O‫ت ا‬4S&‫ *"ان ا‬.A
b. Flaccid paralysis .X‫ ر‬Cj .B
c. Respiratory arrest .aT& n5 .C
d. Hypotension .:;< ‫ض‬TP‫ ا‬.D
e. Stupor .‫ت‬., .E

Une femme enceinte a sa 32ème semaine de


gestation a eu du sulfate de magnesium pour pré-
éclampsie. La première indication clinique de
hypermagnésémie est
a. Perte des réflexes ostéo-tendineux
b. Paralysie flasque
c. Arrêt respiratoire
d. Hypotension
e. Stupeur
58 A
89. Five days after an uneventful cholecystectomy, an ‫ &" اأة‬J. ،‫[ل ارة‬WQ,‫ ا‬OJC  ‫ أم‬O4X "S# B
asymptomatic middle-aged woman is found to have [‫دم ا‬i ‫ى‬Q4 ‫ أن‬،OJ< K ،S‫ ا‬O2,Q
a serum sodium level of 120 meq/L. Proper :‫ ه‬JF[‫ ا‬6vS‫ ا‬J#"Q‫ ا‬.‫ ل‬/ ‰ 6CC 120 ‫ه‬
management woul be .Q‫ ا‬-T 6FC ‫ل‬CF ‫ء‬2‫ إ‬.A
a. Administration of hypertonic saline .F‫ء اء ا‬2”# "J*Q‫ ا‬.B
solution .‫ز‬v.C "A‫ا‬%‫ ا‬JjQ‫ ا‬.C
b. Restriction of free water .‫ل دي‬F .D
c. Plasma ultrafiltration ."JJ,‫و‬J ‫ل‬SQ,# ""j 6# ‫ إدرار‬.E
d. Hemodialysis
e. Aggressive diuresis with furosemide

Cinq jours après un incident cholécystectomie, une


asymptomatique femme d'âge moyen presente un
niveau sérique de sodium de 120meq / L. Une
bonne conduite serait être une.
a. L'administration de sérum salé hypertonique
solution
b. Restriction d'eau libre
c. Plasma ultrafiltration
d. L'hémodialyse
e. Diurèse agressive avec le furosémide
90. A 50-year-old patient presents with symptomatic ‫ت‬J[  6S ‫  وه‬، 50  / D
nephrolithiasis. He reports that he underwent ) ‫" أي‬5 ‫ *" أد أ) آن‬.‫ أاض‬- O*‫ا‬Q OC‫آ‬
jejunoileal bypass for morbid obesity when he was 39  ‫ &" آن‬O‫"ا‬.‫ ا‬g.4# )JTAT )JAi ‫زة‬l
39. One would expect to find :‫ ود‬-5Q‫  ا‬.
a. Pseudohyperparathyroidism .‫ ط ط درق آذب‬.A
b. Hyperuric aciduria .‫ل‬.‫ ا‬/ ‫ ط‬g.4# OJ OCJ# .B
c. “Hungry bone” syndrome ."-Al‫ ا‬3?S‫ " ا‬O‫ز‬vQ .C
d. Hyperoxaluria .‫ل‬.‫ت ا‬K‫ا‬%‫ ط أوآ‬.D
e. Sporadic unicameral bone cysts .O;‫ ا‬O‫ اد) أد‬OJ? ‫ت‬4J‫ آ‬.E

Un homme de 50 ans se présente avec lithiase


renal symptomatique. Il rapporte qu'il a subi un
bypass jéjuno-ileal pour l'obésité morbide quand il
avait 39 ans. On s'attend à trouver une.
a- Pseudohyperparathyroidism
b. Hyperuricosurie
c. Syndrome de «l'os Hungry"
d. Hyperoxalurie
e. Sporadiques kystes osseux unicamérales

59 A
91. A 65-year-old man undergoes a technically difficult OJ&* ‫ ذات‬6&2# 6l Q. -P ، 65  ‫ ر‬C
abdominoperineal resection for a rectal cancer OJCS‫ل ا‬vX D*CQ qJ ،3J*Q4 ‫\ن‬, g.4# O.Si
during which he receives three units of packed red 4 ‫" ور‬S# .‫اء‬F‫ث و"ات  آت ا"م ا‬v|
blood cells. Four hours later in the intensive care  ‫"ة‬# ‫ف‬%& ‫ ه‬O*AT‫ ا‬O&S‫ ا‬6 ‫ت د‬,
unit he is bleeding heavily from his perineal ‫ ز‬،OCS‫] ا‬PQ‫ت ا‬,‫>ت درا‬G‫ أ‬.6lS‫ح ا‬l‫ا‬
wound. Emergency coagulation studies reveal ،‫ف‬%&‫ وز ا‬6A% JQ,v#.‫ ز و‬،J.‫وو‬#
normal prothrombin, partial thromboplastin, and ‫ و‬OST JL .JT‫ب ا‬P ‫ت‬lQ& .OJSJ.\ >C‫آ‬
bleeding times. The fibrin degradation products are ‫"اد‬S‫ و‬/TP& .JT‫ى ا[  " ا‬QF
not elevated but the serum fibrinogen content is FJ ]‫آ‬N‫ ا‬g.4‫ ا‬.QJ ‫و‬J/70000 ‫ت‬FJT[‫ا‬
depressed and the platelet count is 70,000/µL. The :‫ ه‬n%&C
most likely cause of the bleeding is .‫ * ا"م‬S ‫ رد‬6 JXw .A
a. Delayed blood transfusion reaction .6‫ ذا‬6& .J CF .B
b. Autoimmune fibrinolysis .6‫ا‬l‫* ا‬F‫ ا‬6 ‫ وء دي زف‬.C
c. A bleeding blood vessel in the surgical .VIII S‫ ز ا‬.D
field .‫ارة‬F‫ض ا‬TP‫  ا‬u ‫]ي‬P ‫ل‬vQ‫ إ‬.E
d. Factor VIII deficiency
e. Hypothermic coagulopathy

Un homme de 65 ans subit une résection


abdominoperinealel techniquement difficile pour
un cancer du rectum au cours de laquelle il reçoit
trois unités de globules rouges concentrés. Quatre
heures plus tard dans l’unité de soins il saigne
abondamment de sa blessure périnéale. Urgence
études de coagulation révèlent normale
prothrombine, céphaline, et les temps de
saignement. Les produits de dégradation de la
fibrine ne sont pas élevés, mais le fibrinogène
sérique est diminue et la numération plaquettaire
est de 70.000 /ul. La cause la plus probable de
l'hémorragie est une.
a. Réaction de transfusion sanguine retardée
b. Fibrinolyse auto-immune
c. Un vaisseau sanguin qui saigne
d. Le déficit en facteur VIII
e. Coagulopathie par hypothermie

60 A
92. A victim of blunt abdominal trauma requires a OJ&2# O<‫ ر‬OJF ".‫ء  ا‬% ‫[ل‬WQ,‫ إاء ا‬gC2Q C
partial hepatectomy. He is rapidly transfused with 8 "* .‫&Š ا"م‬#  ) ‫ و"ات  ا"م‬8 * 3 "* .OCJC‫آ‬
units of appropriately crossmatched packed red ‫ل‬%.‫ ا‬-5‫ف  ا‬%& )w# ‫ش‬S{‫ ا‬OL 6 )JC m
blood cells from the blood bank. He is noted in the ‫  ا أن ن‬.6‫ا‬l‫ار"ي و ا@ ا‬
recovery room to be bleeding from intravenous ‫ت ا"م‬FJTi E*  u "& ‫]ري‬P‫ل ا‬vQK‫ا‬
puncture sites and the surgical incision. His :‫؟‬OJQ‫] ا‬PQ‫ أي  ا ا‬6 ‫وز‬
coagulopathy is likely due to thrombocytopenia and .:* II .A
deficiencies of which clotting factors? .VII ‫ و‬II .B
a. II only .VIII ‫ و‬V .C
b. II and VII .X ‫ و‬IX .D
c. V and VIII .XII ‫و‬XI .E
d. IX and X
e. XI and XII

Une victime de traumatisme abdominal fermé


nécessite une hépatectomie partielle. elle est
rapidement transfusée 8 unités de manière
appropriée compatibilisé les globules rouges
concentrés à partir de la banque de sang. Il est noté
dans la salle de réveil à saigner des sites de
ponction par voie intraveineuse et l'incision
chirurgicale. Sa coagulopathie est probablement
due à une thrombopénie et une carence dans un des
facteurs de coagulation?
a. II seulement
b. II et VII
c. V et VIII
d. IX et X
e. XI et XII

61 A
93. An elderly diabetic woman with chronic steroid- % 6.[5 u&‫ي و‬4‫"اء ا‬# O#[ O&4 ‫ اأة‬C
dependent bronchospasm has an ileocolectomy for )JTAT O‫ أر‬LT ISX ،"‫و‬JQ4‫ ا‬DC "QS
a perforated cecum. She is taken to the ICU ‫ة‬%‫ اآ‬O&S‫ و"ة ا‬D‫ إ‬ICX‫ *" أد‬.‫ر‬N‫ ا]*ب ا‬g.4#
intubated and is maintained on broad-spectrum J#‫ دو‬،nJ2‫ ا‬-,‫ي وا‬J ‫د‬# OJi - gJ.&Q#
antibiotics, renal-dose dopamine, and a rapid 6]‫م ا‬J‫" ا‬S# .-, J|w ‫و" ذات‬JQ,‫ و‬،OC‫ آ‬Ol#
steroid taper. On postoperative day 2 she develops ،(‫ف‬°102) ‫م‬°39.2 ‫ارة‬F# ‫ع‬T‫ "ث > ار‬O‫ا‬lC
a fever of 39.2ȃC (102.5ȃF), hypotension, E* O.P‫ ا‬3J*‫>ت ا‬G‫ وأ‬،‫ت‬., ،:;< ‫ض‬TP‫ا‬
lethargy, and laboratory values remarkable for ]‫آ‬N‫ ا‬EJPQ‫ إن ا‬.‫م ا"م‬J,# ‫ع‬T‫ ا"م وار‬,
hypoglycemia and hyperkalemia. The most likely :‫د ه‬F‫"ث ا‬F‫ا ا‬b> FJ
diagnosis of this acute event is .‫ دم‬uX .A
a. Sepsis .‫ ا"م‬3l E* .B
b. Hypovolemia .?‫"ة ا‬L ‫[ر‬5 .C
c. Adrenal insufficiency .‫ د‬6.J. P& .D
d. Acute tubular necrosis .‫ي‬, 6CX F .E
e. Diabetic ketoacidosis

Une femme diabétique âgéeavec bronchospasme


chronique cortico-dépendante a eu une
ileocolectomy pour un caecum perforé. Elle est pris
à l'USI intubée et est maintenue sous large spectre
antibiotiques, la dopamine à dose rénale, et un
stéroïde rapide. En postopératoire Jour 2, elle a de
la fièvre de 39,2 ȃ C (102,5 ȃ F), hypotension,
léthargie et de laboratoire valeurs remarquable pour
l'hypoglycémie et hyperkaliémie. Le plus probable
diagnostic de cette manifestation aiguë est une.
a-Sepsis
b. Hypovolémie
c. L'insuffisance surrénale
d. Nécrose tubulaire aiguë
e. L'acidocétose diabétique
94. Signs and symptoms associated with early sepsis .‫ ا"م ا‬uX - O*‫ا‬Q‫اض ا‬N‫ت وا‬vS‫ ا‬E
include :Q
a. Respiratory acidosis .64T& F .A
b. Decreased cardiac output .gC*‫ج ا‬Q 6 E5& .B
c. Hypoglycemia .‫ ا"م‬, E* .C
d. Increased arteriovenous oxygen .‫ ار"ي‬6‫ ا‬Jl4‫وآ‬N‫ ق ا‬6 ‫ زدة‬.D
difference .‫"ي‬C 6A‫ و‬-, .E
e. Cutaneous vasodilation

Les signes et les symptômes associés avec sepsis


précoce comprennentune.
a. Acidose respiratoire
b. Diminution du débit cardiaque
c. Hypoglycémie
d. Augmentation de la différence
artérioveineuse en oxygène
e. Vasodilatation cutanée

62 A
95. Concerning mammography ,all the following are JFi 6C  ‫ آ‬،6S‫[ ا]"ي ا‬Q# @CSQ J C
true except : :‫"ا‬
A. frequently coupled with ecchography .‫ ا["ى‬:J2P - ‫ن‬Q*  ‫ا‬J]‫ آ‬.A
B. It is the best screening way for breast cancer .‫\ن ا]"ي‬,  ‫ي‬FQC O*\ ‫ ه أ‬.B
C. It is more sensitive in young compared to ‫ء‬4&# O‫ت *ر‬#‫ &" ا‬OJ,4 ]‫ ه أآ‬.C
older women .&, .‫آ‬N‫ا‬
D. It will detect stellar opacity .OJl&‫ ا‬OQS‫ى  ا‬FQ ‫ف‬, ‫ ه‬.D
E. It will detect microcalcifications .‫>ي‬l‫ ا‬aCQ‫ى  ا‬FQ ‫ف‬, ‫ ه‬.E

Concernant la mammographie, toutes ces notions


sont varies, sauf une, laquelle ?
A. Souvent couplée à l’échographie
B. Le meilleur moyen de dépistage du cancer
du sein
C. Plus sensible chez les femmes jeunes que
âgées
D. Détecte les opacités stellaires
E. Détecte les micros calcifications
96. In a suspicion of a medullary compression the best ‫"ي‬J‫آ‬wQ‫ر ا‬JP‫ ”ن ا‬6‫آ‬j 6P :;# .QjK‫ &" ا‬C
confirmatory test is : :‫ ه‬N‫ا‬
A- A CT scan focusing on the lesion .Ot‫ ا‬DC %‫ري آ‬F 6*.\ [ .A
B- A myelography .‫ع‬P&‫ [ ا‬.B
C- A magnetic resonance imaging (MRI) .64J\&; J‫ ر‬.C
D- Bone scan with regular XRay .O‫د‬S‫ ا‬OJ&J4‫ ا‬OSjN# 3?S‫ [ ا‬.D
E- A PET scan .6‫و‬Q‫ز‬.‫"ار ا‬i{# 6S2*‫[ ا‬Q‫ ا‬.E

Devant une suspicion d’une compression


médullaire secondaire, le meilleur examen de
confirmation est :
A. Un scanner centre sur la lésion
B. Une myélographie
C. Une IRM
D. Une scintigraphie osseuse couplée à des
clichés radiologiques simples
E. Un Petscan
97. Among the following prognostic factors,which one ]‫آ‬N‫ ا‬S‫  ه ا‬،OJQ‫ ا‬EJPQ‫ ا ا‬J#  C
is the most important in localised breast cancer : :‫؟‬6S<‫\ن ا]"ي ا‬, 6 OJ‫أه‬
A- Hormonal receptors .OJ>‫ت ا‬v.*Q4‫ ا‬.A
B- Tumor size .‫ ارم‬3l .B
C- Axillary nodes .OJ2#{‫*" ا‬S‫ ا‬.C
D- The ethnicity ‫ق‬S‫ ا‬.D
E- Age .S‫ ا‬.E

Quel est parmi ces propositions le facteur pronostic


le plus important du cancer du sein localisé:
A- Les récepteurs hormonaux
B- La taille de la tumeur
C- Les ganglions axillaires
D- L’ethnicité
E- L’âge

63 A
98. A 70 year old man has on a regular XRay ‫ دك‬O‫د‬S‫ ا‬OJ&J4‫ ا‬OSjN‫>ت ا‬G‫ أ‬، 70  ‫ ر‬D
vertebral narrowing at D12-L2 level.Among the K OJQ‫ص ا‬FT‫ أي  ا‬.D12-L2 ‫ى‬Q4 6 ‫*ري‬
following tests ,which one has no utility in the :‫؟‬O]‫?م ا‬S‫ ا‬Oj‫ ه‬- 6*TQ‫ ا‬EJPQ‫ ا‬6 "JT
differential diagnosis with secondary osteoporosis .‫م ا"م‬J4‫ آ‬.A
A- Calcemia .‫ل‬ST‫ ا‬C J‫و‬# .B
B- C reactive protein .6&25-‫>ي‬G 64J\&; J‫ ر‬.C
C- Dorso-lombar MRI .3?S‫ ا‬O]‫س آ‬J5 .D
D- Osteodensitometry .6C[‫ ا‬J‫و‬.C 6A#>‫ن ا‬v‫ ا‬.E
E- Protein electrophoresis

Un homme de 70 ans présente deux tassements


vertébraux en D12 et L2 sur la radiographie
standard du rachis dorso-lombaire. Parmi ces
examens lequel n’a pas d’intérêt dans le
diagnostic différentiel avec une ostéoporose
secondaire ?
A- Calcémie
B- Protéine C réactive
C- IRM dorso-lombaire
D- Ostéodensitométrie
E- Electrophorèse des protéines sériques
99. Which of the following will retard surely the ‫ ا*[ر‬35T JXw D‫†دي إ‬J, "J‫آ‬wQ# 6Q‫ أي  ا‬D
progression of renal insufficiency : :‫ي؟‬C‫ا‬
A- Any antihypertension medication regardless .)QW  ?&‫ ا‬/;# :;C /X ‫ أي دواء‬.A
of their class .Š4‫ ا‬I‫ ز‬.B
B- Fish oil .)C&‫ "رات وة ه‬.C
C- Loop of Henle diuretics .O.C*&‫ ا‬J4&JlN‫ة ا‬JX ‫ت‬2.] .D
D- Angiotensin converting enzyme inhibition .‫م‬J4‫ات ا‬i .E
E- Calcium antagonists

Quels médicaments ralentissent la progression de


l’insuffisance rénale de façon presque certaine ?.
A- Tous les antihypertenseurs quelle soit leur
classe thérapeutique.
B- Les huiles de poisson.
C- Les diurétiques de l’anse.
D- Les inhibiteurs de l’enzyme de conversion.
E- Les antagonistes calciques.

64 A
100. Which of the following is not seen in functionnal 6TJG‫ي ا‬C‫ ا*[ر ا‬6 "‫ ه‬K 6Q‫ أي  ا‬D
acute renal insufficiency : :‫د؟‬F‫ا‬
A- Fraction excretion of sodium <1 .1 ȃ ‫اح ا[دم‬2‫ء ا‬% .A
B- Urinary density >1020 .1020 ȃ ‫ل‬.‫ ا‬O]‫ آ‬.B
C- Urinary sodium < 10 .10 ȃ ‫ل‬.‫دم ا‬i %J‫ آ‬.C
D- Urea over cratinine ratio < 20 .20 ȃ ‫ز‬v.‫ ا‬J&‫آ‬/O.‫ ا‬O.4 .D
E- Urinary sodium (mosm/kg H2O) >500 DC‫( أ‬mOsm/ Kg H2O) OJ.‫ ا‬OJ,‫و‬N‫ ا‬.E
.500 
Les critères biologiques suivantes sont retrouvées
au cours d’une insuffisance rénale aigue
fonctionnelle sauf :
A- Fraction d’excrétion du sodium < 1.
B- Densité urinaire > 1020.
C- Concentration urinaire en sodium < 10
mmol/l.
D- Ratio urée / créatinine plasmatique < 20.
E- Osmolalité urinaire ( mOsm/ Kg H20) >
500
101. A 55 year old patient is suffering from diabetes for .JQ&, b& ‫ي‬, ‫  داء‬6S ، 55  / B
the last 2 years.He has uncontrolled hypertension.In ‫ا‬b> ‫ح‬Q* ‫  أن‬.:.& JL :;< ‫ع‬T‫") ار‬
this patient ,you would suggest : :/‫ا‬
A- Calcium antagonists .‫م‬J4‫ات آ‬i .A
B- Angiotensin converting enzyme inhibition .O.C*&‫ ا‬J4&JlN‫ة ا‬JX ‫ت‬2.] .B
C- Diuretics .‫ "رات‬.C
D- Vasodilators .OJ‫ت أو‬S, .D
E- Beta blockers .QJ# ‫ت‬. .E

Un homme de 55ans souffre de diabète sucré


depuis deux ans. Il présente une hypertension
artérielle mal contrôlée. Chez ce patient, vous
suggérez :
A- Des inhibiteurs calciques
B- Des inhibiteurs de l’enzyme de conversion
C- Des diurétiques
D- Des vasodilatateurs
E- Des β bloquants

65 A
102. A patient is having recurrent renal colics.His renal ‫ ا["ى‬:J2P .‫ر‬Q ‫ي‬C‫ آ‬E;  6S / B
ecchography and a regular XRay do not reveal any .OC‫ت آ‬J[ ‫  أي‬n K O‫د‬S‫ ا‬OJ&J4‫ ا‬OSjN‫وا‬
renal calculi.He is known to have very poorly )JC 2J4 O&, 15 b& ‫ي‬, ‫وف أن ") داء‬S ‫ه‬
controlled diabetes for the last 15 year.His renal : u ‫ي ه‬C‫ إن ;[) ا‬.‫‰ "ا‬J, #
colics are due to : .OSj OTJTj ‫ت‬J[ .A
A- Radiolucent calculi .6JC P& .B
B- Papillary necrosis .‫د‬F‫ ا‬OJC‫ وا‬OF‫>ب ا‬Q‫ ا‬.C
C- Acute pyelonephritis .‫ر‬Q ‫"اد‬4‫ ا‬.D
D- Recurrent obstruction .gF‫@ ا‬J .E
E- Ureteral stenosis

Un malade présente des coliques néphrétiques à


répétition .Son échographie rénale et sa radio
simple de l’abdomen ne révèlent pas de calculs
urinaires. Il est connu diabétique depuis 15 ans et
très mal contrôlé. Les crises de coliques
néphrétiques sont secondaires à :
A- Des calculs radio transparents
B- De nécroses papillaires
C- Des pyélonéphrites aiguës
D- Des obstructions à répétition
E- Une sténose urétérale
103. Which of the following is pathognominic of :‫؟‬O.J.‫ ا‬Ot 3i‫ وا‬6Q‫ أي  ا‬D
glomerular lesion : .O‫ د‬OCJ# .A
A-Hematuria .OJ.J. O‫ا‬2,‫ أ‬.B
B-Coarse granular casts .‫ء‬J.‫ ات ا‬OCJ# .C
C-Leukocyturia .‫اء‬F‫ ات ا‬O‫ا‬2,‫ أ‬.D
D-red cell casts .O, 24/‫غ‬3.5 ȃJ‫و‬# OCJ# .E
E-proteinuria >3.5 gr/24h

Ce qui est pathognomonique d’une lésion


glomérulaire est :
A- Hématurie.
B- Cylindres granuleux.
C- Leucocyturie.
D- Cylindres hématiques.
E- Protéinurie > 3g/ 24h.
104. An elevated ST segment is found in all the :‫  "ا‬6C  ‫ آ‬6 ST -2* -T D
following except : .gC*‫ ا‬OC ‫ء‬Q‫د  ا‬F‫ر ا‬2‫ ا‬.A
A- Acute phase of myocardial infarction .‫ل‬QJ%&# O‫ز‬vQ .B
B- Prinzmetal syndrome .4N‫ ا‬J2.‫ ا‬6 ‫ أم ا"م‬.C
C- Left ventricular aneurysm .‫ل‬QJl"‫ ا‬J|w .D
D- Digitalis effect .‫د‬F‫ر ا‬Q‫>ب ا‬Q‫ ا‬.E
E- Acute pericarditis

Un sus décalage du segment ST est retrouvé dans


toutes les situations suivantes SAUF:
A- infarctus du myocarde à la phase aigue
B- syndrome de Prinzmetal
C- anévrysme du ventricule gauche
D- effet digitalique
E- péricardite aigue

66 A
105. The two factors that should let us fear a rupture of ‫ق أم ا"م‬%  ‫ف‬P &CSl ‫ا‬S‫  ا‬J&|‫ ه&ك ا‬A
an aortic aneurysm is : :O>#N‫ا‬
A- Abdominal pain + Aneurysm diameter .‫ أم ا"م‬25 + 6&2# 3‫ أ‬.A
B- Increase aneurysm diameter + angor .‫&ق‬X + ‫ أم ا"م‬25 ‫ ازدد‬.B
C- Blue toe +Iliac thrombosis .6T5 ]P + ‫ أزرق‬-.i‫ إ‬.C
D- Abdominal pain + peripheral emboli .62JF ‫ ا[م‬+ 6&2# 3‫ أ‬.D
E- Ureteral compression + recurrent emboli .‫ر‬Q ‫ ا[م‬+ 6. :;< .E

Les deux facteurs qui doivent faire craindre une


rupture d’anévrysme de l’aorte abdominale sont :
A- Douleur abdominale + diamètre de
l’anévrysme
B- Augmentation du diamètre de l’anévrysme
+ angor
C- Orteil bleu + thrombose iliaque
D- Douleur abdominale + embolie périphérique
E- Compression urétérale + embolies à
répétition
106. Among the following etiologies ,which one is not :‫>؟‬#N‫‘ ا‬C4 - @‫ا‬Q K OJQ‫ت ا‬..4‫ أي  ا‬D
associated with aortic dissection : .>#N‫@ ا‬J .A
A- Aortic coarctation .‫ داء رن‬.B
B- Marfan Disease .F‫ ا‬.C
C- Pregnancy .‫"ادي‬4‫ ا‬6.C5 6C ‫ل‬vQ‫ ا‬.D
D- Obstructive cardiomyopathy .J‫ا‬j gC[ .E
E- Atherosclerosis

Parmi les facteurs étiologiques suivants, quel est


celui qui n’est pas cite en association avec la
dissection aortique:
A. Coarctation de l’aorte
B. Maladie de marfan
C. Grossesse
D. Cardiomyopathie obstructive
E. Athérosclérose

107. which of the following lesions are not seen in :‫ب؟‬.‫ ا‬g 6 "‫ ه‬K OJQ‫ت ا‬t‫ أي  ا‬C
acnea: .‫ زؤان‬.A
A- comedon .O\2 .B
B- Papules .OC[ .C
C- Vesicule .‫]ة‬# .D
D- Pustule .‫"ة‬J* .E
E- Nodule

Laquelle de ces lésions n’appartient pas à l’acné


A. Comédon
B. Papule
C. Vésicule
D. Pustule
E. Nodule

67 A
108. The manifestations of secondary syphilis include : :‫ ا]ي‬aJCJT4‫  ?ه داء ا‬E
A- Erythematous macules of the trunk .‫ع‬bl‫ ا‬DC OJ -*# .A
B- Papules at the palms and the feet .J"*‫ ا‬DC‫" و‬J‫ ا‬O‫ را‬DC ‫\ت‬2 .B
C- Fever + arthralgias .iT 3‫ أ‬+ ‫ ارة‬.C
D- Incomplete alopecia .‫ آ‬JL -Ci .D
E- All the answers are true .OFJFi v‫ أ‬O#N‫ آ ا‬.E

Les manifestations de la syphilis secondaire


comprennent :
A. Des macules érythémateuses du tronc
B. Des papules palmo-plantaires
C. De la fièvre et des arthralgies
D. Des plaques d’alopécie incomplète
E. Toutes les réponses sont vraies
109. Which of the following options is false inclassical ‫>ب‬QK O.4&# OFJFi JL OJQ‫رات ا‬JP‫ أي  ا‬D
polyarteritis nodosa and renal lesions : :‫؟‬OC‫ت ا‬t‫ وا‬6J,v‫*" ا‬S‫ ا‬J‫اا‬
A- Renal vasculitis .OC‫ ا‬OJ‫و‬K‫>ب ا‬Q‫ ا‬.A
B- Renovascular hypertension .‫ي‬C‫ ا"م ا‬:;< ‫ع‬T‫ ار‬.B
C- renal infarct .OJC‫ء ا‬Q‫ ا‬.C
D- Rapidly progressive glomerulonephritis .-4‫ ا‬6l‫"ر‬Q‫ ا‬OJC‫ت ا‬.J.‫>ب آ‬Q‫ ا‬.D
E- Microaneurysms .O>l‫ أم ا"م ا‬.E

Dans la périartérite noueuse classique avec atteinte


rénale, choisir la réponse fausse.
A- Vasculite rénale
B- Hypertension rénovasculaire
C- Infarcissement des reins
D- Glomérulonephrite rapidement progressive
E- Microanévrysmes

68 A
110. Concerning snake bites ,which of the following is :‫ ؟‬JFi 6Q‫ أي  ا‬،J#S]‫ت ا‬L"C O.4&# C
true : .‫ل‬T\N‫رة &" ا‬2X 5‫ أ‬6‫ ه‬J#S]‫ت ا‬L" .A
A- Snake bites are less serious in children  "& ‫رة‬2X 5‫ أ‬6‫ ه‬J#S]‫ت ا‬L" .B
B- Snake bites are less dangerous when face .‫ع‬bl‫ا) وا‬
and trunk are involved ‫رات‬j‫ إ‬6‫ ه‬:;‫ض ا‬TP‫ارة وا‬F‫ع ا‬T‫ ار‬.C
C- Fever and hypotension are clinical signs and .‫ة‬J2X ‫ة وا‬J2X O,
factors of poor prognosis OS#Q "& OJ‫ > أه‬aJ OJS<‫رات ا‬j{‫ ا‬.D
D- Local signs are not important in the follow .DTQ4‫ ا‬6 /‫ا‬
up of inpatients .34C 644F T  Ol ‫ات‬.F‫ ا‬.E
E- Petechias are due to allergic reaction to the
venin

Concernant les morsures de serpent, la proposition


vraie est :
A- Les morsures de serpent sont moins graves
chez les enfants
B- Les morsures de serpent sont moins
dangereuses quand elles touchent le visage
et le tronc
C- La fièvre et l’hypotension constituent des
signes cliniques de gravité et des facteurs
de sévérité
D- Les signes locaux n’ont pas d’importance
dans la surveillance du malade à l’hôpital
E- Les pétéchies sont dues à une réaction
allergique au venin du serpent
111. A 25 year old woman consults you for severe ‫ع‬T‫ ار‬g.4# ‫رة‬Q,— ‫ ت‬، 25 ‫ اأة ه‬B
systolic hypertension at 180/95 .Her blood work up # ‫ص ا"م‬F .95/180 6<.*K‫ ا‬:;‫ ا‬6 ""j
included urea, creatinine, electrolytes, calcium, J#CLJ‫ ه‬،‫م‬J4‫ آ‬،‫ اارد‬،J&‫ آ‬،O.‫> ا‬J
hemoglobine & hematocrite were all normal as well gC*‫ ا‬:J2P Šb‫ وآ‬OJSJ.\ >SJ ،I‫آ‬J‫وه‬
as an EKG.In this patient, the next step is to: :6‫ ه‬OJQ‫ة ا‬2P‫ ا‬،O‫ ا‬b‫ &" ه‬.6A#>‫ا‬
A- Rule out pheochromocyoma .3‫د ورم ا*ا‬S.Q,‫ ا‬.A
B- Rule out renal artery stenosis .‫ي‬C‫@ ان ا‬J ‫د‬S.Q,‫ ا‬.B
C- Rule out primary hyperaldosteronism .‫ون ا]ي‬JQ,‫"و‬N‫د ط ا‬S.Q,‫ ا‬.C
D- Assume it is primary hypertension .6A"# :;< ‫ع‬T‫اض أ) ار‬Q‫ ا‬.D
E- To only start her on life style modifications .:* >J : JJ;Q# ‫"ء‬.‫ ا‬.E

Une femme de 25 ans vous consulte pour une


hypertension sévère 180/95.Ses examens incluant
urée,créatinine,Calcium,Hémoglobine,et
hématocrite sont normaux.Son ECG est de même
normal.L’étape suivante chez ce patient est :
A- D’ éliminer un phéochromocytome
B- D’ éliminer une sténose artérielle rénale
C- D’ éliminer un hyperaldosteronisme
primaire
D- D’assumer que c’est une hypertension
primaire
E- De débuter des changements de style de vie

69 A
112. A 22-year-old man is brought by his family to the ‫ارئ‬2‫ ا‬345 D‫) إ‬QCA )‫ أ‬، 22  ‫ ر‬D
Emergency room for severe headache and fever of ‫ و‬،O, 12 b& ‫ع ارة‬T‫"" وار‬j ‫"اع‬i g.4#
12 hours duration for which he took eryhtromycin. 6 ‫ك‬#‫) ا{ر‬JC ‫"ا‬# .J4‫و‬Q‫أ ذŠ &ول إر‬
He started to be confused over the last 2 hours and 6 .‫"ي‬C T\ ‫>ر‬G m‫ و‬JQJ<‫ ا‬JQ4‫ا‬
a rash is noted. On physical examination the rash - @L ‫ن أ‬C# T2‫> ا‬G ‫ي‬4‫ ا‬EFT‫ا‬
appears dark red with petechias all over his body. : 6S /‫ا ا‬b‫ ه‬.)4 ‫ء‬F‫ أ‬A, 6 ‫ات‬.
This patient has: .‫ دة‬O‫ >ز‬OJ O.A‫ ذ‬.A
A- Acute systemic lupus erythematosis .‫ي‬JF‫ اد ا‬DC a4F .B
B- Allergic reaction to the antibiotic .OJTJ D .C
C- Typhoid fever .‫ د‬F, ‫>ب‬Q‫ ا‬.D
D- Acute meningitis .‫"ة‬A‫"ر زا‬P O .E
E- Narcotic overdose

Un homme de 22ans est amené par sa famille aux


urgences pour céphalées intenses et fièvre depuis
12 heures pour laquelle il a pris de
l’Erythtromycine Il commence à être confus depuis
2 heures et un éruption cutanée est notée. A
l’examen physique , l’ éruption apparait foncée
rouge avec des pétéchies recouvrant tout son corps.
Ce patient a :
A- Lupus érythémateux systémique aigu
B- Réaction allergique aux antibiotiques
C- Fièvre thyphoide
D- méningite aigue
E- Overdose de narcotiques
113. A 45 year old patient consults you for confusion. O  ‫رة‬Q,v ‫ ت‬، 45 ‫ ه‬O B
Her work up reveals hyponatremia and her physical  J.‫دم ا"م و‬i E* ‫ص‬FT‫>ت ا‬G‫ أ‬.‫ك‬.‫ار‬
examination reveals euvolemia. Her work up is - @‫ا‬Q >[F .‫ي‬, ‫ دم‬3l ‫ي‬4‫ ا‬EFT‫ا‬
compatible with SIADH.Wich of the following you ‫>ن اد {درار‬C g,&‫ ا‬JL ‫ ا{از‬O‫ز‬vQ
would not expect: :‫؟‬S5Q  3 6Q‫ أي  ا‬.‫ل‬.‫ا‬
A- Hypoosmolarity in the serum .[‫ ا‬6 OJ,‫و‬N‫ ا‬E* .A
B- Hyperuricemia .‫ل‬.‫ ا‬/ ‫ ط‬.B
C- Hypomagnesemia .‫م ا"م‬%J&; E* .C
D- Euvolemia .‫ي‬, ‫ دم‬3l .D
E- Normal urinary sodium .6SJ.\ ‫ل‬# ‫دم‬i .E

Un homme de 45 ans vous consulte pour confusion.


Son bilan révèle une hyponatremie et son examen
physique révèle une euvolemie. Son évaluation est
compatible avec un SIADH. Laquelle des réponses
suivante ne serait retouvée:
A- Hypoosmalarité du sérum
B- Hyperuricemie
C- Hypomagnesemie
D- Euvolemie
E- Sodium urinaire normal

70 A
114. What is the MOST common long-term sequela of F, ‫>ب‬QK Jj ]‫آ‬N‫ ا‬N‫ ا‬OC\ J#*S‫ ا‬6‫  ه‬C
bacterial meningitis in a child? :‫ل؟‬T\N‫ &" ا‬6|
A. Seizure disorder .OJi ‫ ب‬.A
B. Intellectual disability .O O5‫ إ‬.B
C. Hearing loss .-4‫ *"ان ا‬.C
D. Hemiparesis .6*j ‫ل‬%X .D
E. None of the Above .OFJFi JL v‫ أ‬O#N‫ آ ا‬.E

Quelle est la séquelle à long terme la plus fréquente


de la méningite bactérienne chez un enfant?
A. Trouble épileptique
B. Handicap intellectuel
C. Perte auditive
D. Hémiparésie
E. Toutes les propositions sont fausses
115. A 12-year-old boy has migratory arthritis with red, iT - ‫ ه‬iT ‫>ب‬Q‫  ا‬6S  12  DQ B
warm, and swollen joints and evidence of recent ‫رات‬# O]" O#i‫ إ‬DC J‫ ود‬O‫ر‬Q‫ و‬OW‫ دا‬،‫اء‬
group A streptococcal infection. Arthritis in this  ‫ي‬w# OF‫ ا‬b‫ ه‬6 iT‫>ب ا‬Q‫ ا‬n[Q . AO"*S‫ا‬
condition is characterized by which of the :‫؟‬6Q‫ا‬
following? .‫ دة‬3† JL .A
A. Usually nonpainful .‫ دون هت‬DT .B
B. Heals without deformity .‫ارة‬F‫* ا‬Q4 ‫" أن‬S# >? .C
C. Appears after the fever subsides  ‫ن‬S b‫ ا‬D<‫ &" ا‬:* "‫ ه‬.D
.‫ا‬%Q ‫>ب ر‬Q‫ا‬
D. Seen only in patients with concurrent carditis
.‫ة‬J;[‫ة وا‬J.‫ ا‬iT‫  ا‬.E
E. Involves large and small joints

Un garçon de 12 ans souffre d'arthrite migratrice


avec des articulations rouges, chaudes et enflées
avec preuve d'une récente infection streptococcique
du groupe A. L'arthrite dans cette condition est
caractérisée par:
A. Généralement non douloureuse
B. Guérit sans déformation
C. Apparaît après la fin de la fièvre
D. Observée seulement chez les patients atteints de
cardite concomitante
E. Implique les grandes et petites articulations

71 A
116. 3. Which statement regarding genetic disorders of OJ|‫ ارا‬OJ#v*Q,K‫ت ا‬#‫ا‬2<K‫ن ا‬w# OJQ‫رات ا‬.S‫ أي  ا‬A
metabolism is NOT true? :‫؟‬w2X
A. In severe disorders, the affected infant may be ‫ ا[ب‬T2‫ ا‬،‫ت ا""ة‬#‫ا‬2<K‫ ا‬6 .A
sick at birth .‫دة‬K‫ر &" ا‬Q "5
B. Most genetic metabolic diseases are treatable  OJ|‫ ارا‬OJ#v*Q,K‫اض ا‬K‫ ا‬3?S .B
C. The majority of genetic metabolic disease have .>QlS
autosomal recessive inheritance 3>" OJ|‫ ارا‬OJ#v*Q,K‫اض ا‬N‫ ا‬OJ.CL‫ أ‬.C
.6F&Q 6|‫"ي ورا‬4 6;.i
D. Early diagnosis is crucial to good prognosis for
EJPQ‫  أ ا‬3> .‫ ا‬EJPQ‫ ا‬.D
most disorders
.‫ت‬#‫ا‬2<K‫ ا‬3?S "Jl‫ا‬
Quelle proposition concernant les troubles
génétiques du métabolisme n'est PAS vraie?
A. Dans les formes sévères, le nourrisson atteint
peut être malade à la naissance
B. La plupart de ces maladies sont traitables
C. La majorité des maladies métaboliques
génétiques ont une transmission autosomique
récessive
D. Un diagnostic précoce de la plupart de ces
troubles est crucial pour un bon pronostic
117. 4. The diagnosis of cystic fibrosis is usually :‫؟‬64J‫ ا‬nJCQ‫ ا‬EJP ‫ †آ" دة‬6Q‫ أي  ا‬A
confirmed by the finding which of the following? .‫ق‬S‫ر" ا‬C‫ع آ‬T‫ ار‬.A
A. Elevated sweat chloride .‫ق‬S‫ر" ا‬C‫ض آ‬TP‫ ا‬.B
B. Decreased sweat chloride .[‫ر" ا‬C‫ع آ‬T‫ ار‬.C
C. Elevated serum chloride .[‫ر" ا‬C‫ض آ‬TP‫ ا‬.D
D. Decreased serum chloride .OFJFi JL v‫ أ‬O#N‫ آ ا‬.E
E. None of the Above

Le diagnostic de la fibrose kystique du pancréas est


généralement confirmé par la découverte de?
A. Taux de Chlorure dans la sueur élevé
B. Taux de Chlorure dans la sueur diminué
C. Taux de Chlorure sérique élevé
D. Taux de Chlorure sérique diminué
E. Toutes les propositions sont fausses
118. All of the following are characteristic OJ*CP‫اء ا‬JF‫ ا‬O‫ز‬vQ ‫ة‬%J‫  ا?ه ا‬6‫ ه‬6C  ‫ آ‬A
manifestations of congenital rubella syndrome :‫ "ا‬
EXCEPT: .nN‫ ا‬-5 -‫ ر‬.A
A. Choanal Atresia .3‫ ا‬X‫ ا& دا‬Xw .B
B. Intrauterine growth retardation .‫د‬, .C
C. Cataracts .OJ&# OJ.C5 ‫ب‬J .D
D. Structural cardiac defects .6.[ 64 -, ‫ *"ان‬.E
E. Sensorineural hearing loss

Toutes les propositions suivantes sont


caractéristiques du syndrome de la rubéole
congénitale SAUF:
A. Atrésie des Choanes
B. Retard de croissance intra-utérin
C. Cataracte
D. Malformation cardiaque
E. Perte auditive neurosensorielle

72 A
119. Which statement about rhinovirus is false? :‫؟‬w2X OJTN‫وس ا‬JT O.4&# OJQ‫رات ا‬.S‫ أي  ا‬C
A. Rhinovirus is the most common cause of the Jj ]‫آ‬N‫ ا‬g.4‫ ه ا‬OJTN‫وس ا‬J .A
common cold .‫آم‬%C
B. Antibiotic treatment is not routinely indicated )J‫ ر إ‬K ‫ي‬JF‫د ا‬# OlS‫ ا‬.B
for upper respiratory tract infections .JCS‫ ا‬OJ4T&Q‫ق ا‬2‫ت ا‬#i{ "JC*
C. Over-the-counter remedies are safe and effective O&x 6‫ ه‬OJ.\ OTi‫ دون و‬O‫دو‬w# OlS‫ ا‬.C
for treatment of rhinovirus in infants .‫ل‬T\N‫ &" ا‬OJTN‫وس ا‬J ‫ج‬vS
D. Human rhinovirus infection may be life ‫  أن‬O.‫ ا‬OJTN‫وس ا‬JT# O#i{‫ ا‬.D
)" / "& ‫ة‬JFC ‫ن >"دة‬
threatening in an immunocompromised person
.OJ& ‫ت‬#‫ا‬2<‫ا‬
E. Rhinovirus are the major infectious trigger for
OJ4JA‫ت ا‬#i{‫ ه  ا‬OJTN‫وس ا‬J .E
asthma in school-aged children .‫ل ا"ارس‬T\‫ &" أ‬#C ‫ة‬%TF‫ا‬

Concernant le Rhinovirus, quelle proposition est


fausse?
A. Il est la cause la plus fréquente du rhume chez
l’enfant
B. Un traitement antibiotique n'est pas
systématiquement indiqué pour les infections des
voies respiratoires supérieures
C. Les médicaments en vente libre sont sûrs et
efficaces pour le traitement du rhinovirus chez les
nourrissons
D. L'infection par le rhinovirus humain peut être
mortelle chez une personne immunodéprimée
E. Les rhinovirus sont le principal déclencheur
infectieux de l'asthme chez les enfants d'âge
scolaire
120. An 18 month old boy manifests failure to thrive, ‫خ‬TQ‫ ا‬،OJ>j nS< ،&‫ ا‬6 v ‫"ي‬. ‫>ا‬j 18  T\ B
poor appetite, abdominal distention, diarrhea, and  9 >‫ ا‬O; ‫"ا‬J )S<‫ آن و‬.‫ح‬J‫ و"م ار‬،‫>ل‬,‫ إ‬،2#
irritability. He had been well until 9 mo of age. D&F& .O‫د‬S‫ ا‬OS\ ‫ ا]"ي‬gJC  32‫" ذŠ أ‬S# .S‫ا‬
Thereafter, he was weaned from breast milk to :‫ ه‬FJ ]‫آ‬N‫ ا‬EJPQ‫ ا‬. 24Q 
regular foods. His growth curve is flattening. The .‫رد‬Jl‫ داء ا‬.A
most likely diagnosis is: .6&2# ‫ داء‬.B
A. Giardiasis .64J‫ آ‬nJC .C
B. Celiac disease .‫&س‬# ‫>ب‬Q‫ ا‬.D
C. Cystic fibrosis .‫آ"ري‬QJ 6C ‫ل‬vQ‫ ا‬.E
D. Pancreatitis
E. Mitochondrial myopathy

Un garçon de 18 mois présente un retard de


croissance, un manque d'appétit, une distension
abdominale, une diarrhée et une irritabilité. Il avait
été bien jusqu'à l’âge de 9 mois. Par la suite, il a été
sevré du lait maternel et mange des aliments
diversifiés. Sa courbe de croissance s’infléchit vers
le bas. Le diagnostic le plus probable est:
A. Giardiase
B. Maladie coeliaque
C. Fibrose kystique du pancréas
D. Pancréatite
E. Myopathie mitochondriale

73 A
121. The initial treatment of choice for a symptomatic - O#F[ ‫ ") أاض‬/ ‫ج‬v 6 ‫ول‬N‫ر ا‬JP‫ ا‬C
patient with isolated pulmonic stenosis is: :‫ول ه‬%S ‫ي‬A‫@ رر‬J
A. Closed surgical blade valvotomy .@C;‫ ا‬6‫ا‬l‫ ا[م ا‬-# .A
B. Open surgical valvotomy .‫ح‬QT‫ ا‬6‫ا‬l‫ ا[م ا‬-# .B
C. Balloon catheter valvuloplasty .‫ة‬245 ‫ن‬.# ‫ راب ا[م‬.C
D. Blalock-Taussig shunt .s&J,-‫ك‬v# OCF .D
E. Valve replacement .‫م‬i ‫"ال‬.Q,‫ ا‬.E

Le traitement initial de choix pour un patient


symptomatique avec une sténose pulmonaire isolée
est:
A. Valvotomie chirurgicale fermée par lame
B. Valvotomie chirurgicale ouverte
C. Valvuloplastie par cathéter à ballonnet
D. Shunt de Blalock-Taussig
E. Remplacement de la valve pulmonaire
122. Which of the following best describes atopic :‫ل؟‬T\N‫ &" ا‬6.wQ‫" ا‬Cl‫>ب ا‬Q‫ ا‬n[ 6Q‫ أي  ا‬D
dermatitis in children? .J‫را‬b‫د  ا) وا‬SQ#v J ‫ ه‬.A
A. It tends to spare the face and arms .OJ%*‫>ب ا‬Q‫ ا‬- ‫ا@ دة‬Q .B
B. It is frequently associated with uveitis . S‫ا‬  D‫و‬N‫ا‬ JQ&4‫"أ أ|&ء ا‬.  ‫ درا‬.C
C. It rarely begins during the first 2 years of life .‫ز‬%F‫ و‬OF# n[Q .D
D. It is characterized by pruritus and lichenification OJC[‫ ا‬IgA ‫ت‬Q4 ‫ع‬T‫ ار‬- @‫ا‬Q .E
E. It is associated with elevated serum levels of IgA .IgE ‫ت‬Q4 6 ‫ض‬TP‫ وا‬IgM‫و‬
and IgM and decreased levels of IgE

Lequel des propositions suivantes décrit le mieux la


dermatite atopique chez les enfants?
A. Elle tend à épargner le visage et les bras
B. Elle est fréquemment associée à l'uvéite
C. Elle débute rarement pendant les 2 premières
années de vie
D. Elle est caractérisée par le prurit et une
lichénification
E. Elle est associée à des taux sériques élevés d'IgA
et d'IgM et à une diminution des taux d'IgE

74 A
123. A 3-month-old infant has had upper respiratory OS# b& OC OJ4T& ‫  أاض‬6S >j‫ أ‬3  T\ A
symptoms for a few days and presents to the ،OJ4T& O*A< O 6 ‫ارئ وه‬2‫ ا‬345 D‫أم وأ إ‬
emergency department with respiratory distress,  .‫ت‬.J[*‫>ب ا‬Q# >[JP 3 "* .Ol4‫ أآ‬E*‫ و‬،%‫أز‬
wheezing, and hypoxia. You diagnose :‫؟‬Jj ]‫آ‬N‫ ا‬g.4‫ه ا‬
bronchiolitis. What is the most common cause? .‫وي‬vP‫ ا‬64T&Q‫وس ا‬J .A
A. Respiratory syncytial virus .‫ا‬%CT{‫ة ا‬J? ‫وس‬J .B
B. Parainfluenza virus .OC\S‫ ا‬vP‫وس ا‬J .C
C. Cytomegalovirus .‫ا‬%CT{‫وس ا‬J .D
D. Influenza virus .OA‫>ب ا‬QK 6Q‫وس ا‬J .E
E. Human metapneumovirus

Un nourrisson de 3 mois présente depuis quelques


jours des symptômes respiratoires supérieurs et
arrive aux urgences avec une détresse respiratoire,
une respiration sifflante et une hypoxie. Vous
diagnostiquez une bronchiolite. Quelle est la cause
la plus fréquente?
A. Virus respiratoire syncytial
B. Virus parainfluenza
C. Cytomégalovirus
D. Virus de la grippe
E. Métapneumovirus humain
124. In iron deficiency anemia (IDA), which of the :‫ ؟‬JFi 6Q‫ أي  ا‬،""F‫ز ا‬S# ‫ * ا"م‬6 C
following is true? ‫ل‬T\N‫* &" ا‬K ""F‫زن ا‬P "T&Q4 .A
A. Iron stores are depleted later in low-birth weight .‫دة‬K‫ &" ا‬/TP&‫ذات ازن ا‬
infants 2X  "% ‫ي‬4‫ ا‬.F‫ ا‬:* JXw .B
B. Delayed clamping of the umbilical cord increase .""F‫ز ا‬S# ‫"وث * ا"م‬
the risk of IDA ]‫* )أآ‬.‫ ا‬gJC  ‫ة‬J.‫ آ‬OJ‫ك آ‬v>Q,‫ ا‬.C
C. Consumption of large amount of cow’s milk .‫م( ه  >"د‬J‫ ا‬/  750 
(>750ml/day) is a risk factor "& ""F# OlS‫  ا‬n5Q‫ ا‬gl .D
.6SJ.2‫*"ار ا‬# J#CLJ>‫ ا‬.[
D. Iron treatment should stop when hemoglobin is
normalized

Dans l'anémie ferriprive, laquelle des propositions


suivantes est vraie?
A. Les réserves en fer sont épuisées plus tard chez
les nourrissons de faible poids de naissance
B. Le clampage retardé du cordon ombilical
augmente le risque d'anémie ferriprive
C. La consommation de grandes quantités de lait de
vache (> 750ml / jour) est un facteur de risque
D. Le traitement par le fer doit être arrêté lorsque le
taux d'hémoglobine est revenu à la normale

75 A
125. You see a baby of 15 months for a seizure. He had aJ .OJi ‫ت‬#&# ‫>ا [ب‬j 15  T\ S ‫ أن‬D
no personal or family history of seizure. The )w# )"‫ *" أدت وا‬.OJi OJCA ‫ أو‬OJ[Pj ‫ة‬J, )"
mother tells you he was febrile since the morning. ) ‫ "ث‬.‫ة‬wl O#&‫ "ث ا‬.‫ح‬.[‫ ا‬b& ‫ع ارة‬T‫ر‬# gJi‫أ‬
The crisis came suddenly. He first extended all 4 )‫"أت أ\ا‬# 3| ‫ و‬6‫ |ا‬-. OS#‫ر‬N‫\ا) ا‬w# ‫ "د‬K‫أو‬
members for a few seconds and then presented ‫ي‬4‫ ا‬EFT‫  ا‬J. .@A5‫ د‬5 D‫ إ‬4 ‫ة‬QT ‫از‬%Q‫ه‬K#
.‫ د‬D& D2,‫>ب أذن و‬Q‫ ود ا‬ITQ‫ واآ‬.‫م‬°39.5 ‫ارة‬
members shaking for 4 to 5 minutes. Clinical
.JSJ.\ ‫ آن‬6.[S‫ ا‬EFT‫ا‬
examination found a temperature to 39.5 ° C. You :‫؟‬OF‫ ا‬b> EJPQ‫ ه ا‬
find a right acute otitis media. The neurological .F, ‫>ب‬Q‫ ا‬.A
examination was normal. .‫ع‬i .B
What diagnosis do you mentioned? .qJ.X 6T\ ‫ع ارة‬T‫ ار‬.C
A. Meningitis. .O‫ت ور‬vQX‫ ا‬.D
B. Epilepsy .6AF, 6L‫>ب د‬Q‫ ا‬.E
C. Malignant hyperthermia infant
D. Febrile convulsion
E. Meningo-encephalitis

Vous examinez un bébé de 15 mois pour une


convulsion. Il n'a pas d'antécédents personnels ou
familiaux de convulsion. La mère vous dit qu'il
était fébrile depuis le matin. La crise est arrivée
soudainement. Il a présenté d'abord une extension
des 4 membres pendant quelques secondes et a
ensuite a présenté des mouvements cloniques
pendant 4 à 5 minutes. L'examen clinique retrouve
une température à 39,5 ° C. Vous diagnostiquez
une otite moyenne aiguë droite. L'examen
neurologique était normal.
Quel est votre diagnostic?
A. Méningite
B. Epilepsie
C. Hyperthermie maligne infantile
D. Convulsion fébrile
E. Meningo-encéphalite
126. Acute Bacterial Pharyngitis, the most :6‫ ه‬Oi‫رات اا‬j{‫ ا‬،‫د‬F‫ ا‬6|l‫م ا‬SC.‫>ب ا‬Q‫ ا‬D
Pathognomonic signs are: .-C.‫ &" ا‬3‫ أ‬،‫ل‬S, ،OF# .A
A. Hoarsness, cough, pain on swallowing .‫>ل‬,‫ء وإ‬J5‫ إ‬،‫"اد‬4‫ ا‬- j‫ ر‬.B
B. Rhinorrhea with obstruction, vomiting and .‫ل‬S, ،6C 3‫ أ‬،‫ة‬S5 .C
diarrhea DC ‫ات‬. + ‫ة‬Q& 6 - J‫ ز‬3P .D
C. Chills, muscle pain, cough .Š&F‫اع ا‬j
D. Tonsillar enlargement with diffuse erythema + .OFJFi JL v‫ أ‬O#N‫ آ ا‬.E
petichae on soft palate
E. None of the Above

Dans la Pharyngite bactérienne aiguë, les signes les


plus pathognomoniques sont:
A. Enrouement, toux et douleur à la déglutition
B. Rhinorrhée avec obstruction, vomissement et
diarrhée
C. Frissons, douleurs musculaires et toux
D. Élargissement des amygdales avec érythème
diffus et pétéchies au niveau du palais mou
E. Toutes les propositions sont fausses

76 A
127. A 4-year-old boy squeals and cries but uses no )‫ إن >را‬.3CQ ‫ دون أن‬gFQ&‫ و‬6. ‫&ات‬, 4  6.i B
words. His gross and fine motor skills are well K‫ء و‬JjN# ;& ‫ ه‬."J # ‫رة‬2Q OJ4l‫ وا‬OJ‫آ‬F‫ا‬
developed. He is preoccupied with objects and does ‫"و‬. ‫ات و‬i 6SJ.\ # gJlQ4 .‫ل‬T\N‫ ا‬- gSC
not play with other children. The child responds :‫ ه‬FJ ]‫آ‬N‫ ا‬EJPQ‫ ا‬.i‫ ودون ا‬3Q> JL
normally to sounds but appears disinterested and .%J‫آ‬Q‫ ا‬E*‫ و‬O‫آ‬F‫اب ط ا‬2<‫ ا‬.A
detached. The most likely diagnosis is: .‫اب "ي‬2<‫ ا‬.B
A. Attention-deficit/hyperactivity disorder .-, ‫ *"ان‬.C
B. Autistic disorder .6.[ 64& ‫اب‬2<‫ ا‬.D
C. Hearing loss .6C* nCP .E
D. Neurodegenerative disorder
E. Mental retardation

Un garçon de 4 ans crie et pleure mais n'utilise


aucun mot. Sa motricité globale et fine est bien
développée. Il est préoccupé par les objets et ne
joue pas avec les autres enfants. L'enfant répond
normalement aux sons mais semble désintéressé et
détaché. Le diagnostic le plus probable est:
A. Trouble déficitaire de l'attention / hyperactivité
B. Trouble autistique
C. Perte auditive
D. Trouble neurodégénératif
E. Retard mental
128. A 2-year-old girl, developed noisy breathing on ‫ظ‬FC -‫ ا‬،@J>‫ &" ا‬gXi >4T& ،‫ن‬Q&, ‫ة ه‬Q D
inspiration, marked retractions of the chest wall, 6S I‫ آ‬.6. ‫ل‬S,‫ و‬،nN‫ ا‬6 @ ،‫ "ار ا["ر‬6
flaring of the nostrils, and a barking cough. He has ]‫آ‬N‫ ا‬EJPQ‫ إن ا‬.J b& OC OJ4T& ‫ن \ق‬Q‫ ا‬
had a mild upper respiratory infection for 2 days. :‫ ه‬FJ
The most likely diagnosis is .#‫ ر‬.A
A. Asthma .‫ر‬%‫ن ا‬4 ‫>ب‬Q‫ ا‬.B
B. Epiglottitis .‫ت‬.J[*‫>ب ا‬Q‫ ا‬.C
C. Bronchiolitis .6,‫و‬J ‫&ق‬X .D
D. Viral croup .OFJFi JL v‫ أ‬O#N‫ آ ا‬.E
E. None of the above

Une fille de 2 ans développe une respiration


bruyante en inspiration, rétractions marquées de la
paroi thoracique, battements des ailes du nez et une
toux aboyante. Il a eu une légère infection des voies
respiratoires supérieures pendant 2 jours. Le
diagnostic le plus probable est:
A. Asthme
B. Épiglottite
C. Bronchiolite
D. Croup viral
E. Toutes les propositions sont fausses

77 A
129. All of the following statements concerning the "& ‰T‫ ات ا‬O‫ز‬vQ O|‫د‬F O.4&# OJQ‫رات ا‬.S‫ آ ا‬D
incidence of the sudden infant death syndrome :‫  "ا‬،OFJFi ‫ل‬T\N‫ا‬
(SIDS) are correct, except:  S‫ ا‬6 O#i‫آ] إ‬N‫ ا‬6‫ ه‬O|‫د‬F‫ ا‬b‫ ه‬.A
A. The incidence of SIDS is the highest at the age .>j‫ أ‬5-4 J#
of 4-5 months ‫ت ذوي‬vAS‫ &" ا‬Jj ]‫ أآ‬6‫ ه‬.B
B. It is more frequent in families with poor social .‫ة‬J*T‫ ا‬OJJS‫ت ا‬KF‫ا‬
conditions ‫*ء‬jN‫ ا‬J# |‫آ] "و‬N‫ ا‬6‫ ه‬O|‫د‬F‫ ا‬b‫ ه‬.C
.‫ا اض‬b‫ ه‬F<
C. The incidence is higher among siblings of SIDS
.‫&ت‬.‫آ] &" ا‬N‫ ا‬6‫ ه‬O|‫د‬F‫ ا‬b‫ ه‬.D
victims
D. The incidence is higher among girls

Toutes les propositions suivantes concernant


l'incidence du syndrome de mort subite du
nourrisson (MSN) sont correctes, sauf:
A. L'incidence de MSN est la plus élevée à l'âge de
4-5 mois
B. Elle est plus fréquente dans les familles avec
mauvaises conditions sociales
C. L'incidence est plus élevée chez les frères et
soeurs des victimes de MSN
D. L'incidence est plus élevée chez les filles
130. Which of the following is the most frequent ‫ "اء‬Jj ]‫آ‬N‫ ا‬OJ&JS‫  ا?ه ا‬6‫ ه‬6C  ‫ أي‬C
ophthalmologic manifestation of congenital :‫؟‬6*CP‫ت ا‬,*‫ا‬
toxoplasmosis? gl‫ي دة ن أدي ا‬b‫ وا‬،‫د‬4‫ ا‬.A
A. Cataracts, which are usually unilateral and .‫دة‬K‫و?> &" ا‬
present at birth gl‫ ا‬6A&| ‫ي دة ن‬b‫ وا‬،‫د‬4‫ ا‬.B
B. Cataracts, which are usually bilateral and present .‫دة‬K‫و?> &" ا‬
at birth "S# ‫ ?> دة‬6Q‫ وا‬OJ*‫ ا‬6 ‫ هت‬.C
‫ل‬T\N‫ة "ا &" ا‬XwQ >?‫دة و‬K‫ا‬
C. Retinal abnormalities which often appear after
.J;.‫وا‬
birth and flare up later in childhood and
6A&| ‫ي إ أن ن أدي أو‬b‫ وا‬،‫د‬4‫ ا‬.D
adolescence D‫و‬N‫ ا‬S‫ ا‬J&4‫ ا‬6 >?‫ و‬gl‫ا‬
D. Cataracts, which are either unilateral or bilateral .S‫ ا‬
and appear late in the first decade of life

Laquelle des propositions suivantes est la


manifestation ophtalmologique la plus fréquente de
la toxoplasmose congénitale?
A. Cataractes, généralement unilatérales et
présentes à la naissance
B. Cataractes, généralement bilatérales et présentes
à la naissance
C. Anomalies rétiniennes qui apparaissent souvent
après la naissance et qui se manifestent plus tard
dans l'enfance et l'adolescence
D. Cataractes unilatérales ou bilatérales
apparaissant tard dans la première décennie de la
vie

78 A
131. In hemophilia A, which one of the following JL ‫ ه‬OJQ‫ ا‬O.P‫ص ا‬FT‫ أي  ا‬،A JCJJ>‫ ا‬6 D
laboratory exam is abnormal? :‫؟‬6SJ.\
A. Bleeding time .‫ف‬%&‫ ز ا‬.A
B. Platelet count .‫ت‬FJT[‫"اد ا‬S .B
C. PT (Prothrombin Time) .J.‫وو‬.‫ ز ا‬.C
D. PTT (Partial Thromboplastin Time) .6A%l‫ ا‬JQ,v#.‫و‬Q‫ ز ا‬.D
E. Fibrinogen .JTJC‫ " ا‬.E

Dans l'hémophilie A, lequel des examens de


laboratoire suivants est anormal?
A. Temps de saignement
B. Nombre de plaquettes
C. PT (temps de prothrombine)
D. PTT (temps de thromboplastine partielle)
E. Fibrinogène
132. The most common congenital heart defect is: :‫ ه‬Jj ]‫آ‬N‫ ا‬6*CP‫ ا‬6.C*‫ ا‬gJS‫ ا‬B
A. Valvar aortic stenosis .‫>ي‬#N‫@ ا[م ا‬J .A
B. Ventricular septal defect .6&J2. %F‫ ا‬6 gJ .B
C. Coarctation of the aorta .>#N‫@ ا‬J .C
D. Tetralogy of Fallot .‫ ت‬OJ#‫ ر‬.D
E. Hypoplastic left heart syndrome .u4&Q‫ ا‬O[5 OJ.C5 O‫ز‬vQ .E

La cardiopathie congénitale la plus fréquente est:


A. La Sténose valvulaire aortique
B. La Communication Inter ventriculaire
C. La Coarctation de l'aorte
D. La Tétralogie de Fallot
E. L’Hypoplasie du coeur gauche
133. Concerning Hereditary Spherocytosis, choose the O#{‫ ا‬6‫  ه‬،OJ|‫ ارا‬O‫ او‬F‫ ]ة ات ا‬O.4&# B
right answer: :‫؟‬OFJF[‫ا‬
A. Bilirubin gallstones only occur in adults :* O‫او‬T[‫ ا‬J#‫و‬JCJ.‫ت ا‬J[ ‫"ث‬F .A
B. The severity of the disease is variable .‫ر‬.‫&" ا‬
C. Folic acid is not recommended in severe .O‫و‬TQ ‫ا اض‬b‫رة ه‬2X .B
Hereditary Spherocytosis O 6 ‫ ارق‬/ ‫ء‬2”# -[& K .C
D. Splenectomy is recommended for all types of .OJ|‫ ارا‬O‫ او‬F‫آ]ة ات ا‬
the disease ‫ا‬b‫ل  أاع ه‬F2‫[ل ا‬WQ,# [& .D
.‫اض‬
Concernant la sphérocytose héréditaire, choisissez
la bonne réponse:
A. Les lithiases biliaires de bilirubine se produisent
seulement chez les adultes
B. La gravité de la maladie est variable
C. L'acide folique n'est pas recommandé en cas de
sphérocytose héréditaire grave
D. La splénectomie est recommandée pour tous les
types de la maladie

79 A
134. Diagnostic criteria established for the diagnosis of ]‫آ‬N ‫ ارة "وم‬6‫ ه‬6‫آ‬,‫ ض آوا‬OJ[JPQ‫ ا‬JS‫ ا‬C
Kawasaki Disease are fever lasting longer than 5 :‫ "ا‬OJ4JA‫ ا‬O4‫  ا?ه ا‬6Q‫ أم وا‬5 
days and the following main clinical features @&S‫ ا‬6 J*Q JL ‫ د‬OJT "* 3P .A
except: .3, ١.5  .‫ أآ‬OJT ‫ *"ة‬25 -
A. Acute nonpurulent cervical lymphadenopathy .O‫ن او‬4 .B
with lymph node diameter greater than 1.5 cm . J*Q OFQC ‫>ب‬Q‫ ا‬.C
B. Strawberry Tongue .‫ل‬jN‫"د ا‬SQ T\ .D
C. Purulent Conjonctivitis .OJ2JF‫\اف ا‬N‫ ا‬6 ‫ات‬J; .E
D. Polymorphous Rash
E. Changes in the peripheral extremities

Les critères de diagnostic de la maladie de


Kawasaki sont la fièvre qui dure depuis plus de 5
jours et les principales caractéristiques cliniques
suivantes, sauf:
A. Lymphadénopathie cervicale non-purulente
aiguë avec un diamètre de ganglion supérieur à 1,5
cm
B. Langue framboisée
C. Conjonctivite purulente
D. Rash polymorphe
E. Changements périphériques des extrémités
135. In autosomal dominant inheritance the trait will be OJ|‫ ارا‬O4‫ ا‬DC ‫]ر‬S‫  ا‬،‫"ة‬A4‫ ا‬OJ4l‫ ا‬O|‫ ارا‬6 C
found in one parent and: :‫ أ" اا" و‬6
A. 25% of daughters and 75% of sons .‫&ء‬#N‫  ا‬%75‫&ت و‬.‫  ا‬%25 .A
B. 25% of sons and 75% of daughters .‫&ت‬.‫  ا‬%75‫&ء و‬#N‫  ا‬%25 .B
C. 50% of daughters and 50% of sons .‫&ء‬#N‫  ا‬%50‫&ت و‬.‫  ا‬%50 .C
D. Only in daughters .:* ‫&ت‬.‫ &" ا‬.D

Dans l'hérédité autosomique dominante, le trait sera


présent chez un parent et:
A. 25% des filles et 75% des fils
B. 25% des fils et 75% des filles
C. 50% des filles et 50% des fils
D. Seulement chez les filles

80 A
136. A newborn infant presents with cyanosis and mild aT ‫ع‬4‫ق و‬5‫  ازر‬6S ‫دة وه‬K‫ ا‬q" T\ ‫ أ‬B
tachypnea at about 6 hours of life. The infant is 6 -J<‫ ا‬-<‫ و‬.‫دة‬K‫ت  ا‬, 6 6‫" ا‬S# nJTX
placed in 100% oxyhood (100% oxygen) and ‫ ه‬6Q‫ أي  ا‬.‫ع‬.j{‫ ا‬-J.2‫ و‬%100 Jl4‫ أوآ‬OS&*
saturations normalize. Which of the following is :‫؟‬T2‫ا ا‬b> FJ ]‫آ‬N‫ ا‬EJPQ‫ا‬
the most likely diagnosis in this infant? .655‫ ازر‬6*CX gC5 ‫ ض‬.A
A. Cyanotic Congenital Heart Disease .‫ي‬A‫ ض ر‬.B
B. Lung Disease .‫ي‬%‫ اآ‬6.[S‫>ز ا‬l‫ ا‬6 ‫ ض‬.C
C. Central Nervous System Disease .‫واح ا["ر‬Q,‫ ا‬.D
D. Pneumothorax .‫ ا"م‬J#CLJ>Q .E
E. Methemoglobinemia

Un nouveau-né présente une cyanose et une légère


tachypnée à environ 6 heures de vie. Le nourrisson
est placé dans l'oxyhood à 100% (100%
d’Oxygène) et les saturations se normalisent.
Lequel des propositions suivantes est le diagnostic
le plus probable?
A. Cardiopathie congénitale cyanogène
B. Pathologie pulmonaire
C. Pathologie du système nerveux central
D. Pneumothorax
E. Méthémoglobinémie
137. The most important agents causing pharyngitis are: :6‫م ه‬SC.‫>ب ا‬QK O..4‫ ا‬OJ‫آ] أه‬N‫ا ا‬S‫ ا‬A
A. Viruses. .‫ت‬,‫و‬JT‫ ا‬.A
B. Mycoplasma pneumoniae. .OA‫رات ا‬2T‫ ا‬.B
C. Haemophilus influenzae. .OJ%&‫ ا‬OJ"Q4‫ ا‬.C
D. Streptococcus pneumoniae. .OA‫ ا‬O"*S‫ ارات ا‬.D
E. Group C streptococcus. .C OW O"*S‫ ارات ا‬.E

Les agents les plus importants causant la pharyngite


sont:
A. Virus.
B. Mycoplasma pneumoniae
C. Haemophilus influenzae
D. Streptococcus pneumoniae
E. Streptocoque du groupe C
138. A young girl is found to have bilateral iridocyclitis ‫>ب‬Q# O#[ >‫ أ‬J. ،‫ة‬J;i ‫ة‬QT EFT‫ &" إاء ا‬B
on examination. On further questioning, it is found ‫ج‬S I‫ أ> آ‬J. ،>#‫ا‬lQ,‫ &" ا‬.6#">‫ ا‬34l‫ وا‬OJ%*‫ا‬
that she has been limping for the last several weeks. :‫ ه‬FJ ]‫آ‬N‫ ا‬EJPQ‫ إن ا‬.OJ< -J#,‫ "ة أ‬b&
The most likely diagnosis is: .6‫آ‬,‫ داء آوا‬.A
A. Kawasaki disease .6#.j ‫ي‬J|‫ ر‬iT ‫>ب‬Q‫ ا‬.B
B. Juvenile rheumatoid arthritis .:J4#  .C
C. Herpes simplex .‫ت‬,*‫ داء ا‬.D
D. Toxoplasmosis

Une jeune fille présente une iridocyclite bilatérale à


l'examen clinique. En réponse à d'autres questions,
on constate qu'elle boite depuis plusieurs semaines.
Le diagnostic le plus probable est:
A. La maladie de Kawasaki
B. Polyarthrite rhumatoïde juvénile
C. Herpès simplex
D. Toxoplasmose

81 A
139. An 8 yr. old female child following Upper OJ4T&Q‫ق ا‬2‫ن ا‬Q”# >Q#i‫" إ‬S# ،‫&ات‬, 8 ‫ ه‬D]‫ أ‬C
Respiratory Tract Iinfection developed DC Q&‫Š و‬T‫ ا‬DC 6\2 6S*# T\ >" w OCS‫ا‬
maculopapular rash on the jaw spreading onto the "* 3P‫ و‬n, ‫"ون‬# q]‫م ا‬J‫ ا‬6 Djv ‫ي‬b‫ع وا‬bl‫ا‬
trunk which cleared on the 3rd day without :‫ ه‬EJPQ‫ ا‬.‫ذن‬N‫ ا‬nCX OJT
desquamation and tender post auricular and .6‫آ‬,‫ داء آوا‬.A
lymphadenopathy. The diagnosis is .OJ O#i‫ إ‬.B
A. Kawasaki disease .‫اء‬JF‫ ا‬.C
B. Erythema infectiosum .O.[F‫ ا‬.D
C. Rubella .OFJFi JL v‫ أ‬O#N‫ آ ا‬.E
D. Measles
E. None of the Above

Une fille de 8 ans développe après une infection


des voies aériennes supérieures, une éruption
maculopapuleuse sur la mâchoire, puis s’est
poursuivie sur le tronc et s'est éclaircie le troisième
jour sans desquamation et présence d’une
lymphadénopathie rétro-auriculaire. Le diagnostic
est:
A. Maladie de Kawasaki
B. Érythème infectieux
C. Rubéole
D. Rougeole
E. Toutes les propositions sont fausses
140. A1-year-old is noted to have a microcytic anemia. J;i ‫* دم‬T# ‫ أ) [ب‬m ،‫ وا"ة‬O&,  T\ B
A follow-up hemoglobin electrophoresis ‫ أن ه&ك‬J#CLJ>C 6A#>‫ن ا‬v‫  ا‬J. .‫ات‬
demonstrates an increased concentration of ‫ر‬N‫ ا‬DC ‫ ه‬T2‫ا ا‬b‫ ه‬.A2 J#CLJ>‫ ا‬%J‫ آ‬6 ‫زدة‬
hemoglobin A2. The child is most likely to have? :‫[ب ب‬
A. Iron deficiency .""F‫ ز ا‬.A
B. β-thalassemia trait .QJ#-JJ,v .B
C. Sickle cell anemia .6Cl&‫ * ا"م ا‬.C
D. Chronic systemic illness .% ‫ ض >ز‬.D
E. Lead poisoning .‫ص‬i# 34Q‫ ا‬.E

On note une anémie microcytaire chez un enfant de


1 an. Une électrophorèse de l'hémoglobine de
contrôle démontre une augmentation de la
concentration de l'hémoglobine A2. Le diagnostic
le plus probable est:
A. Carence en fer
B. Trait de β-thalassémie
C. Anémie falciforme
D. Maladie systémique chronique
E. Intoxication au plomb

82 A
141. A 7-year-old presents to the ER after three days of ‫ أم‬O|v| "S# ‫ارئ‬2‫ ا‬345 D‫ أ إ‬،‫&ات‬, 7  T\ A
worsening, unsteady gait. Among the following, the >?‫ ا‬،6Q‫ < ا‬.OWJ, O ‫ ذات‬OQ#| JL OJ 
finding most typical of Guillain-Barré syndrome is: :‫ر) ه‬#-‫ن‬vJL O‫ز‬vQ 62&‫ا‬
A. Absent deep tendon reflexes. .O*JS‫ ا‬O‫ت ا‬4S&‫ب ا‬JL .A
B. Urticaria .‫ى‬j .B
C. Proximal weakness. .O.*‫ت ا‬vS‫ وه ا‬.C
D. Ptosis. .6" .D
E. None of the above .OFJFi JL v‫ أ‬O#N‫ آ ا‬.E

Un enfant de 7 ans se présente aux urgences après


trois jours d'aggravation et d'instabilité de la
marche. Parmi les éléments suivants, la
constatation la plus typique du syndrome de
Guillain-Barré est:
A. Absence de réflexes tendineux profonds
B. Urticaire
C. Faiblesse musculaire proximale
D. Ptosis
E. Toutes les propositions sont fausses
142. A previously healthy 8-year-old boy has a 3-week 3 b& 6S ،‫"ة‬J OF[# -QQ *#, ‫ آن‬،‫&ات‬, 8  DQ D
history of low-grade fever of unknown source, ،‫رة وزن‬4X ،gS ،‫ ا["ر‬O>l OTJTX ‫  ارة‬-J#,‫أ‬
fatigue, weight loss, myalgia, and headaches. On J. ،I5‫ا ا‬b‫ل ه‬vX ‫ص‬FT‫ار ا‬Q# .‫"اع‬i‫ و‬،6C 3‫أ‬
repeated examinations during this time, he is found EJPQ‫ ا‬.OTJTX ‫ل‬F\ OP<‫ و‬،‫ات‬. ،OJ.C5 OPT )" ‫أن‬
to have developed a heart murmur, petechiae, and :‫ ه‬FJ ]‫آ‬N‫ا‬
mild splenomegaly. The most likely diagnosis is: .OJ|‫ ر‬D .A
.6‫آ‬,‫ داء آوا‬.B
A. Rheumatic fever
.I‫ر‬, D .C
B. Kawasaki disease
.6| gC5 ‫;ف‬j ‫>ب‬Q‫ ا‬.D
C. Scarlet fever
.4‫ داء ا‬.E
D. Infective Endocarditis
E. Tuberculosis

Un garçon de 8 ans, auparavant en bonne santé,


présente une fièvre peu élevée de cause inconnue
depuis trois semaines, une fatigue, une perte de
poids, des myalgies et des maux de tête. Pendant
cette période, il a développé un souffle cardiaque,
des pétéchies et une légère splénomégalie. Le
diagnostic le plus probable est:
A. Une Fièvre rhumatismale
B. La maladie de Kawasaki
C. La scarlatine
D. Une Endocardite infectieuse
E. Une Tuberculose

83 A
143. A 85-year-old man is being evaluated for gait 6 O#Si g.4# )JJ* ‫ي‬l  85  ‫ر‬ E
difficulties. On examination, it is found that joint @JS‫ ا‬aF‫*"ام ا‬Q,‫ب ا‬JL J. ،EFT‫ &" ا‬.)QJ
proprioception is absent in his toes. People with  ‫ن‬S b‫ص ا‬PjN‫ ا‬.)J"5 6 6C[T‫ا‬
impaired position sense will usually fall if they
"& <‫ن أر‬S* ‫ف‬, )Q‫س ا‬4‫ إ‬6 nS<
stand with their feet together and do which of the
following ?  ‫ وأي‬g& D‫ إ‬.& 3>J"5 QC# ‫ف‬5‫ ا‬O‫و‬F
A- Flex the neck :‫)؟‬# ‫م‬J*‫وض ا‬T‫ ه ا‬6Q‫ا‬
B- Extend their arms in front of them .O.5‫ ا‬6&| -A
C- Flex the knees .‫م‬N‫ ا‬D‫ إ‬J‫را‬b‫ " ا‬-B
D- Turn the head .JQ.‫ اآ‬6&| -C
E- Close their eyes .‫ "و اأس‬-D
.3>&J‫ق أ‬vL‫ إ‬-E
Un homme de 85 ans est évalué pour des difficultés
de marche. A l'examen, on constate que la
proprioception des articulations est absente dans
ses orteils. Les personnes ayant un mauvais sens de
position tombent généralement si elles se tiennent
debout avec les pieds joints et font laquelle des
propositions suivantes?
A- Plier le cou
B- Étendre les bras devant eux
C- Fléchir les genoux
D- Tourner la tête
E- Fermer les yeux
144. A 22-year-old woman reports a scotoma OQ  6S >w# "JT  22 ‫اأة ه‬ C
progressing across her left visual field over the 30 ‫ "ى‬DC 4N‫[ي ا‬.‫> ا‬l . ‫"رج‬Q
course of 30 minutes, followed by left hemicranial ،‫ن‬J]L ،4N‫ ا‬nF*‫@ ا‬j 6 /# 3‫> أ‬JC ،O*J5‫د‬
throbbing pain, nausea, and photophobia. Her
>" >"‫ه ووا‬X‫ *" أدت أن أ‬.‫ورهب اء‬
brother and mother have similar headaches. Which
of the following is present in this patient’s b‫ ه‬O 6 ‫ ه د‬6Q‫ أي  ا‬.| ‫"اع‬i
condition but not in common migraine ? :‫؟‬6T[&‫ ا["اع ا‬6 SAj aJ‫ و‬O‫ا‬
A. Photophobia .‫ رهب اء‬-A
B. Familial pattern .6CAS‫ ا‬:&‫ ا‬-B
C. Visual aura .O‫[ر‬#‫ أورة إ‬-C
D. Hemicranial pain .nF*‫@ ا‬j 6 3‫ أ‬-D
E. Nausea
.‫ن‬J]L -E
Une femme de 22 ans rapporte un scotome
progressant dans son champ visuel gauche sur une
période de 30 minutes, suivi d'une douleur
lancinante hémicrânienne gauche, de nausées et
d'une photophobie. Son frère et sa mère ont des
maux de tête similaires. Lequel des éléments
suivants est présent chez ce patient mais pas dans la
migraine courante?
A. Photophobie
B. Predisposition familiale
C. Aura visuelle
D. Douleur hémicrânienne
E. Nausées

84 A
145. A 32-year-old woman is being evaluated for ‫"أ‬# "* .‫["اع‬C JJ* ‫  أت‬32 ‫اأة ه‬ A
headaches. They started about 6 months ago and ‫ و"وم‬،‫ع‬.,N# ‫ ات‬-# ‫"ث‬F‫> و‬j‫ أ‬6 b&
occur a few times per week, lasting until she falls ‫ء‬%l‫ ا‬6 %‫ وآ‬I#| 3N‫ ا‬.‫ ا&م‬6 ‫ أن ;ق‬D‫إ‬
asleep. The pain is constant and focused at the front
O5v ) aJ 3N‫ ا‬.‫  اأس‬6TCP‫ وا‬6N‫ا‬
and back of the head. The pain is unrelated to
position and tends to be worse later in the day. 6A< ‫ ه&ك رهب‬.‫م‬J‫أ أ|&ء ا‬, J‫ و‬l{#
There is mild photophobia. Which of the following :‫ر ؟‬N‫ ا‬6‫ ه‬OJQ‫ أي  ا?ه ا‬.nJTX
findings is most likely ? ‫م‬v‫ وإ‬O.5‫ ا‬O‫ق آ‬2 6 JW< -‫ ا‬-A
A. Slightly reduced neck range of motion and .@&SC ‫ور‬l
paracervical tenderness .‫ت‬JC O‫ وذ‬-B
B. Papilledema .6SJ.\ JL ‫"غ‬C 64J\&;‫ ا‬J‫ ا‬-C
C. Abnormal brain MRI
JL ‫"غ‬C ‫ري‬F‫ ا‬6*.2‫[ ا‬Q‫ ا‬-D
D. Abnormal brain CT
E. Abnormal EEG .6SJ.\
.6SJ.\ JL 6A#>‫ ا["ى ا‬:J2P -E
Une femme de 32 ans est évaluée pour des maux de
tête. Ils ont commencé il y a environ 6 mois et se
produisent quelques fois par semaine, jusqu'à ce
qu'elle s'endorme. La douleur est constante et
concentrée à l'avant et à l'arrière de la tête. La
douleur n'est pas liée à la position et tend à être pire
plus tard dans la journée. Il y a une légère
photophobie. Lequel des résultats suivants est le
plus probable?
A. Amplitude du cou légèrement réduite et
sensibilité paracervicale
B. Œdème papillaire
C. IRM cérébrale anormale
D. Tomodensitométrie cérébral anormal
E. EEG anormal
146. A 39-year-old man was treated 1 year ago for a ‫اج‬CX  O&, b& u  39  ‫ر‬ C
brain abscess. He has largely recovered, but still )" ‫ و&)  زال‬،J.‫ آ‬# 6Tj "* .6L‫د‬
has occasional word-finding difficulties. Which of ‫ ه‬6Q‫ أي  ا‬.J‫ت أ‬C‫*ء ا‬Q‫ ا‬6 O#Si
the following is the most common symptom in
‫ن‬#[‫ ا‬D<‫ &" ا‬Jj ]‫آ‬N‫ض ا‬S‫ا‬
patients with brain abscess ?
A- Nausea and vomiting :‫؟‬6L‫اج د‬P#
B- Ataxia .‫ء‬J5‫ن وإ‬J]L -A
C- Headache . ‫ ر‬-B
D- Neck stiffness .‫"اع‬i -C
E- Seizures .O.5‫ ر‬gC[ -D
.OJi ‫ت‬# -E
Un homme de 39 ans a été traité il y a 1 an pour un
abcès cérébral. Il s’est largement rétabli, mais a
toujours des difficultés occasionnelles à trouver ses
mots. Lequel des symptômes suivants est le plus
fréquent chez les patients présentant un abcès
cérébral?
A- Nausées et vomissements
B- Ataxie
C- Maux de tête
D- Rigidité du cou
E- Convulsions

85 A
147. A 37-year-old man presents with visual ‫ت‬T   ‫   وه‬37  ‫ر‬ D
impairement. Examination reveals a bitemporal J*‫ ا‬D ‫ص‬FT‫>ت ا‬G‫ أ‬.O‫ اؤ‬6
hemianopsia. Which of the following tumors is FJ ]‫آ‬N‫ ه ا‬OJQ‫ورام ا‬N‫ أي  ا‬.JJL"[‫ا‬
most likely responsible for this finding ?
:‫ ا?هة؟‬b‫  ه‬K‫†و‬4 ‫أن ن‬
A. Optic glioma
B. Occipital astrocytoma .‫[ي‬# 6*#‫ ورم د‬-A
C. Brainstem hemorrhage .6‫ا‬b5 6l ‫ ورم‬-B
D. Pituitary adenoma .‫ع ا"غ‬b 6 n% -C
E. Sphenoid wing meningioma .OP&‫"ة ا‬L 6 ‫ ورم‬-D
.‫ ا"ي‬3?S‫ &ح ا‬6 6AF, ‫ ورم‬-E
Un homme de 37 ans se présente avec des troubles
visuels. L'examen révèle une hémianopsie
bitemporale. Laquelle des tumeurs suivantes est
probablement responsable de cette observation?
A. Gliome optique
B. Astrocytome occipital
C. Hémorragie du tronc cérébral
D. Adénome hypophysaire
E. Méningiome de l'aile sphénoïde
148. A 72-year-old woman has a head CT performed 6*.\ [ > ‫  أي‬72 ‫اأة ه‬ A
because of headaches. It is significant for a left 6 OCQ ‫>ر‬G ‫ ه&ك‬.‫ ا["اع‬g.4# ‫أس‬C ‫ري‬F
hemisphere mass with an overlying hyperostosis of DC 6‫ ه‬.OllC 3?S ‫ط‬T# ‫ة‬2; 4N‫ا@ ا‬
the skull. She most likely has which of the
:‫؟‬6Q‫  أي  ا‬6S ‫ر‬N‫ا‬
following ?
A. Meningioma .6AF, ‫ ورم‬-A
B. Pituitary adenoma .OP&‫"ة ا‬L 6 ‫ ورم‬-B
C. Astrocytoma .6l ‫ ورم‬-C
D. Schwannoma .6Tj ‫ ورم‬-D
E. Hemangioblastoma .6A‫ و‬6‫ ورم أرو‬-E

Une femme de 72 ans a subi une tomodensitométrie


cérébrale à cause de maux de tête. Elle révèle une
masse dans l'hémisphère gauche avec une
hyperostose recouvrante le crâne. Lequel des
énoncés suivants est un diagnostic probable chez
cette patiente?
A. Méningiome
B. Adénome hypophysaire
C. Astrocytome
D. Schwannome
E. Hémangioblastome

86 A
149. A 15-year-old boy has multiple angiomatoses of the 6 ‫"د‬SQ 6A‫رم و‬# ‫  [ب‬15  DQ C
retina and cysts of the kidney and pancreas. Which  ‫ أي‬.‫&س‬.‫ وا‬OJC‫ ا‬6 ‫ت‬4J‫ وآ‬OJ.‫ا‬
of the following brain tumors is most likely to w& ‫ أن‬FJ ]‫آ‬N‫ ا‬6‫ ه‬OJQ‫ ا‬OJL"‫ورام ا‬N‫ا‬
develop in this child ?
:‫؟‬DQT‫ا ا‬b‫&" ه‬
A. Glioblastoma multiforme
B. Meningioma .‫ل‬jN‫"د ا‬SQ 6*#‫ د‬6‫ ورم أرو‬-A
C. Hemangioblastoma .6AF, ‫ ورم‬-B
D. Ependymoma .6A‫ و‬6‫ ورم أرو‬-C
E. Pinealoma .6.[ 62# ‫ ورم‬-D
.O#&[‫ ورم ا;"ة ا‬-E
Un garçon de 15 ans a de multiples angiomatoses
de la rétine et des kystes du rein et du pancréas.
Laquelle des tumeurs cérébrales suivantes est la
plus susceptible de se développer chez cet enfant?
A. Glioblastome multiforme
B. Méningiome
C. Hémangioblastome
D. Ependymome
E. Pinéalome
150. A 30 y male operated for appendicitis one month >j b& OJ‫ ا‬OJC ) I‫ أ‬O&, 30  / D
ago presents for abdominal pain and bloating. All OPT‫ و‬3‫  وه   أ‬،O‫"ة دود‬A‫>ب زا‬Q‫ ا‬g.4#
of the following would make you suspect  ،‫"اد‬4{# Š ŠCSl ‫ف‬, 6C  ‫ آ‬.2.‫ ا‬6
occlusion, except : :‫"ا‬
a. Severity of the pain .3N‫"ة ا‬j .A
b. Eructation .†lQ‫ ا‬.B
c. Absence of gaz emission .‫ق ا;زات‬v2‫ب ا‬JL .C
d. Petechia ..F‫ ا‬.D
e. Similar transient episodes during this .>‫ا ا‬b‫ل ه‬vX ‫ة‬# O># ‫ ب‬.E
month

Un homme de 30 ans opéré de l'appendicite il y a


un mois se présente pour des douleurs abdominales
et des ballonnements. Tout ce qui suit vous ferait
soupçonner une occlusion, à l'exception:
a. La gravité de la douleur
b. Eructation
c. Absence d'émission de gaz
d. Pétéchies
e. Episodes transitoires similaires au cours
de ce mois-ci

87 A
151. You are asked to see a 65 year old unconscious 6 6C "5 O&, 65  / O&S Š& gC\ "* C
patient in the emergency room. He was brought in .‫ف‬S,‫رة إ‬J4# DTQ4‫ ا‬D‫ أ إ‬.‫ارئ‬2‫ ا‬OL
by ambulance. He looks pale and his peripheries ‫ أن‬EFT‫  ا‬J. .O.\‫ردة ور‬# )‫ وأ\ا‬.j ‫"و‬.
are cold and clammy. On examination his pulse ‫ع‬.j{‫ وا‬45/70 ‫ ا"م‬:;< ،‫ د‬/‫ ن‬60 /.&‫"ل ا‬S
rate is 60 bpm, BP 70/45 and saturations are 96% ‫ي‬%‫ ار"ي اآ‬:;‫ ا‬.JlJ4‫ أوآ‬QJ 4 DC %96
on 4 L of oxygen. His Central Venous Pressure :‫ ه‬FJ ]‫آ‬N‫ ا‬EJPQ‫ إن ا‬.(H2O 3, 14) -T
(CVP) is raised (14 cm of H2O.) The most likely .OJQ{‫ ا‬O"[‫ ا‬.A
diagnosis is .JA‫ اآ‬6\S .B
a) Septic shock .6.C5 w& ‫ ذات‬O"i .C
b) Cocaine abuse .OJ5w O"i .D
c) Cardiogenic shock .‫ ا"م‬3l E*  Ol O"i .E
d) Anaphylactic shock
e) Hypovolemic shock

Vous êtes demandé pour un patient de 65 ans


inconscient dans la salle d'urgence. Il a été amené
par ambulance. Il a l'air pâle et ses extremités sont
froides et moites. À l'examen son pouls est de 60
bpm, TA est 70/45 et la saturation est à 96% sous 4
L d'oxygène. Sa pression veineuse centrale (CVP)
est élevée (14 cm de H2O.) Le diagnostic le plus
probable est
a. choc septique .
b. abus de cocaïne
c. choc cardiogénique
d. choc anaphylactique .
e. choc hypovolémique

88 A
152. A 20 year old man who was involved in a road )JC ] "5‫ و‬،J, ‫دث‬F ‫ض‬S O&, 20  ‫ ر‬C
traffic collision is found to be in respiratory ‫&ع‬5 ) -<‫) وو‬.J.& 3 "* .OJ4T& O*A< O 6 ‫وه‬
distress. He is intubated and is bag valve masked. ‫ج *ة  أ‬QF )‫" أ‬PQ‫ ا‬gJ.\ ‫ك‬.X‫ أ‬.O>Q‫ا‬
The anaesthetist tells you that he has to use a lot of 6 -‫ أن ه&ك ا‬-4Q‫ &" ا‬J. ./‫ ا‬O>
force to ventilate the patient. On auscultation there O.[*‫ ا‬.‫  ا["ر‬4N‫ ا‬gl‫ ا‬D‫ل ا>اء إ‬X‫د‬
is reduced air entry on the left hand side of the ‫ ه‬6Q‫ أي  ا‬.N‫ ا‬gl‫ ا‬F OF& OJA‫ا>ا‬
chest. The trachea is deviated to the right hand side. :‫؟‬/‫ا ا‬b> OAv ]‫آ‬N‫ ا‬J#"Q‫ا‬
Which one of the .‫["ر‬C OJ&J4‫ ا‬OSjN‫ ا‬.A
following is the most appropriate management .OJA‫ ا>ا‬O.[*C OJS2* ‫رة‬i .B
option for this patient? .‫ر‬4J‫ ا‬DC ‫ة‬J.‫ آ‬OJ&5 ‫"ام‬PQ,# :;‫ ا‬nJTP .C
a) A chest X-Ray .JJ‫ ا‬DC ‫ة‬J.‫ آ‬OJ&5 ‫"ام‬PQ,# :;‫ ا‬nJTP .D
b) A CT thorax .‫["ر‬C n[ ‫ إاء‬.E
c) Decompression using a large bore
cannula on the left
d) Decompression using a large bore
cannula on the right
e) Insert a chest drain

Un homme de 20 ans qui a été impliqué dans une


collision de la route se trouve être en détresse
respiratoire. Il est intubé et l’anesthésiste vous dit
qu'il doit utiliser beaucoup de force pour aérer
le patient. A l'auscultation, on a un murmure
vesiculaire diminué sur le côté gauche de la
poitrine. La trachée est déviée vers le côté droit.
Lequel de ce qui suit est l'option la plus appropriée
pour ce patient?
a) Une radiographie pulmonaire
b) A CT scan du thorax
c) Décompression en utilisant une grande
canule sur la gauche
d) Décompression en utilisant une grande
canule sur la droite
e) Insérer un drain thoracique

89 A
153. A 73 year old man is admitted with severe  6S ‫ وه‬DTQ4‫ى ا‬N‫ إ‬X‫ أد‬O&, 73  ‫ ر‬B
abdominal pain. His abdomen is soft with )J, Q .3S ‫م‬v‫ إ‬- J )&2# .""j 6&2# 3‫أ‬
generalised tenderness. His past medical history O‫ر‬T5‫ت إ‬#‫ري و‬T5‫ إ‬gC5 ‫ ض‬OJ<‫ ا‬OJ<‫ا‬
includes ischemic heart disease and transient )iF .)‫ا‬j† - g,&Q K )<‫ إن أا‬.‫ة‬#
ischemic attacks. His symptoms are out of ،113 CRP ،21.5 ‫ء‬J.‫ آت ا"م ا‬:6‫ ه‬O"‫ا‬
proportion to his signs. His blood tests are: ،187 J&‫ ا‬،13 O.‫ ا‬،13.5 J#CLJ>‫ا‬
WBC 21.5, CRP 113, Hb 13.5, Urea 13, Creatinine .5.1 ‫م‬J,.‫ ا‬،139 ‫ا[دم‬
187, Na 139, K 5.1. ،98 PO2 ،7.32 PH :6Q‫ ه آ‬6‫ز ا"م ا‬L
His arterial blood gas is as follows: PH 7.32, PO2 EJPQ‫ إن ا‬.14.1 HCO3 ،15.9 BE ،3.1 PCO2
98, PCO2 3.1, BE -15.9, HCO3 14.1. The diagnosis :‫ ه‬QF‫ا‬
is probably: .6#v*Q,‫ ا‬F .A
a) Metabolic acidosis .64T& /S - 6#v*Q,‫ ا‬F .B
b) Metabolic acidosis with respiratory .64T& F .C
compensation .6#v*Q,‫ ا‬/S - 64T& F .D
c) Respiratory acidosis .‫"رات‬P 6\S .E
d) Respiratory acidosis with metabolic
compensation
e) Drug abuse

Un homme de 73 ans est admis avec des douleurs


abdominales sévères. Son abdomen est souple avec
une sensibilité généralisée. Ses antécédents
médicaux comprennent une cardiopathie
ischémique et des attaques ischémiques
transitoires. Ses symptômes sont hors
de proportion de ses signes. Ses tests sanguins
sont:
WBC 21,5, CRP 113, Hb 13,5, Urée 13, Créatinine
187, Na 139, K 5.1.
Son gaz du sang artériel est la suivante: PH 7.32,
PO2 98, PCO2 3.1, BE -15.9,
HCO3 14.1. Le diagnostic est probablement:
a) acidose métabolique
b) acidose métabolique avec compensation
respiratoire
c) acidose respiratoire
d) acidose respiratoire avec compensation
métabolique
e) E. abus des drogues

90 A
154. A 48-year old obese lady presents to the ER with 3‫  أ‬6S 6‫ وه‬O&, 48 ‫ ه‬O&"# ‫"ة‬J, ‫ ت‬A
severe colicky RUQ pain associated with nausea ‫ن‬J]L - @‫ا‬Q N‫ي ا‬CS‫ ا‬-#‫ ا‬6 E;# )J.j
and 2 episodes of vomiting. On examination there ‫ "م ود * دم أو‬EFT‫ &" ا‬J. .†J*Q‫  ا‬JQ#‫و‬
is no anaemia or jaundice. She has a temperature of ‫م‬v‫  إ‬6S 6‫ ه‬.OW°38 >‫ ار‬O‫ در‬.‫ن‬5
38C. She has rebound tenderness and guarding in ‫ي‬CS‫ ا‬-#‫ ا‬6 2.‫ت "ار ا‬v 6 u&‫ و‬-‫را‬
the RUQ. WBC – 13.1, CRP 20 and liver function ‫ص‬F‫ و‬20 CRP ،13.1 ‫ء‬J.‫ آت ا"م ا‬.N‫ا‬
tests are normal. What is the most likely diagnosis? .OJSJ.\ ".‫ ا‬OTJG‫و‬
a) Acute cholecystitis :‫؟‬FJ ]‫آ‬N‫ ا‬EJPQ‫ ه ا‬
b) Cholelithiasis .‫>ب ارة د‬Q‫ ا‬.A
c) Biliary colic .O‫او‬Ti ‫ت‬J[ .B
d) Cholangitis .‫اوي‬Ti E; .C
e) Diverticulitis .O‫او‬T[‫>ب ا*&ة ا‬Q‫ ا‬.D
.u‫>ب ا‬Q‫ ا‬.E
Une dame de 48 ans obèse se présente aux urgences
avec de graves douleurs au QSD à type de
coliques associées à des nausées et 2 épisodes de
vomissements. À l'examen, il n'y a pas d'anémie ou
d’ictere. Elle a une température de 38C. Elle
presente une douleur à la decompression
abdominale et une defense dans le QSD. WBC -
13.1, les tests de CRP 20 et la fonction hépatique
sont normaux. Quel est le diagnostic le plus
probable?
A.Cholécystite aiguë
B. Cholélithiase
C. Colique biliaire
D. Cholangitis
E. Diverticulite

91 A
155. A 69 year old lady who is on Ibuprofen and J‫و‬#.”# ‫ج‬vS -P O&, 69 ‫"ة ه‬J, C
dexamethasone for rheumatoid arthritis presented "* .‫ي‬J|‫ ر‬iT ‫>ب‬Q# >Q#i‫ إ‬g.4# ‫زون‬QJ4‫ودآ‬
to the E R with sudden onset severe epigastric pain 3N ‰T ‫"ء‬# g.4# ‫ارئ‬2‫ ا‬345 D‫ت إ‬
and vomiting. On examination her abdomen was &J ‫&> آن‬2# ‫ أن‬EFT‫  ا‬J. .†J*‫"" و‬j 6,j
soft with mild tenderness in the epigastrium and 3 .N‫ي ا‬CS‫ ا‬-#‫ف ا‬,‫ ا‬6 :J4# ‫م‬v‫ إ‬-
RUQ. There was no guarding or rigidity. Bowel ‫آن ه&ك‬.O#vi ‫ أو‬2.‫ت ا‬v 6 ‫ت‬l& "
sounds were decreased. What is the single most OJ‫آ] أه‬N‫ي ا‬FQ‫  ه ا‬.‫ء‬S{‫ات ا‬i‫ أ‬6 E5&
important investigation you should request for this :‫؟‬/‫ا ا‬b> ).C\ gl ‫ي‬b‫ا‬
patient? .2.C 64J\&; J‫ ر‬.A
a) MRI of the abdomen .‫ء‬J.‫"اد آت ا"م ا‬S .B
b) WBC count .O.[Q& ‫["ر‬C OJ&J, OSj‫ أ‬.C
c) Erect Chest X-Ray .2.C OJS2* ‫رة‬i .D
d) CT scan of the abdomen 6W J?& / ‫ي‬C 6‫ ه‬6CX‫ دا‬J?& .E
e) Upper GI endoscopy / OGD .‫ ي‬6&|‫"ي ا‬S
(OesophagoGastroDuodenoscopy)

Une dame de 69 ans qui est sous Ibuprofen et


dexaméthasone pour arthrite rhumatoïde se
présente aux urgences avec l'apparition soudaine de
douleurs épigastriques et vomissements sévères. À
l'examen son abdomen est souple avec une légère
sensibilité à l'épigastre et QSD. Il n'y avait pas de
defense. Les bruits de peristaltisme intestinal sont
diminués. Quelle est l’examen à demander dans ce
cas ( une réponse)?
a) IRM de l'abdomen
b) FNS pour voir si leucocytose
c) Radio thorax debout
d) CT scan de l'abdomen
e) Endoscopie digestive / OGD
(oesophagogastroduodénoscopie)
156. A young man is admitted to the emergency room 6 O&S2 )<S g.4# ‫ارئ‬2‫ ا‬345 D‫ب إ‬j ‫ أ‬E
for a stab in the abdomen one hour ago. There is no ‫ ا"م‬:;< .J.‫ آ‬6‫ر‬X n% " K .O, b& 2.‫ا‬
major external bleeding. His BP is 80/30, Pulse is 6Q‫ ا{اء ا‬..j ‫"و‬.‫ و‬O*J5"# 130 /.&‫ ا‬،30/80
130bpm and he looks pale. The next thing to do is: :‫ه‬
a) Take a detailed history .OC[T ‫ة‬J, DC ‫[ل‬FQ,K‫ ا‬.A
b) Ask for an abdominal ultrasound .2.C ‫ إاء ق ا[ت‬gC\ .B
c) Ask for a CT Scan .OJS2* ‫رة‬i ‫ إاء‬gC\ .C
d) Ask for an abdominal MRI .2.C 64J\&; J‫ إاء ر‬gC\ .D
e) Prepare for immediate laparotomy .‫ ري‬2# QT JFQ‫ ا‬.E

Un jeune homme est admis dans la salle d'urgence


pour un coup de poignard dans l'abdomen il y a une
heure. Il n'y a pas de saignement externe majeur. Sa
tension arterielle est 80/30 et le pouls est à 130bpm
et il a l'air pâle. La conduite à faire est:
a) Prendre une histoire détaillée
b) Demander une échographie abdominale
c) Demander un CT Scan
d) Demander une IRM abdominale
e) Préparer pour une laparotomie immédiate

92 A
157. A 22 year old unmarried man complains of OS# b& OX‫   دو‬O&, 22  ‫ ر زب‬D
dizziness for several weeks with nausea and EFT‫  ا‬J. .‫ن و"م ازن‬J]L - O*‫ا‬Q -J#,‫أ‬
imbalance. A neurological examination reveals a )‫ >را‬.‫هت‬l{‫ ا‬-J 6 ‫ ود رأرأة‬6.[S‫ا‬
nystagmus in all directions. Motor skills and – O‫) ا‬4S& ‫ وأ‬،OJSJ.\ ),4‫ وإ‬OJ‫آ‬F‫ا‬
sensitivity are normal, as are osteo-tendinous :;< - O"C ‫;ت‬.[ ‫م‬S‫ ا‬EFT‫> ا‬G‫ أ‬.OJ?S‫ا‬
reflexes. The general examination reveals ‫ ه‬.OW O‫ در‬37 ‫ ارة‬O‫ ودر‬،7/12 6j
pigmented skin lesions along with a blood pressure :‫؟‬EJPQ# ‫"ء‬.C Š‫ا‬Q5‫إ‬
of 12/7, a temperature of 37 degrees Celsius. What .6S, :J2P .A
do you suggest for attaining a diagnosis? .O.5C OSj‫رة أ‬i .B
a) Audiogram .6&25 ‫ل‬%# .C
b) cervical Radiology .‫"غ‬C 64J\&; J‫ ر‬.D
c) Spinal Tap .OllC OSj‫رة أ‬i .E
d) MRI
e) Radiology of the skull

Un homme de 22 ans celibataire se plaint de vertige


depuis quelques semaines avec nausée et
déséquilibre. A l’examen neurologique il a un
nystagmus dans toutes les directions. La motricité
et la sensibilité sont normales, de même que les
réflexes osteo-tendineux. L’examen général révèle
des tâches cutanées pigmentées avec une tension
artérielle à 12/7, une température de 37 degrés
celsius. Que suggérez vous pour avancer dans le
diagnostic ?
a) Audiogramme
b) Radio cervicale
c) Ponction Lombaire
d) IRM cérébrale
e) Radio du crâne

93 A
158. A 56-year-old woman with no particular history ‫ ت‬،‫"دة‬F OJ.\ ‫ة‬J, ‫ دون‬،O&, 56 ‫"ة ه‬J, B
presents a recent abdominal bloating. The ‫ق ا[ت‬T# "‫آ‬wQ‫ ا‬3 .]" 6&2# ‫خ‬TQ‫   ا‬6‫وه‬
ultrasound confirmed the presence of ascites and a - 3,15 N‫ ا‬/J.‫ ا‬DC O4J‫*ء وآ‬4Q,‫ ود ا‬
right ovarian cyst of 15 cm with thick wall and with D‫ إ‬2  OJCX‫ت دا‬4J ‫"دة‬SQ I#‫ ا‬- ‫ "ار‬O‫آ‬,
multiple intra-cystic vegetations of 2 to 3 cm ‫ي‬b‫\ن ا‬4‫  ه ا‬،O‫ ر‘ ا‬bX‫ أ|&ء أ‬.3,3
.During history taking, what type of cancer would :‫>؟‬QCA ‘‫ ر‬6 )& qF.Q,
you search in her family history (one answer) .3‫ ا‬O2#‫ ا["ر و‬.A
A. Breast and endometrial ./J.‫ ا["ر وا‬.B
B. Breast and Ovarian .3‫ ا‬O2#‫ و‬/J.‫ ا‬.C
C. Ovarian and endometrial ./J.‫ وا‬O]‫ ا‬.D
D. Bladder and ovary .3‫ ا‬O2#‫ و‬O]‫ ا‬.E
E. Bladder and endometrial

Une patiente de 56 ans sans antécédents particuliers


se présente pour ballonnement abdominal récent.
L’échographie confirme la présence d’ascite ainsi
qu’un kyste ovarien Dt de 15 cm, à cloisons
épaisses, avec des végétations intra-kystiques
multiples de 2 à 3 cm chacune.Dans
l’interrogatoire, l’histoire familiale va rechercher
quels antécédents familiaux de cancer ( une
reponse)
A. Sein et endomètre
B. Sein et ovaire
C. Ovaire et endometre
D. Vesicule et ovaire
E. Vesicule et endometre
159. Ms. Nada, is pregnant and presents at 8 w of ‫س‬.F‫  ا‬-J#,‫ أ‬8 "S# ‫" ت‬5‫ و‬، ،‫"ة "ى‬J4‫ ا‬E
amenorrhea an episode of bleeding of moderate < .O2,Q ‫ارة‬%L ‫ ذات‬n%   6‫ وه‬q2‫ا‬
abundance. Among the tests below, which one is ‫ا‬b‫ ه‬OFi "‫ي †آ‬b‫ ا‬N‫  ه ا‬،OJQ‫ص ا‬FT‫ا‬
the best for asserting the viability of :‫؟‬F‫ا‬
thispregnancy? .6.‫ ا‬HCG ‫س‬J5 .A
A. Measurement of urinary HCG .‫ز‬v.‫ ا‬6 HCG QJ# ‫ة‬S .B
B. Assay of plasma beta HCG .‫ز‬v.‫ون ا‬JQ4‫و‬# ‫ة‬S .C
C. The assay of plasma progesterone .FC 6&‫ ا‬EJPQ‫ ا‬.D
D. immunodiagnostic pregnancy .OJi ‫رة ق‬i .E
E. Ultrasound

Madame Nada, enceinte, à 8 semaines


d’aménorrhée des saignements d'abondance
modérée. Parmi les examens ci-dessous, quel est
celui qui permet d'affirmer au mieux l'évolutivité
de la grossesse ?
A. Le dosage de l'HCG urinaire
B. Le dosage de l'HCG bêta
plasmatique
C. Le dosage de la progestérone
plasmatique
D. Le diagnostic immunologique de
grossesse
E. L'échographie

94 A
160. A 1 month old infant presented, at rest, an ،‫ق هم‬5‫  ازر‬O‫ &" اا‬6S "‫> وا‬j  T\ C
important cyanosis, which increased when he cried :O4‫ ا?ه ا‬.(""j ‫ق‬5‫ء )ازر‬.‫" &" ا‬% ‫ي‬b‫وا‬
(severe cyanosis). Physical findings: No dyspnea or DC‫ أ‬.[ OPT&‫ ا‬،aT ‫رع‬4 ‫ أو‬aT @J< ‫"م ود‬
tachypnea, murmur loudest at the upper left sternal 6S‫[ ا‬Q‫ ا‬.4N‫ي ا‬CS‫ ا‬E*‫ ا‬O "&
border. Chest Radiogram: decreased pulmonary .OA‫ ا‬OJA‫رات ا‬j{‫ ا‬6 E5& :‫["ر‬C
vascular markings. :‫ ه‬FJ ]‫آ‬N‫ ا‬EJPQ‫إن ا‬
The most likely diagnosis is: .‫>ي‬#N‫@ ا[م ا‬J .A
A. Aortic valve stenosis .u4&Q‫ ا‬E5 4N‫ ا‬gC*‫ ا‬O‫ز‬vQ .B
B. Hypoplastic left heart syndrome . OJ#‫ ر‬.C
C. Tetralogy of Fallot ""j ‫ع‬T‫ ار‬- 6&J2.‫ ا‬%F‫ ا‬6 J.‫ آ‬gJ .D
D. Large ventricular septal defect with severe .‫ي‬A‫ ا‬:;‫ ا‬6
pulmonary hypertension

Un petit nourrisson âgé de 2 mois présente au repos


une importante cyanose augmentant aux pleurs
(cyanose sévère). L’examen physique ne retrouve
pas de dyspnée ou tachypnée, mais un souffle
systolique intense au 2eme espace intercostal
gauche. La radiographie du thorax met en évidence
une diminution de la vascularisation pulmonaire.
Le diagnostic le plus probable est:
A. Sténose valvulaire aortique
B. Hypoplasie du cœur gauche
C. Tétralogie de Fallot
D. Large communication inter ventriculaire
avec
hypertension artérielle pulmonaire

95 A
161. A 64 year old woman with known type 2 diabetes ‫ع‬T‫ ار‬،2: ‫ي‬, ‫  داء‬6S O&, 64 ‫"ة ه‬J, B
mellitus, hypertension and chronic atrial   6‫ ت وه‬،% 6&‫ن أذ‬T‫ ور‬:;<
fibrillation presents for anorexia, nausea and ‫ت‬2.] ،J4{# uSQ .‫ء‬J5‫ن وا‬J]L ،OJ>j ‫*"ان‬
vomiting. She has been treated with insulin, ACE >w# ‫أدت‬.‫] ي‬P ‫ ود‬O.C*&‫ ا‬J4&Jl‫ة أ‬JX
inhibitors and oral anticoagulant. She reports ‫ ا"م‬, ‫ط‬.‫ ه‬g.4# ‫ا‬X† J4{‫  ت ا‬ICC5
decreasing her insulin doses lately because of :;< 6 ‫ط‬.‫ ه‬g.4# :;C /P‫ ا"واء ا‬IT5‫وأو‬
hypoglycemia and has stopped her HCT ‫ و‬12.8 HB :‫ص ا"م‬F ‫>ت‬G‫ أ‬.‫ا"م‬
antihypertensive medication because of low blood :‫ ه‬FJ ]‫آ‬N‫ ا‬EJPQ‫ ا‬.5=K ‫ و‬132=Na،%40
pressure. Her blood tests show :Hb of 12.8 and .‫ د‬OJ.C5 OC ‫ء‬Q‫ ا‬.A
HCT of 40% ; Na= 132 and K= 5. The MOST .O?‫ ا;"ة ا‬6 ‫[ر د‬5 .B
LIKELY diagnosis is : .‫ دة‬OA‫ ر‬O‫ وذ‬.C
A. Acute myocardial infarction .‫ د‬6L‫ د‬6A‫ و‬n% .D
B. Acute adrenal failure .6‫ ه‬n% .E
C. Acute pulmonary edema
D. Acute cerebrovascular bleed
E. Gastrointestinal bleed

Une femme de 64ans connue avoir un diabète de


type 2, hypertension et une fibrillation auriculaire
chronique se présente pour anorexie, nausées et
vomissements. Elle a été traitée avec de l’insuline,
inhibiteurs de l’ACE (angiotensine converting
enzyme) et des anticoagulants oraux. Elle rapporte
dernièrement une diminution des doses d’insuline à
cause d’une hypoglycémie et un arrêt des
médicaments antihypertenseurs à cause de la
diminution de la tension artérielle. Les examens de
sang montrent Hb à 12,8 et Ht à 40%, Na=132, K =
5. Le diagnostic le plus probable est :
A. Infarctus aigu du myocarde
B. Insuffisance surrénalienne aigue
C. Oedème aigu pulmonaire
D. Hémorragie vasculaire cérébrale aigue
E. Hémorragie gastrointestinale

96 A
162. A 25-year-old male patient, smoker, presents with ‫"ري‬i 3‫   أ‬،X" ،O&, 25  / ‫ ر‬C
chest pain of 3-hours duration, relieved ‫ة‬2P‫ ا‬.‫&ء م‬F{‫ &" ا‬4FQ ،‫ت‬, 3 b& Q4
by leaning forward. The next step in management :6‫ ه‬OlS‫ ا‬6 OJQ‫ا‬
is:
A. Administer NSAID’s and send home O"JA‫و‬JQ, JL ‫>ب‬Q‫ء دات ا‬2‫إ‬ .A
B. Request ventilation perfusion scans .‫ل‬%&‫ ا‬D‫) إ‬,‫وإر‬
C. Request echocardiography .OJF&‫ ا‬O>Q‫  ا‬qF.‫ ا‬gC\ .B
D. Administer sublingual nitroglycerin and .gC*‫"ى ا‬i :J2P gC\ .C
reassure patient Ow\‫ن و‬4C‫ ا‬IF J4JCL‫و‬Q ‫ء‬2‫إ‬ .D
E. Advise psychiatric evaluation ./‫ا‬
.64T 3JJ* gC\ .E
Un homme âgé de 25 ans fumeur, se présente pour
une douleur thoracique qui dure depuis 3 heures,
soulagée par la position penchée en avant. La
prochaine étape dans la conduite à tenir est :
A. Administrer des antiinflamatoires non
stéroîdiens et renvoyer le patient
B. Demander une scintigraphie ventilation
perfusion
C. Demander une échographie cardiaque
D. Administrer de la nitroglycémie sublinguale et
rassurer le patient
E. Conseiller une evaluation psychiatrique
163. A 75-year-old woman presents to the emergency 6‫ارئ وه‬2‫ ا‬345 D‫ ت إ‬O&, 75 ‫"ة ه‬J, B
room with abrupt onset of left-sided weakness that gl‫ ا‬6 nS# ‰T ‫"ء‬#  O, b& 6S
began 1 hour ago. Assuming all imaging tests are ‫ة‬Q 6‫ ه‬OJS‫ر ا‬i‫ ا‬-J ‫اض أن‬Q‫ ا‬- .4N‫ا‬
immediately available and there are no [Q‫@ ا‬A‫ا‬2 ‫ب‬.2Q,‫ ه&ك دات ا‬aJ‫را و‬
contraindications to the following imaging O”# O‫"ا‬.‫ ا‬6 ‫ ااؤ‬gl ‫ي‬b‫ ا‬EFT‫  ه ا‬،OJQ‫ا‬
modalities, which test should initially be :‫؟‬OJ&J‫ او‬O4‫ ا‬O‫ا‬
performed in routine clinical care? .O.5‫ اأس وا‬OJ‫و‬N 6S2* [ .A
A. CT angiogram of the head and neck .OSj‫"ون أ‬# ‫أس‬C 6S2* [ .B
B. CT scan of the head without contrast .‫"غ‬C 64J\&; J‫ ر‬.C
C. Magnetic resonance image (MRI) of the 64J\&; J‫أس ور‬C 64J\&; J‫ ر‬.D
brain .O.5‫ اأس وا‬OJ‫و‬N
D. MRI of the brain and magnetic resonance .OSj‫ أ‬- ‫أس‬C 6S2* [ .E
angiogram (MRA) of the head and neck
E. CT scan of the head with contrast

Une femme âgée de 75ans se présente à l’urgence


pour une apparition brutale d’une faiblesse gauche
débutante depuis une heure considérant que tous
les tests d’imagerie sont disponibles et ne sont pas
contreindiqués. Quel test de routine parmi les
suivants devrait être pratiqué :
A- Angiographie par CT de la tête et du cou
B- CT scanner de la tête sans contrast
C- IRM cerebral
D- IRM cerebral et MRA de la tête et du cou
E- CT scan cerebral avec contrast

97 A
164. A 20-year-old sportsman presents with pain and 6 ‫ ورم‬3‫   أ‬O&, 20  6<‫ ر ر‬A
swelling of he knee following a rotational trauma in ‫ة‬# gSC‫  ا‬Ol O‫"ار‬Q,‫ ا‬OF< 6 O.‫اآ‬
football. X-Rays are normal. The most probable FJ ]‫آ‬N‫ ا‬EJPQ‫ ا‬.OJC, OJS‫ا[ر ا‬.‫ا*"م‬
diagnosis would be :‫ه‬
A. A tear of the medial meniscus .OJ4{‫ ا‬Ov>‫ ا‬6 ‫ق‬% .A
B. A tear of the lateral meniscus .OJ‫ ا‬Ov>‫ ا‬6 ‫ق‬% .B
C. A tear of the anterior cruciate ligament .6N‫ ا‬6.[Q‫ط ا‬#‫ ا‬6 ‫ق‬% .C
D. A tear of the posterior cruciate ligament .6TCP‫ ا‬6.[Q‫ط ا‬#‫ ا‬6 ‫ق‬% .D

Un sportif de 20 ans se présente avec douleur et


gonflement du genou suite à un traumatisme de
rotation dans le football. Les rayons X sont
normaux. Le diagnostic le plus probable serait
A. Une déchirure du ménisque médial
B. Une déchirure du ménisque latéral
C. Une déchirure du ligament croisé antérieur
D. Une déchirure du ligament croisé postérieur
165. A 35-year-old woman who underwent a ‫[ل‬WQ,‫ ا‬OJCS ‫ع‬.,‫ أ‬b& ISX O&, 35 ‫"ة ه‬J, A
laparoscopic cholecystectomy 1 week ago comes to 6S >N ‫ارئ‬2‫ ا‬345 D‫ ت إ‬.‫&?ر‬# ‫ارة‬
the emergency department with a history of 3 days  ‫ أي‬.‫ن‬5‫ء و‬J5‫ ا‬،‫ن‬J]L ،2.‫ ا‬6 3‫ أم  أ‬3 b&
of abdominal pain, nausea, vomiting and jaundice. :‫؟‬FJ ]‫ ا‬EJPQ‫ ه ا‬6Q‫ا‬
Which of the following diagnoses is most likely? ‫اء‬T[C -Jl - O‫او‬T[‫ ا*&ة ا‬O#i‫ إ‬.A
A. Bile duct injury with intra-abdominal bile .2.‫ ا‬X‫دا‬
collection .6‫ا‬l‫ ا‬S‫" ا‬S# 6SJ.\ ‫ج‬%‫ن وا‬J]L .B
B. Normal postoperative nausea and .6‫ا‬l‫ ا‬S‫" ا‬S# ‫ي‬S ‫"اد‬4‫ص أو ا‬C .C
discomfort .‫ب‬4 - ‫ ي‬6&|{‫ ا‬O#i‫ إ‬.D
C. Postoperative bowel obstruction or ileus
D. Duodenal injury with leakage

Une femme de 35 ans qui a subi une


cholécystectomie laparoscopique il y a une semaine
vient à l'urgence avec une histoire de 3 jours de
douleurs abdominales, nausées, vomissements et
jaunisse. Lequel des diagnostics suivants est le plus
probable?
A. Lésion des voies biliaires avec la collection de
bile intra-abdominale
B. Nausées et malaises postopératoires habituels
C. Obstruction intestinale postopératoire ou iléus
D. Lésion duodénale avec fuite

98 A
166. One of the more common pathogens responsible of ‫رع‬% OJQ‫ ا‬OA‫ت ا‬QK‫†ول  ا‬4‫ اض ا‬S‫ إن ا‬E
pulmonary infections in immunocompromised :‫ ه‬6&‫ز ا‬S‫ ا‬D< "& ‫ء‬N‫ا‬
patients following transplantation is: .O‫و‬.‫ ا‬O‫&*د‬S‫ ارات ا‬.A
A. Staphylococcus epidermidis .OJ%&‫ت ا‬J"Q4‫ ا‬.B
B. Haemophilus influenzae .OJ*Jj‫ت ا‬J]2‫ ا‬.C
C. Clostridium Botulinum .OA‫ ا‬O"*S‫ ارات ا‬.D
D. Streptococcus Pneumoniae .OJ&‫ ار‬OA‫ ا‬O4JQ‫ ا‬.E
E. Pneumocystis Carinii

L'un des agents pathogènes les plus courants et


responsable d'infections pulmonaires chez les
patients immunodéprimés après une transplantation
est:
A. Staphylococcus epidermidis
B. Haemophilus influenzae
C. Clostridium botulinum
D. Streptococcus pneumoniae
E. Pneumocystis carinii
167. Which of the following conditions is a medical :‫؟‬O[J*‫دة ا‬KC J.\ vX" 6"Q4 OJQ‫ت ا‬KF‫ أي  ا‬D
indication for routine delivery by caesarean section .‫ ج‬6,‫و‬JT‫" ا‬.‫>ب ا‬Q‫ ا‬.A
? .‫وان‬N‫ ا‬.5 ‫دة‬K‫ و‬.B
A. Hepatitis C virus .O*#, O[J5 ‫دة‬K‫ و‬.C
B. Preterm labour  6]‫ة وا‬X†‫ ا‬OJ  ‫ول‬N‫أم ا‬Q# F‫ ا‬.D
C. Previous lower segment caesarean section .‫ اأس‬OJ
D. Twin pregnancy with first twin breech and OJ  6]‫ اأس وا‬OJ  ‫ول‬N‫أم ا‬Q# F‫ ا‬.E
second twin cephalic .‫ة‬X†‫ا‬
E. Twin pregnancy with first twin cephalic and
second twin breech
Laquelle des conditions suivantes est une indication
médicale pour la délivrance de routine par
césarienne?
A. le virus de l'hépatite C
B. travail prématuré
C. antécédents d’une césarienne du segment
inférieur
D. Grossesse gémellaire avec le premier
jumeau en présentation de siège et le second
en céphalique
E. Grossesse gémellaire avec le premier
céphalique et le deuxième en siège

99 A
168. A 52-year-old woman attends the general qJ OS‫ ا‬OF[‫ ا‬gJ.\ "& ‫ ت‬O&, 52 ‫ إأة ه‬A
practitioner saying that she last had a period many ‫آ"ة‬wQ JL 6‫ ه‬.>j‫ "ة أ‬b& I‫ آ‬O>j ‫ دة‬Xx ‫أدت أن‬
months ago. She is not sure if she has undergone  .‫  أاض‬6S K >N q2‫ع ا‬2*‫ة ا‬QT# ICX‫> د‬w#
the menopause as she has no symptoms. A serum EJPQ‫ ا‬6 "4 ‫ي  أن‬b‫ ا‬6C[‫ ا‬EFT‫ه ا‬
test of which of the following could aid a clinical :‫؟‬q2‫ع ا‬2*K ‫ي‬4‫ا‬
diagnosis of menopause ? (FSH).glC ).&‫ ا>ن ا‬.A
(HCG).‫ي‬.‫ ا‬OJ‫ هن ا‬.B
A- Follicle-stimulating hormone (FSH)
.C‫ ا>ن ا‬.C
B- Human chorionic gonadotrophin (HCG)
.J‫و‬Q,‫ إ‬.D
C- Luteinizing hormone
.‫ون‬JQ4‫و‬# .E
D- Oestrogen
E- Progesterone
Une femme de 52ans se présente chez le médecin
généraliste en disant qu'elle avait eu une
menstruation il y a plusieurs mois. Comme elle n'a
pas de symptômes, elle ne sait pas si elle a eu la
ménopause. Un test sérique qui pourrait aider au
diagnostic clinique de la ménopause?
A-Hormone stimulante folliculaire (FSH)
B- La gonadotrophine chorionique humaine (HCG)
C-Hormone lutéinisante
D- Oestrogène
E- Progesterone
169. An 18-year-old man is brought to the emergency ‫ ح‬g.4# ‫ارئ‬2‫ ا‬OL D‫ أ إ‬، 18  ‫ب‬j A
department with a stab wound just to the right of -C‫ ا‬J# O4‫ ا‬6 E*‫ ا‬J DC O&S\  u
the sternum in the sixth intercostal space. His blood OQX gC5 ‫ات‬i‫ أ‬I? .@.A‫ ز‬3C 80 ‫ ا"م‬:;< .‫دس‬4‫ا‬
pressure is 80 mm Hg. Faint heart sounds and  D&J‫ ا‬O>l‫ ا‬6 -4Q‫  ا‬J. . O5&Q ‫ت‬.‫و‬
pulsus paradoxus are noted. Auscultation of the ‫ أن ن‬gl 6Q‫ أي  ا‬.aT&‫ت ا‬i 6 E5& ‫ا["ر‬
:‫ ؟‬/‫ا ا‬b> 6A".‫ ا‬6vS‫ ا‬J#"Q‫ه ا‬
right chest reveals decreased breath sounds. The
. N‫ف ا["ر ا‬l nj‫ ر‬.A
initial management of this patient should be which
.‫ * دم‬.B
of the following?
.‫["ر‬C ‫ري‬F 6*.\ [ .C
a- Aspiration of the right chest cavity
.‫ ا["ر‬-# .D
b- Transfusion
.‫ ا["ر‬J?& .E
c- CT scan of the thorax
d- Thoracotomy
e- Thoracoscopy
Un jeune de 18 ans est amené aux urgences avec un
coup de couteau juste à droite du sternum dans le
sixième espace intercostal. Sa tension artérielle est
de 80 mm de Hg. Les bruits cardiaques sont
inaudibles et un pouls paradoxal sont notés.
L’auscultation pulmonaire révèle une diminution
du murmure vésiculaire du coté droit. La prise en
charge initiale de ce patient doit être:
a- Aspiration de la cavité pleurale
droite
b- Transfusion
c- Tomodensitométrie du thorax
d- Thoracotomie
e- Thoracoscopie

100 A
170. A 65 year old man presents with dysphagia. He has >j b& mK "* . -C# 4  6S  65  ‫ ر‬E
noticed a 2 month history of indigestion and post >j b&‫ و‬، ‫آ‬N‫" ا‬S# ‫ء‬vQ{# ‫س‬4‫ وإ‬6‫ ه‬4S# )Q#i‫إ‬
prandial fullness and a one month history of nausea :‫؟‬N‫ج ا‬vS‫  ه ا‬.‫زن‬C ‫ن و*"ان‬J]L ‫ "وث‬mv ‫"أ‬#
and weight loss. What is the best management: .2.C ‫ ق ا[ت‬.A
a- Abdominal ultrasound .‫رم‬# OSC# .B
b- Barium swallow .‫ وا["ر‬2.C ‫ري‬F 6*.\ [ .C
c- CT abdomen and thorax 6CX‫ دا‬J?& .D
d- Endoscopy O%X‫ و‬6CX‫ دا‬J?& .E
e- Endoscopy and biopsy
Un homme de 65 ans se présente avec une
dysphagie. Il signale une histoire de 2 mois
d'indigestion et de plénitude postprandiale, une
histoire d’un mois de nausée et de perte de poids.
Quelle est la meilleure conduite:
a- Echographie abdominale
b- Repas baryté
c- CT scan abdominal et thoracique
d- Endoscopie
e- Endoscopie et biopsie
171. Which of the following findings of chest x-ray is ]‫ أآ‬6‫ ه‬OJ&J4‫ ا["ر ا‬OSjN OJQ‫أي  ا?ه ا‬ A
most consistent with the diagnosis of chronic :‫؟‬%‫ي ا‬A‫"اد ا‬4{‫ داء ا‬EJP - *#2
obstructive pulmonary disease (COPD)? .OJ.J[5 O‫آ‬, ،‫خ‬TQ‫ ط ا‬.A
A. Hyperinflation, bronchial thickening .J*&‫ ا‬:JF 6 OTX‫ت ر‬j‫ ر‬.B
B. Perihilar fluffy infiltrates .‫ ب‬6J‫ آ‬:X، 6.& ‫ب‬.[‫ ا‬.C
C. Pleural effusion, Kerley B lines .‫ة‬Q& O"* OQ .D
D. Diffuse nodular opacities .OJCT, OJ‫ل ز‬vG .E
E. Ground glass shadowing

Lequel parmi les résultats radiographiques suivants


est le plus compatible avec le diagnostic de la
maladie pulmonaire obstructive chronique
(MPOC)?
A. Hyperinflation, épaississement
bronchique
B. Infiltrats moelleux périhilaires
C. Épanchement pleural, Ligne de Kerley
B
D. Opacités nodulaires diffuses
E. Aspect des poumons en verredépoli
172. The earliest visual disturbance in a patient with a 6 ‫رم‬# ‫ [ب‬/ "& .‫[ي ا‬.‫اب ا‬2<{‫ إن ا‬B
pituitary adenoma is: :‫ ه‬OJP&‫ا;"ة ا‬
A. Papilledema .OJCF‫ ا‬O‫ وذ‬.A
B. Bitemporal homonymous hemianopia .gl‫ | ا‬JJL"[‫ ا‬J*‫ ا‬D .B
C. Central monocular scotoma .JS‫ ا‬O‫ أد‬O%‫ آ‬OQ .C
D. Pupillary anisocoria .JQ5"F‫وت ا‬T .D
E. Ptosis .Tl‫ ا‬6" .E

Le premier signe de la perturbation visuelle chez un


patient avec un adenoma hypophysaire est:
A. Oedèmepapillaire
B. Hémianopsie homonyme bitemporale
C. Scotome monoculaire central
D. Anisocorie pupillaire
E. Ptose
101 A
173. An 83 y.o. woman complains of generalized fatigue 6 b& .>j‫ أ‬3 b& 3S gS   O&, 83 ‫ ااة ه‬C
since 3 months. Since 6 months she eats only ‫ ود‬6T& >‫ إ‬.‫ وب اي‬gCS ‫م‬S\ :* ‫آ‬w >j‫أ‬
canned food and tea. She denies fever, pain, or EF .O.j ‫"و‬. EFT‫ &" ا‬.‫از‬.‫ ا‬6 ‫ أو دم‬،‫م‬Kx ،‫ارة‬
blood in stools. On exam, she appears pale. A /TP& J#CLJ‫ي ه‬.P‫ ا‬EFT‫> ا‬G‫ أ‬.6.C, 6TP‫ا"م ا‬
hemocult test is negative. Laboratory showed low O‫ ا‬b> )Ti‫ و‬gl 6Q‫ أي  ا‬.-T MCV ‫و‬
:‫ أا<>؟‬DC ‫ة‬2J4C
hemoglobin and high MCV. Which of the
.J,J .A
following should be given to the patient to control
.‫ ج‬JQJ .B
her symptoms?
.‫ ارق‬/ .C
A. Niacin
.S‫  \@ ا‬B12 JQJ .D
B. Vitamin C
.3T‫  \@ ا‬B12 JQJ .E
C. Folic acid
D. Intramuscular Vitamin B12
E. Oral vitamin B12

Une femme de 83 ans se plaint de fatigue


généralisée depuis 3 mois. Depuis 6 mois, elle ne
mange que la nourriture en conserve et le thé. Elle
nie la fièvre, la douleur, ou le sang dans les selles.
À l'examen, elle apparaît pâle. Un test de hemocult
est négatif. Le laboratoire a montré un taux faible
d'hémoglobine et un VCM élevé. Lequel des
éléments suivants devrait être donné à la patiente
pour contrôler ses symptômes?
A. Niacine
B. Vitamine C
C. Acide folique
D. La vitamine B12 par voie
intramusculaire
E. Vitamine B12 par voie orale
174. An 83 y.o. man is brought to the ER after a fall )<S "S# ‫ارئ‬2‫ ا‬345 D‫  إ‬83  ‫ أ ر‬B
with trauma to head. He is unresponsive. Head CT [Q‫ ?> ا‬.gJlQ4 K ‫ه‬.‫ اأس‬6 O<‫ ر‬- ‫*ط‬4C
shows a bright concave area sandwiched between J# -* OSK ‫ة‬S* O4 ‫أس ود‬C ‫ري‬F‫ ا‬6*.2‫ا‬
the brain and the skull. The most likely diagnosis .Oll‫ا"غ وا‬
is: :‫ ه‬FJ ]‫آ‬N‫ ا‬EJPQ‫إن ا‬
A. Brain tumor .6L‫ ورم د‬.A
.OJl‫ ورم دي ق ا‬.B
B. Epidural hematoma
.OJ% OJL‫ د‬OQ, .C
C. Hemorrhagic stroke
.OJl‫ ا‬IF ‫ ورم دي‬.D
D. Subdural hematoma
.‫ج ا"غ‬l‫ ار‬.E
E. Concussion

Un homme de 83 ans est amené aux urgences après


une chute avec traumatisme à la tête. Il ne répond
pas. Un Scan de la tete montre une zone concave
brillanteprise en sandwich entre le cerveau et le
crâne. Le diagnostic le plus probable est:
A. Tumeur au cerveau
B. Hématome extradural
C. AVC hémorragique
D. Hématome sous-dural
E. Contusion

102 A
175. A 78 y.o. male presents with a hand tremor at rest, ،‫ن‬4‫" &" ا‬J‫ ا‬6 O‫    ر‬78  ‫ ر‬D
shuffling gait, and difficulty getting started O#vi >? )‫ آ أ‬.6# ‫"ء‬.‫ ا‬6 O#Si‫ و‬،OC5]Q OJ
walking. He also exhibits rigidity with passive ،EJPQ‫آ"  ا‬wQC .OCS‫\اف ا‬N‫ ا‬6 OCST& O‫ آ‬-
motion of his upper extremities. To confirm the :‫؟‬gC2 ‫ف‬,
diagnosis, you would do: .62JF‫ ا‬g[SC O%X .A
A. Peripheral nerve biopsy .6.[S‫ ا‬JiQ‫ ا‬O,‫ درا‬.B
B. Nerve conduction study .EEG .C
C. EEG .‫ص‬FT‫"  ا‬% ‫ "م إاء‬.D
D. No further testing .6TAG‫ و‬64J\&;‫ ا‬J# [ .E
E. Functional MRI

Un homme de 78 ans note un tremblement de main


au repos, démarche en trainant les pieds, et une
difficulté à debuter la marche. Il présente également
une rigidité au movement passif de ses membres
supérieurs. Pour confirmer le diagnostic,
vousferiez:
A. Une biopsie du nerf périphérique
B. Une étude de la conduction nerveuse
C. EEG
D. Pas d'autres tests
E. Une IRM fonctionnelle
176. A 78 y.o. man is brought to ER after he “fainted “at 6L‫" أن أ‬S# ‫ارئ‬2‫ ا‬345 D‫  أ إ‬78  ‫ ر‬C
a funeral. In ER he was alert and oriented. The 6.>‫> و‬.&Q ‫ارئ آن‬2‫ ا‬345 6.‫) أ|&ء &زة‬JC
family did not observe any abnormal jerky JL Ol&Q ‫ أي آت‬OCAS‫) ا‬JC mv 3 ،Š‫ن ذ‬L
movements meanwhile. What is the next best step ‫ا‬b> 6vS‫ ا‬J#"Q‫ ا‬6 N‫ ا‬OJQ‫ة ا‬2P‫ ا‬6‫  ه‬.OJSJ.\
in the management of this patient? :‫؟‬/‫ا‬
A. PET scan .6‫و‬Q‫ز‬.‫"ار ا‬i{# 6S2* [ .A
.">l‫ ا‬EF .B
B. Stress test
.6A#>‫ آ‬gC5 :J2P .C
C. ECG
.OA‫ ا‬O"‫ ا‬OJ‫و‬N‫ [ ا‬.D
D. Pulmonary angiogram
),‫) و إر‬JC 6L‫" أ‬5 /‫ا ا‬b>،‰Jj K .E
E. Nothing; this patient has just fainted and
.)J‫ل دة و‬%&‫ ا‬D‫إ‬
can be sent home now that he has
recovered

Un homme de 78 ans est amené aux urgences après


s’être “évanoui” à un enterrement. Aux urgences, il
est alerte et orienté. La famille n'a pas observé de
mouvements saccades anormaux entre-temps.
Quelle est la meilleure prochaine étapedans la prise
en charge de ce patient?
A. PET scan
B. Test de stress
C. ECG
D. Angiographie pulmonaire
E. Rien; ce patient s’es tjuste évanoui et
peut être envoyé à la maison maintenant
qu'il a récupéré

103 A
177. A 32 y.o. woman is brought to the ER having "J‫" أ‬S# ‫ارئ‬2‫ ا‬345 D‫ت إ‬C‫  أ‬32 ‫ اأة ا‬E
reportedly taken a large dose of heroin. She is >" ،gJlQ4 K 6‫ ه‬.JA‫و‬J>‫ة  ا‬J.‫ آ‬O >‫ &و‬
unresponsive, has small pupils and has a respiratory gl 6Q‫ أي  ا‬.O*J5‫ د‬/ aT 6 aT&Q‫ ا‬O,‫ و‬O5" @J
rate of 6 breaths/min. Which of the following :‫ؤ؟‬2‫ا‬
should be administered? .J#‫ أو‬.A
A. Atropine .‫ل‬Q‫ إ‬.B
B. Ethanol .J&‫ز‬C .C
C. Flumazenil .JWQ4J, JQ,‫أ‬-‫ ن‬.D
D. N-acetylcysteine .‫ن‬4‫ آ‬.E
E. Naloxone

Une femme de 32 ans est amenée aux urgencies; la


famille suspecte qu’elle a pris une grande dose
d'héroïne. Elle ne réagit pas, a des pupilles serrées
et a une fréquence respiratoire de 6 respirations /
min. Lequel des traitements suivants doit être
administré?
A. Atropine
B. Ethanol
C. Flumazénil
D. N-acétylcystéine
E. Naloxone
178. Which of the following does not cause T-wave gC*‫ ا‬:J2P "& T - O 6 ‫ب‬v*‫ ا‬g.4 K 6Q‫ أي  ا‬A
inversion on ECG? :‫؟‬6A#>‫ا‬
A. Hyperkalemia .‫م ا"م‬J,# ‫ ط‬.A
B. Left bundle branch block .‫ى‬4J‫ ا‬O%F‫ ع ا‬gl .B
C. Left ventricular hypertrophy .4N‫ ا‬J2.‫ ا‬3P .C
D. Myocardial infarction .OJ.C5 OC ‫ء‬Q‫ ا‬.D
E. Myocardial ischemia .gC*‫ ا‬OC ‫ر‬T5‫ ا‬.E

Leque lparmi les suivants ne provoque pas


l'inversion de l'onde T surl'ECG?
A. Hyperkaliémie
B. Bloc de branche gauche
C. Hypertrophie ventriculaire gauche
D. Infarctus du myocarde
E. Ischémie myocardique

104 A
179. A 48 year old man presents with epigastric pain. It ‫"أ‬# "* .6,j 3‫   وه   أ‬48  ‫ ر‬C
began suddenly, and reached a constant ache which  ‫ *" أد‬.>?‫ ا‬F nQC I#| 3‫ أ‬D‫ إ‬i‫ و‬DQ ،‫ة‬wl
radiates through to the back. He reports nausea, OF# ‫) أ) آن‬QCA ‫ *" أدت‬.‫>ل‬,‫‰ وإ‬J5 ،‫ن‬J]L ‫"وث‬
vomiting and has diarrhoea. His family noted he "& .I>Q‫" ا‬5 OF‫ ا‬b‫ أن ه‬J. ‫ و‬،‫ أم‬3 b& ‫ن‬5
was jaundice 3 days ago, but this appears to have .6,j 3‫ وأ‬،‫ت‬,S‫ ا‬،‫ارة‬F# ‫ع‬T‫ آن ") ار‬EFT‫ا‬
: ‫ر ه‬N‫ ا‬EJPQ‫ ا‬.## O"5 OJ< ‫ة‬J, )"
resolved. On examination he is pyrexial, is
.‫ د‬O‫"ة دود‬A‫>ب زا‬Q‫ ا‬.A
guarding, and has epigastric tenderness. He has a
.‫>ب ارة د‬Q‫ ا‬.B
past medical history of Asthma. What is the most
.‫&س د‬# ‫>ب‬Q‫ ا‬.C
probable diagnosis
.u‫>ب ا‬Q‫ ا‬.D
a- Acute appendicitis
.gC*‫ ا‬OC ‫ء‬Q‫ ا‬.E
b- Acute cholecystitis
c- Acute pancreatitis
d- Diverticulitis
e- Myocardial infarction

Un homme de 48 ans se présente avec une douleur


épigastrique qui a commencé subitement, et a
atteint un niveau constant irradiant au dos. Il
rapporte des nausées, des vomissements et une
diarrhée. Sa famille a noté qu'il était ictérique
pendant 3 jours, mais cela semble etre résolu. À
l'examen, il est fébrile, avec une defense , et une
sensibilité épigastrique. Il a des antécédents
médicaux d'asthme. Quel est le diagnostic le plus
probable
a- appendicite aiguë
b- cholécystite aiguë
c- pancréatite aiguë
d- diverticulite
e-infarctus du myocarde

105 A
180. A 25-year-old male presents to the clinic for ‫ة‬J, )" .3*S‫ ا‬3JJ*Q ‫دة‬JS‫ ا‬D‫   إ‬25  ‫ ر‬C
evaluation of infertility. He has a life-long history "& .‫رة‬Q‫ ا‬OA‫ت ا‬Q{‫ وا‬uQ&‫ل ا‬S4‫  ا‬OC\ ‫ة‬J
of a productive cough and recurrent pulmonary ،‫>ل‬,‫ إ‬،6&2# 3‫ أ‬- O&% ‫ آ‬D‫ر إ‬j‫ أا<) أ‬OS‫ا‬
infections. On his review of symptoms he has .‫ي‬, ‫  داء‬6S ‫ ه أ‬.‫ ازن‬g4‫ آ‬6 O#Si‫و‬
indicated chronic problems with abdominal pain, EJPQ‫  ه ا‬.‫ت‬.[5 -, ‫ح‬Q* OJS‫"ر ا‬i ‫رة‬i
:‫؟‬FJ ]‫آ‬N‫ا‬
diarrhea, and difficulty gaining weight. He also has
.% ‫ي‬A‫"اد ر‬4‫ ا‬.A
diabetes mellitus. His chest x-ray suggests
.‫ي‬C 64T& ‫ن >ز‬Q‫ ا‬.B
bronchiectasis. Which is the most likely diagnosis?
.64J‫ آ‬nJC .C
a. COPD
.6CX‫ي دا‬A‫ ر‬n% .D
b. Upper respiratory infections
.#‫ ر‬.E
c. Cystic fibrosis
d. Intrapulmonary hemorrhage
e. Asthma

un homme de 25 ans se présente à la clinique pour


évaluation de l'infertilité. Il a une histoire tout au
long de sa vie d'une toux productive et d’infections
pulmonaires récurrentes. A la revue de ces
symptômes, il a des problèmes chroniques de
douleur abdominale, de diarrhée, et une difficulté à
prendre du poids. Il a aussi un diabète sucré. Sa
radiographie pulmonaire suggère une
bronchectasie. Quel est le diagnostic le plus
probable?
a. BPCO
b. infections des voies respiratoires
supérieures
c. Fibrose kystique (mucoviscidose)
d. hémorragie intrapulmonaire
e. Asthme

106 A

Vous aimerez peut-être aussi